柳沢って誰だ?

このエントリーをはてなブックマークに追加
1名無しさん@お腹いっぱい。
日本人の医者で、ノーベル賞に最も近い人だそうだが、詳細キボーン!
2既出 :2000/10/19(木) 06:52
医学部卒のノーベル賞受賞者はいない!
http://cocoa.2ch.net/test/read.cgi?bbs=hosp&key=971260169&ls=50
47 名前:ほう。 投稿日:2000/10/15(日) 22:00
>45
既出かもしれませんが、エンドセリンの柳沢氏は
東大第二薬理の教授を蹴ったそうですね。
まあ、現職がテキサスの教授では当然ですかね。
3名無しさん@お腹いっぱい。 :2000/10/19(木) 08:06
さっかー少年へなぎ。
4名無しさん@お腹いっぱい。 :2000/10/19(木) 11:06
age
5名無しさん@お腹いっぱい。 :2000/10/21(土) 03:35
アメリカの大学で主任教授までなったのは柳沢さんと利根川さんだけでしょ。
6名無しさん@お腹いっぱい。 :2000/10/21(土) 03:40
7>5 :2000/10/21(土) 04:12
ハワードヒューズのという但し書きが付けばその通り。ただし、日系二世のジョセフ=高橋氏が最近なったので、その人を含めると3人。
8名無しさん@お腹いっぱい。 :2000/10/21(土) 04:36
柳沢敦と親戚?
9MY :2000/10/21(土) 07:59
わはははは〜。いろいろとお褒めの言葉をいただき、大変光栄です。
その張本人でございます。どうぞよろしく。m(__)m
アメリカの大学で教授になっている日本人なら、
医学生物学系だけでも結構沢山いると思いますよ。
ハワードヒューズでも、日本生まれは利根川さんと私だけだと思いますが、
日系2世なら既出のJoe TakahashiやMitzy Kurodaがいます。
10田舎脳外科  :2000/10/21(土) 08:40
>9
柳沢先生?
なんかわからんが愉快な人のようだ。
11>9 :2000/10/21(土) 08:56
柳沢先生も2chらーだったとは・・・。
12>8 :2000/10/21(土) 09:03
義理兄弟らしいよ。
13柳沢先生の友人 :2000/10/21(土) 13:43
すみません。柳沢先生にこの掲示板を教えたのはわたしです。
まさか書き込んでしまうとは思いませんでした・・・かなり責任を感じています。
先生は長くアメリカに住んでいらっしゃるので
にちゃんねるや日本のネット事情についてわかってらっしゃらないと思います。
ご本人は大変素直で純粋な方なので、よろしくお願いいたします。
14名無しさん@お腹いっぱい。 :2000/10/21(土) 14:49
本人か?それはすごい!
質問ある人はしてみたら?ただし学術的内容に限る。
15名無しさん@お腹いっぱい。 :2000/10/21(土) 15:52
世の中分からないものだね(笑)
でもなんか興奮してきた。また来てくれるといいな。
16名無しさん@お腹いっぱい。 :2000/10/21(土) 16:12
だれか、あのアドレスへメールを送った奴はいるのか?
17名無しさん@お腹いっぱい。 :2000/10/21(土) 17:15
>16
本人だよ。くれぐれも煽ったりするなよ。
18MY :2000/10/21(土) 18:11
はいはい。書き込みしてから、ひょっとしてヤバかったかなと気づきました。
13の友人氏にも注意されたし。
まぁでも、俺のアドレスなんて、半分パブリックだからいいや。見つけようと思えばすぐ見つかるし。
というわけで、本人は結構こうして暇人なのです。
暇でいられることが、アメリカの研究環境の強みとも言えるでしょう。

何でも質問してくれ。特に医学生・業界人の人!
19名無しさん@お腹いっぱい。 :2000/10/21(土) 18:15
>18

素晴らしい!
本物だったら、2CHから世界が覗けるかもしれん。
20名無しさん@お腹いっぱい。 :2000/10/21(土) 18:18
超感激。質問考えます。
21名無しさん@お腹いっぱい。 :2000/10/21(土) 18:19
>19
だから、マジで本物だって!!
22名無しさん@お腹いっぱい。 :2000/10/21(土) 18:50
エンドセリンはSAHのときの遅発性脳血管攣縮の主因として
注目されていますが、こうした疾患への抗エンドセリン療法
の可能性についてどう考えられますか?
特異抗体が有効なのでしょうか?
23学生です :2000/10/21(土) 19:04
卒業後は精神科もしくは脳科学の分野に進んで、
精神疾患の発現機序について研究したいと考えています。
日本を代表する学者、例えば光通信の東北大学西沢元学長や
インターロイキン6の大阪大学岸本総長などは、
「皆と違うことをやりつづけなさい。」と著書などに
書いているのですが、先生はどうお考えですか?
脳+ゲノムなら皆がとっくに始めていることで、
今からやってもスタートラインにすらたどりつかないのでは、
と思ってしまいます。すみません、くだらない質問で。
24MY :2000/10/21(土) 19:05
>22 早速のまじめな質問、ありがとう。

くも膜下出血後の脳血管攣縮は、たしかに抗エンドセリン療法の適応として注目されるものの一つです。
いま製薬業界が追求しているのは中和抗体ではなく、エンドセリン受容体拮抗薬です。
うっ血性心不全、高血圧、肺高血圧などを適応として、現在、数社がPhase IIまで行っています。
残念なのは、SAHは心不全や高血圧と比べると患者数が圧倒的に少ないので、
いわゆるオーファン疾患になってしまっている点です。
でも、いったん別の疾患で拮抗薬が認可されれば、当然SAHへの臨床試験も行われるようになるでしょう。
少なくとも、動物のSAHモデルでは、ものすごく効きます。
25田舎脳外科 :2000/10/21(土) 19:06
文献よみたいけどみつけきれん!
だれか key word のスペルおしえて!
我ながらなさけない。
26名無しさん@お腹いっぱい。 :2000/10/21(土) 19:10
>少なくとも、動物のSAHモデルでは、ものすごく効きます。
ものすごく期待しています。
手術のうまくいった患者が、朝にこにこしていてその日の晩に
亡くなったのをICUで経験しました。あれが防げればSAH治療の
未来は明るい。
27MY :2000/10/21(土) 19:11
そうそう、日本人(日系人)のノーベル医学生理学賞候補といえば..
私などは、そんなことをmentionするのもおこがましいくらい遠いですが、
前出のJoe Takahashiさんなどは、時計遺伝子の発見ということで、マジで近未来の候補だと思います。
彼は、シカゴのNorthwestern Universityに居ます。気さくなオッチャンです。本当にいい人だよ。
28名無しさん@お腹いっぱい。 :2000/10/21(土) 19:17
22です。ありがとうございます。
エンドセリンはかなり長い作用時間を持っていると聞きました。
受容体拮抗剤では、すでに放出されたエンドセリンの作用を阻害するの
は難しいのではないでしょうか?
フリーラジカル拮抗剤の、抗ショック治療研究も同じ理由で頓挫していますが。
それが有効であるならば、エンドセリンはある程度持続的に放出
されつづけて局所効果を維持していると考えられるのでしょうか?
29名無しさん@お腹いっぱい。 :2000/10/21(土) 19:22
endothelinだったかな〜。
30MY :2000/10/21(土) 19:24
>23 いえいえ、本当にくだらない質問は、聞かなかった質問、って言うじゃないですか。

精神疾患の分子機序は、まさに来世紀の花形分野になると言ってもいいのではないでしょうか。
今からでも全然遅くないでしょう。分裂病のゲノム解析など、まだまだ緒にもついていない状態だし。
俺も最近、ちょっとしたラッキーな出来事から、ナルコレプシーの原因の発見に貢献することが出来ました。
よかったら、10月5日号のNatureにニュース記事が出ていますので、見てみて下さい。
脳科学はほんとうに面白いと思いますよ。頑張ってくれ!
31名無しさん@お腹いっぱい。 :2000/10/21(土) 19:30
エンドセリンについてよく知らない奴はここでも読んでおけ。
ttp://dasnet02.dokkyomed.ac.jp/ksap/endo111.html
3223 :2000/10/21(土) 19:38
ありがとうございます!!!
世界のYanagisawaからコメントを頂けるなんて最高です!!!
Nature10月5日号に先生の顔写真入りで記事がありました。
ハンサムですね。これからメインテナンスで全館停電に
なるので記事をコピーして拝見します。
本当にありがとうございました!!!
33舐め次 :2000/10/21(土) 19:39
クッキーオーン
34MY :2000/10/21(土) 19:40
>28

エンドセリンの長時間作用は、受容体に長く居座ることと持続的放出との両方が絡んでいる、ということになっています。
いま臨床試験中の受容体拮抗薬は、いったん受容体に結合したエンドセリンをも追い出すことが出来るのです。
だからこそ、モデル実験でも効くのでしょう。
フリーラジカルの場合は、作用機序が化学的に不可逆で、しかもすごく速いので、その辺難しいのでしょうね。

スペルはendothelinです。ただし、他のキーワードと掛けないと数千件出てきます。
35名無しさん@お腹いっぱい。 :2000/10/21(土) 19:56
私の友人は研究でアメリカへいったままもう6年になります。
いくつか成果を上げ論文もモノにしてますが、まだ帰ってくる気は
ないようです。アメリカの研究環境は日本と比してどう良いのでしょうか?
36>35 :2000/10/21(土) 20:11
>>18 に書いてある。
37名無しさん@お腹いっぱい。 :2000/10/21(土) 20:25
本物だ〜 しかもレスしてくれてる。感激&不勉強で質問すら思いつかない・・
皆さん、くれぐれも煽ったりせずにこのスレッド維持しましょう。
またレスしていただけるかもしれません。
38名無しさん :2000/10/21(土) 20:28
>37
ぼくもおなじことを書き込もうとしていました。
みなさん大事にしましょう。
39眠たい大学院生 :2000/10/21(土) 20:37
実験の待ち時間にこのスレッド発見。
目が覚めました。
40名無しさん@お腹いっぱい。 :2000/10/22(日) 01:18
東大でノックアウトマウスを先に論文発表した件なんかはどう思ってるのかね。
久利腹くんあたりは大きな顔をしているようだが。
41MY :2000/10/22(日) 01:46
>40 おっ、詳しい人ですな?

しかし、彼とは今でも親しい友人であり仲間です。確執はぜんぜんないですね。
エンドセリンのKOに関しては、彼の方がずっと早くから着手していたし。流石ですよ。
それに彼、別に大きな顔してないと思いますよ。今回の教授就任、心からおめでとうと言いたいです。
42名無しさん@お腹いっぱい。 :2000/10/22(日) 01:49
ところでご本人にここを教えたのは、やっぱり筑波大関係者ですか?
43MY :2000/10/22(日) 01:51
35>

う〜む、その人の場合、単に今している仕事がエキサイティングだから、帰りたくないのかも???
俺はアメリカの環境は大好きですが(だからこそここに居る)、研究環境と言っても話題として広うござんす。
どんな側面が聞きたいですか?
44MY :2000/10/22(日) 01:56
42> いいや、筑波とは関係のない仲間が、たまたま。
45田舎脳外科 :2000/10/22(日) 02:42
エンドテリンなんて虚血屋の先輩が文献よんでたのを
ぼーっとっきたことがあるだけのわたし。
いま一杯のんだあと夜中の医局で必死こいて文献こぴーしまくり。
>MY先生 明日から当直あんど学会出張なんで
4日後に質問させて下さい。いまから泥縄で勉強します。(めっちゃ失礼)
同門に若い虚血屋の脳外科医がいっぱいいますけど
例えば8万ドルほど貯金があったら、2〜3年ほど
そちらで勉強させていただくことは可能ですか。
MDとっとく必要はありますか。
46名無しさん@お腹いっぱい。 :2000/10/22(日) 02:49
47Y.T :2000/10/22(日) 05:05
ホンモノの柳沢先生ですか??
2ch見ててよかった〜!
また、とても気さくな若々しい印象で感動しました。
ところで、研究環境はとてものびのびとしていらっしゃる
ようですが、研究以外のお時間で、最近はまっている趣味は
どのようなものがありますか?
48電波妨害 :2000/10/22(日) 06:04
>柳沢先生
こんにちは。
うーん、いっぱい有りすぎてどれから読んでけばいいのかわからないです。

・Ishikawa T@` Yanagisawa M@` Kimura S@` Goto K@` Masaki T. Positive inotropic action of novel vasoconstrictor peptide endothelin on guinea pig atria. Am J Physiol 1988;255:H970-H973
・Yanagisawa M@` Inoue A@` Ishikawa T@` Kasuya Y@` Kimura S@` Kumagaye S@` Nakajima K@` Watanabe TX@` Sakakibara S@` Goto K@` Masaki T. Primary structure@` synthesis@` and biological activity of rat endothelin@` an endothelium-derived vasoconstrictor peptide. Proc Natl Acad Sci USA 1988;85:6964-6967
・Ishikawa T@` Yanagisawa M@` Kimura S@` Goto K@` Masaki T. Positive chronotropic effects of endothelin@` a novel endothelium-derived vasoconstrictor peptide. Pflugers Arch 1988;413:108 110
・Li L@` Ishikawa T@` Miyauchi T@` Yanagisawa M@` Kimura S@` Goto K@` Masaki T. Pressor response to endothelin in guinea pigs. Japan J Pharmacol 1989;49:549-552
・Goto K@` Kasuya Y@` Matsuki N@` Takuwa Y@` Kurihara H@` Ishikawa T@` Kimura S@` Yanagisawa M@` Masaki T. Endothelin activates the dihydropyridine-sensitive@` voltage-dependent Ca2+ channel in vascular smooth muscle. Proc Natl Acad Sci USA 1989;86:3915-3918
・Kasuya Y@` Ishikawa T@` Yanagisawa M@` Kimura S@` Goto K@` Masaki T. Mechanism of contraction to endothelin in isolated porcine coronary artery. Am J Physiol 1989;257:H1828-H1835
・Miyauchi T@` Tomobe Y@` Shiba R@` Ishikawa T@` Yanagisawa M@` Kimura S@` Sugishita Y@` Ito I@` Goto K@` Masaki T. Involvement of endothelin in the regulation of human vascular tonus: potent vasoconstrictor effect and existence in endothelial cells. Circulation 1990;81:1874-1880
・Tomobe Y@` Ishikawa T@` Yanagisawa M@` Kimura S@` Masaki T@` Goto K. Mechanisms of increased sensitivity to endothelin-1 in aortic smooth muscle of spontaneously hypertensive rats. J Pharmacol Exp Ther 1991;257:555-561
・Ishikawa T@` Li L@` Shinmi O@` Kimura S@` Yanagisawa M@` Goto K@` Masaki T. Characteristics of binding of endothelin-1 and endothelin-3 to rat hearts: developmental changes in mechanical responses and receptor subtypes. Circ Res 1991;69:918-926
・Yamanaka A@` Ishikawa T@` Goto K. Endothelium-dependent relaxation independent of NO@` prostaglandins and epoxyeicosatrienoic acids in guinea pig coronary artery. J Pharmacol Exp Ther 1998;285:480-489
石川智久、柳沢正史、後藤勝年、真崎知生 エンドセリン 臨床科学 25巻
内皮由来血管収縮ペプチド・エンドセリン 日本臨床 47巻 p. 2121 - 2130

49田舎脳外科 :2000/10/22(日) 08:52
>48
なんか大昔の文献が多いな。
てっとりばやく理解するには最近のreviewがいいでしょ。
って俺も思って探したら何故か肝心のreviewの号だけ
医局に無いや内科医!
とりあえず三つゲット。いまからよむぞー。
50MY :2000/10/22(日) 10:37
>田舎脳外科さま

留学希望ですか?(ここは求人サイトだったのか?)
いやいや、やる気のある方なら、歓迎しますですよ。
現在、うちのラボには、日本人が6名いて、そのうち半数は医学部出身者です。
51MY :2000/10/22(日) 10:49
>文献

たしかに48にリストされてるのは、かなり古いやつばっかですねえ。
面倒なので、大学の俺の紹介ページにある論文のIntroduction等から、孫引きしてみて下さいな。

http://swnt240.swmed.edu/gradschool/webrib/yanagasa.htm
52名無しさん@お腹いっぱい。 :2000/10/22(日) 11:09
98年と99年にcell、2000年にNature・・・
世界が違う・・・・。
ちなみに私は旧帝大の基礎系。
53MY :2000/10/22(日) 11:10
>47 趣味ですかぁ。ここへの書き込み?(爆)

冗談さておき、ここんとこカメレオンに凝ってたんですが、ごく最近死んでしまったんですよね。
雌だったのだが、どうも小鳥のように卵が腹につまってしまったらしいのです。
あれは面白い動物だぁ。今度は雄にします。
54馬鹿医学生 :2000/10/22(日) 12:27
血管収縮物質:エンドセリン、トロンボキサンA2
血管拡張物質:プロスタグランディンI2、ブラジキニン、Nitric oxide
国試勉強で出てきました。
覚えにくいっす。何かいい語呂合わせありますか?

あまり学生の勉強量を増やすような研究しないで下さい。
55Y.T :2000/10/22(日) 13:16
わー、趣味のお話まで教えてくださってどうもありがとうございます。
カメレオンは、おもしろいといいますね。私も動物はいろいろ
飼ってますので、いつかチャレンジしてみたいところです。
文献拝見しますと、先生は非常にお若い方ですよね。
自分もこうしてはおれない・・と焦りに似たいい刺激を受けました。
ちゃんと勉強した上で、また質問させていただきます。
56名無しさん@お腹いっぱい。 :2000/10/22(日) 13:21
>54 自分で考えなさい。ったく、先生を捕まえてなんて質問を。
>先生 非礼をお詫びします。なにかあったら、ここの医者みんな
でフクロにするので、どうぞご寛容に。
>ALL みんな、このスレッド大切にしよう。
57名無しさん@お腹いっぱい。 :2000/10/22(日) 13:36
先生、オレキシンってどうなんですか?
拒食症に効くって本当ですか?
58名無しさん@お腹いっぱい。 :2000/10/22(日) 15:10
>57
調子にのんな
まずは自分で調べろ
そんな調子だからこの板から良心的なコテハンがいなくなったというのに
59名無しさん@お腹いっぱい。 :2000/10/22(日) 15:58
ああ、楽しい日曜の朝だった。先生も(爆)って書くんですね。
テキサスはきっとこれから土曜日の夜なのでしょうね。
6040 :2000/10/22(日) 17:20
そうか、やっぱりこれは本物か。半分疑っていたのは疑い深すぎるかな?
でも、臨床と研究の合間に2chなんて見ているのは教室の皆には内緒だったんだけどね。
面白いことが起きるもんだ。やっててよかった。
実は、他にも結構有名な先生方がPubMedをロードする合間にここを見ていたりして(^_^)
61>1 :2000/10/22(日) 20:35
感想はいかが?
621じゃないけど。 :2000/10/22(日) 21:19
もう、15年くらいになるのか。
俺は筑波じゃないけど、まだ院生だった先生を
うちの薬理の教授が呼んできて、エンドセリンの
特別講義があった。
学生時代を思い出した。
63MY :2000/10/22(日) 21:19
>55  1960年生まれでございます。
>57  まずは51のリンクあたりからどうぞ。

今ダラスは日曜の朝7時半。面白い質問、どしどししてくれ!
64名無しさん@お腹いっぱい。 :2000/10/22(日) 22:38
>63 いやはや筑波大?いやいまや日本を代表するスーパースターの方がこんな場所にいらっしゃって感動してます。


ということで、私も質問。
先生のラボのあるYビルのエレベータの扉の色は?
同じフロアにノーベル賞の方がいらっしゃると思いますが、その方のお名前は?
65名無しさん@お腹いっぱい。 :2000/10/22(日) 22:42
>64
馬鹿野郎。詐称喚問なんかするな。失礼だぞ。
何度も書いてあるけど本物だって。
661 :2000/10/22(日) 22:44
>65
馬鹿が偽者相手にまんまとダマされてるよ
67名無しさん@お腹いっぱい。 :2000/10/22(日) 22:45
>64
昨日からずっと見ていますがどう考えても本物です。
68名無しさん@お腹いっぱい。 :2000/10/22(日) 22:53
柳沢先生
 アメリカでポスドクをしている者です。医者ですが研究職で生きていこうと考えています。
米国での研究生活は快適ですが、英語のリスニングが今一つです。
柳沢先生の場合は、何か特別な教材をお使いだったでしょうか?
サイエンスと直接関係ない質問で申し訳ありません。
691 :2000/10/22(日) 22:59
>67
素人厨房が分かったような口を・・
70MY :2000/10/22(日) 23:26
>64
銀色。
塗ってない。ステンレスかなんか。
名前は、L5にいるのがJLGとMSB。K5にいるのがAG。Y4にいるのがJD。

>1
自分の目でみたことだけを信じるのは立派なことだ。
そこから情報を導き出すのも重要だぞ。

>68
特に無い。
自信が無いなら、大学付属のESLで学んではいるもよし。
物価が安くて、気候の似ている、シアトルあたりで
語学だけ習った後で、アクセプトのとれたところにいくのもいいんじゃないか?

腹が減った!飯くうぞ!


711 :2000/10/22(日) 23:28
>70
おまえ、筑波の学生だろ
72名無しさん@お腹いっぱい。 :2000/10/22(日) 23:30
京都大学講師からアメリカですぐにPIがとれたとお聞きしましたが、
どのような経緯があって、そのようなjumpできたのでしょうか?
もしよろしければお教えください。
私の先輩はアメリカでNature 2報書いたのにまだPIになれません。
7368 :2000/10/22(日) 23:37
柳沢先生
御返事ありがとうございました。
それから引用最高栄誉賞の御受賞 おめでとうございます。
74MY :2000/10/22(日) 23:48
>1
俺は筑波の学生か?

>72
わからん。

>68
表彰式、行くんだよねー、来週。
秋葉に行くのが超☆楽しみ
招待 cha cha cha(爆)
75名無しさん@お腹いっぱい。 :2000/10/23(月) 00:14
かつてないパターンの「悲惨な1」のスレになるかも?
76本物のMY :2000/10/23(月) 00:14
困った困った。本当に偽物が登場しました。見てもおわかりの通り、70と74は明らかに偽物です。
しかし他は全部本物です。今時間がないので、詐称検証についてはまた後ほど書きます。
今週末からISIの授賞式に行くというのは本当です。
77名無しさん@お腹いっぱい。 :2000/10/23(月) 00:16
>75
1は自分の作業仮説にどこまでこだわれるんだろうな、もっと笑わせてくれ。
78独身Dr :2000/10/23(月) 01:14
このスレッド、すごすぎるな。
79柳沢大先生へ :2000/10/23(月) 01:23
39 名前:36 投稿日:2000/10/18(水) 18:51
ノーベル医学生理学賞は、医学・生物学の分野で教科書を塗り替える
ような基礎的な仕事をした人に与えられる。ラスカー賞受賞者の6割が
ノーベル医学生理学賞を受賞すると言われており、ラスカー賞受賞者は
ノーベル賞の有力候補。今回のカンデルもだいぶ前にラスカー賞を受賞
しており、候補には挙がっていたが、なかなかノーベル賞を取らなかっ
たので、もう無理だといわれていた矢先に受賞した。日本人では昨年
増井というおっさん(ノンエムディー)がとっており、この人が現在
日本人のノーベル医学生理学賞の最有力候補。ちなみに、ラスカー賞に
は基礎部門と臨床部門があり、臨床部門での受賞者は、ノーベル賞には
縁が無い。ヘリコバクターピロリの人もラスカー賞を受賞したが、
臨床部門なのでノーベル賞はとらないだろう。それから、ラスカー賞の
選考委員長がゴールドシュタインで、ノーベル賞の選考委員は毎回組織
されて厳密に固定されていないが、事実上、ゴールドシュタインが最高
実力者と言われており、ゴールドシュタインに気に入られるかどうかが
ノーベル賞受賞に大きく響いてくる。柳沢氏はゴールドシュタインの
愛弟子で、相当気に入られており、それらを含めても、日本人の医者で
は柳沢氏が最短距離と考えられる。
http://mentai.2ch.net/test/read.cgi?bbs=doctor&key=971453403&ls=50

↑のような意見がありますが、どうでしょうか?
期待したいのですが。
80元祖ポスドク@海外生活 :2000/10/23(月) 01:33
ゴールドシュタインて誰?
81>80 :2000/10/23(月) 01:43
調べろスイスブラック
82>柳沢先生 :2000/10/23(月) 01:52
好きになってしまいました。
先生のその知的な所が魅力的です。
83To Prof. Yanagisawa :2000/10/23(月) 01:56
外科医です。
日本では医学部から基礎へ進む人が少ないとおもいますが、
アメリカではどうなんですか?
8482です :2000/10/23(月) 02:05
いきなりこんな事を書いた私は軽蔑されて当然です。
わかっています。でもこれだけは本当です。
先生が好きです。
85MY :2000/10/23(月) 02:10
既婚です。
8682です :2000/10/23(月) 02:33
>85
怒ってらっしゃいます…?
8782です :2000/10/23(月) 02:39
>柳沢先生
メール送ったら迷惑でしょうか?
88真面目な研究者 :2000/10/23(月) 02:55
変なカキコは辞めようよ。
真面目な質問しよう。貴重なスレじゃないか。
研究関係ではない人は遠慮して下さい。
8982です :2000/10/23(月) 03:54
>88
でも好きなんです…
90名無しさん@お腹いっぱい。 :2000/10/23(月) 04:41
>真面目な研究者
>研究関係ではない人は遠慮して下さい。

随分な意見だな。君の論法だとMDでないと、この板の閲覧や書き込みも
出来ないことになるが。君は柳沢先生にどういう質問をするのかな。
その前に、もちろんMDだろうね、君は。
91真面目な研究者 :2000/10/23(月) 04:51
>90
82などの書き込みに対してです。
御気に障ったのであれば、謝罪します。
92:2000/10/23(月) 05:58
先生、日本にいらしたら何を召し上がりたいですか?
考えておいてくださいね。
このスレッドを紹介してしまった責任を取らせて頂きます<(_ _)>
93本物のMY :2000/10/23(月) 07:28
いやはや、さすが2ch、トラフィックが凄いですなぁ。数時間仕事してる間に収拾つかなくなってるではないか。ではまず、詐称問題から...

ここに来てる人間のインテリジェンスを持ってすればすぐ判ると思うが、この掲示板の中だけで俺が「彼」であることを証明するのは絶対に不可能なのです。最初からそれは判っていたので、あえて実メールアドレスを公開したのです。

まずは、51で紹介した、大学の俺の紹介ページへ行ってみてください。そこには、俺のここでのメールアドレスがそのまま出ています。さらに、そこにリストしてある最近の論文(例えば1999年Cell paper)を引いて、corresponding addressを見てみてください。そこにも、俺のこのメールアドレスが出ています。つまり、このアドレスは、「彼」が日常業務で使っているアドレスであることは疑いようがないでしょう。

あとは簡単。このアドレスにメールして、「彼」から、確かにここに書いたよー、っていう返事が貰えれば疑問は解消するわけです。または、AOL Instant Messengerを使って、「彼」がログオンしているときにIMを送ってやって下さいな。ついでに、紹介ページにはオフィスの電話番号まで出ていますから、「彼」に電話してみたらどうですか?

実際に、メールないしはIMして来た方が、今のところ二人おられます。二人とも納得されたようです。

...と、ここまで書いてきた所で、64の方に一言。そのやり方は、あまり利口ではありませんね。まず第一に、上述の通り、そんなことをいくら訊いたところで、俺が「彼」である証明は得られないのです。現実に、偽者が70で全て正解を出しているじゃあないですか。第二に、そんなローカルなことを聞いたら、貴方がウチの大学(UT Southwestern)の人間か、以前ウチの大学に居たことのある人間だって判ってしまうじゃないですか。つまり、俺は貴方を個人的に知っている可能性が大いにあるのです。もし俺が本物の「彼」だったら、ひょっとしたら気まずい事になってしまうとは考えなかったですか?

以上。詐称問題に関しては、もうこれ以上は書きませんので悪しからず。
94名無しさん@お腹いっぱい。 :2000/10/23(月) 07:51
アメリカでサバイバルするために、一番重要な事はなんですか?
プレゼンテーションの技術を身につける事ですか?
グラント申請のためのライティングの技術ですか?
勿論、両方とも重要だと思いますが。

現在、アメリカでポスドクしています。
95名無しさん@お腹いっぱい。 :2000/10/23(月) 07:55
さすがです、柳沢先生。
この板ではこうして収拾のつかなくなることが往々にして
ありますが、93のようにスマートに整理することもできる
んですね。
>ALL このスレ、大事にしようじゃありませんか。
私はまもなくオペ入りますので、質問考えてまた
来ます。
96本物のMY :2000/10/23(月) 08:13
>68 英語の問題ですね..

正式な英会話のレッスンというのは(中高大学の授業以外)俺は受けたことがないし、自己学習用の教材というのも使ったためしがありません。

俺の場合、ラッキーだったのは、高校の時、生意気なフュージョンのバンド(ああ、懐かしい用語だぁ)を組んでいて、そのバンドを通じてアメリカ人の留学生(たしか、上智のベース弾き)と仲良くなれたことです。あと、大学3〜4年生の時、当時筑波の医学に外国人講師として来ていた英国人の生化学者が、全くのボランティアというか彼の趣味で、「英語で何でも語り合おう」みたいな課外セッションを週数回夕方やってくれて、それに参加したことですね。

そんなこんなで、使っているうちに使えるようになった英語でした。でも、本当に英語が進歩したと自分で思えるのは、やはりアメリカにPIとして来てからですね。なにせ、最初にやらなきゃならなかったのが、秘書と技官の採用面接ですから。

あまり役に立たない回答で申し訳ない。やはり何と言っても、折角アメリカに居るのだからアメリカ人と(ラボの中でも外でも)喋りまくるというのが一番なのでは?
97元祖ポスドク@海外生活板 :2000/10/23(月) 08:41
俺も燃えてきた!
やるぞ〜!!
98本物のMY :2000/10/23(月) 09:06
>72 とうとう出ましたね、この質問。

まずご質問の後半に対する結論からいいます。アメリカでPI(研究室主宰者)になるには、良い雑誌に論文を載せるだけではダメです。少しキツイ物言いになりますが、思いつく条件を挙げると:

1.論文に述べた実験を計画・遂行したのみならず、根本的なアイデアが本人のものであること。有名なボスの下で彼(彼女)のアイデアに従って論文の主著者になっただけでは、何報あろうがダメです。
2.そのことを人に十分に納得させられるだけのプレゼンテーション能力。英語力は最重要。
3.それを高く正当に評価してくれる目上の人間(サポーター)の存在。もちろんパワフルな人ほど良い。
4.何といっても、同じ機関にとどまらず、米国中を廻ってでも自分を売り込むアグレッシブさと「背水の陣」の精神。日本の出身教室なんかを「安全弁」と思っているようではダメです。


で、ちょっと長くなりますが、少し自分史を復習してみようと思います。

俺の場合は、やはりラッキーな出来事の連続だったと思います。1990年にエンドセリンB受容体のクローニングを発表しました(筑波の4年後輩で、現同大助教授の桜井君の仕事です)。それが、京大中西研のエンドセリンA受容体の論文と一緒にNatureに載り、その号に英国のDr. Sir John Vane(アスピリン作用機序とプロスタサイクリンの発見でノーベル賞)がNews and Viewsを書いたのです。そのNews and Views記事が一風変わっていて、当時筑波の講師になりたてだった俺の個人史にずいぶんと触れたものでした。その論文が出る直前に開かれた「国際エンドセリン会議」の時、John Vaneに誘われて二人で一緒に喰った昼食が、実はそれ用の取材だったことは後から気付きました。

その記事が、今いるデパートメントのヘッドであるDrs. Joe GoldsteinとMike Brown(LDL受容体でノーベル賞)の眼にとまり、マドリッドで彼らの主催する小さな研究会で発表する機会を与えられました。そこでの発表の次の日に、ディナーの時に丁度Joe Goldsteinの隣に座ることになり、いろいろ話をしたのです。当時、俺の筑波でのボスは現国立循環器病センター研究所長の眞崎知生元京大教授でした。彼のおおらかな指導のもと、俺は日本で決してunhappyではありませんでしたが、やはり本当にfinancially independentになって独り立ちするには、日本ではまだ何年も時間がかかりそうでした。その辺の事情を話し、もしチャンスがあればアメリカで仕事をしたい旨Goldsteinに言いました。実際、俺は当時アメリカで職を探し始めており、ハーバードのMGHやGenentech社からもオファーは来ていたのです。すると、Joeは、「お前ならHHMI Investigatorにしてやるから、ダラスに来い」と、いとも簡単に言いました。最初は半信半疑でしたが、実際1週間後にJoeから手紙が来て、いろいろあって結局ダラスへの就職が決まったのです。最初からAssociate Professor & HHMI Associate Investigatorという、破格のオファーでした。決まった時、すでに眞崎教授には京都に一緒について行く旨約束してあったので、京都経由でのダラス行きとなりました。京都には結局8ヶ月ほどしか居ませんでしたが。

折角書いたこの文章、どこかにセーブして置こうっと。(^^)  ではでは...
99名無しさん@お腹いっぱい。 :2000/10/23(月) 09:19
すごい。
やはり、最初にNatureありきですね。
貴重な経験談ありがとうございました。
100本物のMY :2000/10/23(月) 09:22
>79

自分で言うのもなんですが、79のアナリシスは、てんで甘いですね。いくらJoe Goldsteinに気に入られたからって、それでノーベル賞が取れるわけないじゃあないですか。「教科書を塗り替える」仕事、つまり医学生物学全体に影響するようなパラダイム・シフトを起こすような仕事は、なかなか出来るものではないのです。俺の仕事など、これまでのノーベル賞の範疇にとっくにカバーされていますよ。エンドセリンは、数年前のFurchgottらの内皮由来弛緩因子とNOのストーリーからの延長ですし、オレキシンの話も、それこそ今年の対象である"slow (nonclassical) neurotransmitter"の範疇であると言い切れます。世の中そんなに甘くないよ。
101名無しさん@お腹いっぱい。 :2000/10/23(月) 09:31
みんなの願望ってことで。。
102本物のMY :2000/10/23(月) 09:32
>83

アメリカの医学部(medical school)の出身者で、基礎へ(特に直接)進む学生は、日本以上に少ないです。しかし、アメリカの主要医学部にはM.D./Ph.D.コース(いわゆるMSTP = medical scientist training program)があり、最優秀中の最優秀の学生が入学してきます。現在アメリカで活躍しているM.D.基礎医学者のかなりが、MSTP graduatesです。うちの大学にも、年間平均15人ほどのMSTP studentsがおり、俺のラボでも2人がthesis workをしています。彼らは本当に優秀です。期待したいと思うし、メンターとしての重大な責任も感じています。
103>100 :2000/10/23(月) 09:45
まあ、謙遜されているのでしょう。けど、やはり日本人の中では最有力候補でしょう(また謙遜されるかも知れないので、返答されなくてもいいです)。
10482です :2000/10/23(月) 09:49
>柳沢先生
好きです…
10572 :2000/10/23(月) 09:50
Goldsteinとの出会いはそこから始まるのですね!
まさに運命の出会いですね!
先生の個人史をお伺いして、本当の実力者とは、
「自分で運を引き込む才能がある人」だとつくづく感じました。

アメリカ上陸から、94年の恐るべき3連ちゃんのCellで「ここにYanagisawaあり」、
の最初のゴングがなるまで、果たして順風満帆だったのでしょうか?
私は第2幕をお伺いしたかったのです。というには、アメリカ人ですら、実力があっても、
ポスドクからここに至るまで大変な努力を要すると聞いたからです。

最初からPIですから就労ピザ(H-1B ?)で、グリーンが取れなくても、NIHグラントはすぐに
取れたのでしょうか?もしグラントが取れなかったなら研究室を軌道に乗せるまで、大変だったように思うのですが?
どのようなご苦労があったのでしょうか?
また教室運営での苦労談などお聞かせ願えればうれしいです。

お仕事でお忙しいところ、いろいろ興味深いことに答えていただきありがとうございました。私もこの上の文章を貴重なものとしてセーブしておきます。
ノーベル賞まで突っ走ってください。
もっとも先生にとってはノーベル賞など一つの通過点に過ぎないでしょうが!
106名無しさん@お腹いっぱい。 :2000/10/23(月) 10:09
世界が違いすぎる。
俺は今日も外来患者の山だ・・・・・。
107名無しさん@お腹いっぱい。 :2000/10/23(月) 10:13
これからの長い熾烈なレースを考えると憂鬱になる。
成功すればいいけど、大概のヤツは敗退を余儀なくされる。
自分は、どうだろ?ため息が出る。
先生は凄過ぎる。
108名無しさん@お腹いっぱい。 :2000/10/23(月) 10:17
>106
私も黙々と目の前の仕事をかたずけるので精一杯・・
でもなんだか燃えてきた。このスレいい刺激になりました。
109名無しさん@お腹いっぱい。 :2000/10/23(月) 10:22
Yanagisawa先生の存在は、ほんとに日本の若い研究者や学生の
励み、誇りになります。(先生も十分お若いですが)
なんか、レスからもpowerを感じます。
110>ひろゆき :2000/10/23(月) 10:32
よかったねぇーー!
11182です :2000/10/23(月) 10:33
>柳沢先生
メール送ったらご迷惑でしょうか?
112名無しさん@お腹いっぱい。 :2000/10/23(月) 10:36
>82
だから、やめれって。
このスレを壊すなよ。
113馬鹿医学生 :2000/10/23(月) 10:37
54の語呂を自分で作ってみました。どうぞご賞味下さい。

刑事トロンボ(トロンボキサンA2) the エンド(エンドセリン) で 収まる(血管収縮)
プロのアイツ(プロスタグランディンI2) は ノーブラ(NO、ブラジキニン) 賀来千賀子(血管拡張)

なんかみんなかしこまってますね。
普段の調子こいたカキコはどこへやら。
114本物のMY :2000/10/23(月) 10:38
>105 う〜む、そう来るのではないかと思いました...(^^;;

ダラスに来て最初の2年半(91年終盤〜94年春頃)は、自分でも本当に心配になりましたよ。ラボの物理的セットアップは何とか終え、幾つかのambitiousなプロジェクトに着手したものの、何だか、いつまで経ってもincubation periodから抜け出せない感じでした。しかしまぁ、種をまいて、自分の勘を信じてじーっと我慢して、気長にやって居ればそのうち花は開くものです。94年に、エンドセリン変換酵素の同定と、エンドセリンB受容体およびエンドセリン3遺伝子のノックアウトといった、それまでのincubationが一気に孵化したのです。勇気をだして、ゼロ状態の研究室に最初から馳せ参じてくれた2人の優秀な日本人M.D.ポスドク(現神戸大の江本さん・現京大の細田さん)の弛まぬ努力の成果でした。

初期の苦労で一番キツかったのは、とにかく最初にリクルートした一代目(?)の技官・ポスドク(上記日本人2人以外)・大学院生が、ほとんど皆ことごとくダメ人間だったことです。要するに、アメリカで自分自身のトレーニング経験がなく、人材システムや人間関係の全く異なる日本から来た俺には、当地で良い人材を見分ける眼がぜんぜん無かったのですね。HHMIのルールでは、一度雇うとなかなかクビに出来ないので、彼らに去ってもらうには、ほんとうに時間とエネルギーを費やしました。

ビザは最初からH−1でしたから、NIH等のグラント申請権は問題なかったのですが、実際には全然申請しませんでした。つまり、HHMIからの研究費に完全に頼ってきたのです。お陰様で、HHMIのアポイントメントも、昨年2回目のレビュー・更新を無事終えました。今から考えると、最初からHHMI Investigatorとしてスタート出来たことが、本当に大きかったと思います。
11582です :2000/10/23(月) 10:48
>112
それだけ答えてください!
116to prof. Yanagisawa :2000/10/23(月) 10:48
>HHMIからの研究費
研究費獲得の難易度、金額は、日本の大学と比べて
どうなんでしょうか?
日本で先生がいらした筑波大、京都大ともに国立大学で、
日本では企業とのやり取りはいろいろと制限がありますが、
アメリカではその辺はどうなんでしょうか?
117116 :2000/10/23(月) 10:54
あと、たいへん無知なんですが、先生のいらっしゃる
Howard Hugh Medical Instituteは
国立、州立、私立という範疇ではどれに当たるんでしょうか?
118ノーベル :2000/10/23(月) 10:56
柳沢さんとは分野が異なるのですが,今回の白川氏の受賞を見て,
ノーベル賞を狙ってみようと言う気になりました.
実際,日本の研究機関にいてノーベル賞は,獲れるもんなのでしょうか?
やはりアメリカでないと無理ですか?
119湯川秀樹 :2000/10/23(月) 11:05
>118
「そういうことを考える時点であかんわな」
120>本物のMY :2000/10/23(月) 11:13
大変答えにくい質問かもしれません。
しかしこの2chを眺めてこの手のスレッドがない日はありません。
2chの普遍的な話題でもあります。
非礼は承知の上です。

「学歴」についてのお考えを是非お聞かせ下さい。
121本物のMY :2000/10/23(月) 11:19
>120 「学歴」といっても...

ふむ。どういった側面に関する俺の意見を聞きたいのですか?
Would you be more specific?
122本物のMY :2000/10/23(月) 11:19
>120 「学歴」といっても...

ふむ。どういった側面に関する俺の意見を聞きたいのですか?
Would you be more specific?
123本物のMY :2000/10/23(月) 11:19
>120 「学歴」といっても...

ふむ。どういった側面に関する俺の意見を聞きたいのですか?
Would you be more specific?
124やっぱり :2000/10/23(月) 11:24
>やはり本当にfinancially independentになって独り立ちするには、
>日本ではまだ何年も時間がかかりそうでした。その辺の事情を話し、
>もしチャンスがあればアメリカで仕事をしたい旨Goldsteinに言いました。
>実際、俺は当時アメリカで職を探し始めており、ハーバードのMGHや
>Genentech社からもオファーは来ていたのです。
>すると、Joeは、「お前ならHHMI Investigatorにしてやるから、
>ダラスに来い」と、いとも簡単に言いました。

この辺がアメリカパワーの根本だとおもう。
日本の頭脳流出の典型的パターン。
125名無しさん@お腹いっぱい。 :2000/10/23(月) 11:26
このスレ面白すぎる。
126名無しさん@お腹いっぱい。 :2000/10/23(月) 11:28
>質問です
PIにとって良いポスドクとはどういう人ですか?
ポスドクがPIと良い関係を築くのはどうしたらいいですか?
127120 :2000/10/23(月) 11:48
specificに質問させて頂きます。

1、この掲示板での様々な学歴に関するスレッドを眺めてどういった感想を持ちましたか?
2、学歴コンプレックスはありますか?あったならば自分でどう考えて解決しましたか?
3、東京大学医学部についてどう思われますか?
4、底辺私立医学部についてどう思われますか?(あえて名は挙げないでおきます)
5、大学入学偏差値についてどう思われますか?
128名無しさん@お腹いっぱい。 :2000/10/23(月) 11:51
このスレに卒後の研究者あんまりいなさそうだね・・
129名無しさん@お腹いっぱい。 :2000/10/23(月) 11:53
>127
全然 specificでないよ。先生に小論文でもやらせるつもり?
130名無しさん@お腹いっぱい。 :2000/10/23(月) 11:54
なんか虎の皮を被った気分スレだ
131名無しさん@お腹いっぱい。 :2000/10/23(月) 11:56
>127
意味ないよ。その質問。良い研究出来れば良いのでは?
学歴が良い研究すると思ってるのかな。
132名無しさん@お腹いっぱい。 :2000/10/23(月) 11:56
>130
あんたなんか日本語間違えてないか?
133本物のMY :2000/10/23(月) 11:58
ごめんなさい、126へのレスが、何故かカイロプラクティックの方へ行ってしまいました。
そちらを参照あれ。
134本物のMY :2000/10/23(月) 11:58
ごめんなさい、126へのレスが、何故かカイロプラクティックの方へ行ってしまいました。
そちらを参照あれ。
135>柳沢先生の友人諸氏 :2000/10/23(月) 11:59
俺はネット依存症の精神病理にも興味のある者だが
あらゆる限りの方法で責任を取れよ、フォローしろよ。
136名無しさん@お腹いっぱい。 :2000/10/23(月) 12:01
先生は根っからのサイエンティストなので、追試をかかされません。
レスがだぶっているのはそのためです。
137名無しさん@お腹いっぱい。 :2000/10/23(月) 12:02
後半だけみえてたから学歴スレかって思った、失礼!
138126です :2000/10/23(月) 12:04
>レスありがとうございました。
PIとの付き合い方に今一つ、自身が持てなかったのですが
これでハッキリしました。明日はPIを追いかけ回します。
139名無しさん@お腹いっぱい。 :2000/10/23(月) 12:06
528 名前: 本物のMY 投稿日: 2000/10/23(月) 11:54

>126 あ、これ良い質問!

もちろん、個々のPIのスタイルによって「良いポスドク」の条件はいろいろ違うと思います。だから、これはあくまでも俺自身の基準です。ハードワーキングとか、勉強家(文献に通じている)とか、実験センスがいいとか、当然の事を別として一つだけ挙げれば、やっぱり、「PIを放っておかない」じゃないかなぁ。PIは(少なくとも俺は)寂しがりやです。毎日のように、うるさいくらいに、あーだこーだと議論を吹っかけてくるタイプが俺は好きですねぇ。あと、PIが何か大事なことをsuggestionしたとき、黙って何となく無視されてしまうタイプは、そのうち逆にPIに無視されるようになります。反対意見があるならあったで、ハッキリ言って欲しいですね。

14072 ・105 :2000/10/23(月) 12:18
>Joeは、「お前ならHHMI Investigatorにしてやるから、
ダラスに来い」と、いとも簡単に言いました。

いやはや、すごいとしかいいようがないですね。
あの利根川さんですら、HHMI取得まで、長い助走が必要だったのに、
アメリカに上陸してすぐに取れてしまうところは、たんなるラッキーだけではないと思います。

トロント大学の増井さんとお話したとき、イエールに留学される前は日本では
(甲南大学)、悲しいことに科研費が全くもらえず、おまけに付属校の実習に
まで駆り出されて研究の時間などなかった、アメリカにきてようやく研究でき
るようになり、あの国へ帰るくらいならば、こちらで歯を食いしばって研究を
した方がずっと報われると言う一念でやってきた、と伺いました。
日本で30代で独立できるなんてよほどの例外がなければほとんどあり得ません。
仮に独立しても、グラントが取れず、干からびてしまうという話を聞いたことがあります。

日本では筑波大学の白川先生がノーベル賞を受賞されてわきかえっていますが、
日本を舞台にしたサイエンスの分野での受賞は、湯川(正確にはコロンビア大学)、
朝永、福井、白川先生ですが、湯川、朝永、福井先生のお仕事は40年代後半から
60年代での仕事で、実にこの30年間の実績ではノーベル賞は白川先生しかいな
いのです。
しかも湯川、朝永、福井先生のお仕事は実験科学ではなく、理論物理、理論化学で、
研究費がなくとも、コンセプト一つで紙と鉛筆で到達できた領域です。

日米をまたいで、研究を続けてこられた経験から、日本のサイエンスのあり方に関して
お伺いしたいです。
研究費も飛躍的に増額し、研究環境はアメリカに比べてそれほど見劣りがすることはなく
なりましたが、それにも関わらず、未だにノーベル賞クラスのパラダイムシフトに繋がる
ような草分け的な仕事は日本では少ないと思いますが、これは日本の研究システムや人材
登用のどのようなところに問題があるとお考えですか?
またどのように変革すれば日本も英米に負けないサイエンス立国として浮上できると考えられますか?
141>140 :2000/10/23(月) 12:20
あなた、どのスレでも同じ事書いてるね。
増井さんと直接話したのどうの。
142>ひろゆき :2000/10/23(月) 12:34
このスレの文章の著作権を確認しておくように。
>141
そんなの読んでないよ。
143>142 :2000/10/23(月) 12:42
yanagisawa先生がノーベル賞をとったら、
出版するつもりですか?
144名無しさん@お腹いっぱい。 :2000/10/23(月) 12:45
インタビューさせていただいて、共著で出版します。
タイトルは「精神と物質2」
145142:2000/10/23(月) 12:52
違うよ、先生がせっかく気楽に書いて下さってる事を
マスコミが利用しないようにだよ!!この雰囲気を大事にしたいんだよ!
146>144:2000/10/23(月) 12:54
そのためには立花隆くらいこの分野のことを
勉強せんとあかんで!
147>145:2000/10/23(月) 12:56
そうかもしれんが、ここ所詮2ちゃんだぜ?
148127:2000/10/23(月) 13:10
大変無礼でリスキーな質問かと思いました。
しかし先生の様な高名な方がどのような答をなさるか固唾を飲んでいました。
おそらく先生と面と向かって聞けないだろうし、この手の話題は日常ではタブーであります。
一応、掲示板でよく話題に上るものをベースに私が5つの問にして勇気を出して質問させて頂きました。
質問3,4,5は各論的で国内の事情に疎いだろう先生には答えにくいと思われます。
この場をお借りしてお詫び申し上げます。
そこで総論的な質問1,2についてお答え頂けると幸いです。
できれば本音を聞きたいところですが、先生もまだ将来のある方です。
答えない方が賢明な質問かもしれません。答えなくても誰も先生を責めたりしないでしょう。
1、この掲示板での様々な学歴に関するスレッドを眺めてどういった感想を持ちましたか?
2、学歴コンプレックスはありますか?あったならば自分でどう考えて解決しましたか?
149名無しさん@お腹いっぱい。:2000/10/23(月) 13:13
日本の頭脳流出は深刻。
日本って、組織のソフト作りがまるでだめだ。
外国の著名人が日本の研究・教育職に就くくらいの
自由度がないと。
きっと、お偉いさんの猛反対にあうんだろうな。
150>148、127:2000/10/23(月) 13:16
>この掲示板での様々な学歴に関するスレッドを眺めて

そんなもん、お前くらいしか眺めてねーよ。
うざいやつだな。無視されてるってことに気付よ。アホ。
151名無しさん@お腹いっぱい。:2000/10/23(月) 13:20
>固唾を飲んでいました

こいつ、相当のオタクだな。
異常さを感じるよ。
152名無しさん@お腹いっぱい。:2000/10/23(月) 13:21
150に同意、148はレッドカード
153名無しさん@お腹いっぱい。:2000/10/23(月) 13:26
レッドカード=一発退場だね。
154名無しさん@お腹いっぱい。:2000/10/23(月) 13:31
>148
コンプレックスというより、むしろ優越感でないか?それに、これだけ有名な人に、ドキュン大学逝ってよしと言わせる気かぼけ!言えるわけないだろ!

>先生、148の質問に関して、どうぞ無視してください。
155>148:2000/10/23(月) 13:36
>この手の話題は日常ではタブーであります。
なら先生に聞くなよ。非常識な奴だな、まったく。2ちゃんも日常の世界だろうが。

>国内の事情に疎いだろう先生には答えにくいと思われます。
とことん失礼な奴だな。なんで勝手に疎いとか言うんだよ。

>先生もまだ将来のある方です。 答えない方が賢明な質問かもしれません。
>答えなくても誰も先生を責めたりしないでしょう。
おまえが心配することじゃないだろう。おまえの学歴ワールドに先生を
引きずり込むんじゃないよ。
156名無しさん@お腹いっぱい。:2000/10/23(月) 13:39
学歴って騒ぐ人間にかぎって業績ないんだよね。
157名無しさん@お腹いっぱい。:2000/10/23(月) 13:42
有名国立医学部出身の先生にとって、「ドキュン大学イッテよし」と心で思っていても、しゃれにならんので言えんわなあ。
158今、ダラスは夜11時45分:2000/10/23(月) 13:44
先生お休みなさい。
俺達は勉強だぞ、ほれ!
159アメリカン:2000/10/23(月) 13:44
>柳沢先生
日本人研究者がアメリカ人研究者と対等に渡り合って行くには,どのような努力をすればよいですか?
160名無しさん@お腹いっぱい。:2000/10/23(月) 13:45
161名無しさん@お腹いっぱい。:2000/10/23(月) 13:46
先生、ズバリ、嫌いな研究者(身近な人)誰ですか?
162名無しさん@お腹いっぱい。:2000/10/23(月) 13:50
俺も寝ようっと。アメリカは深夜だよ。
163堕ちた研究者@JHU:2000/10/23(月) 13:54
柳沢、ハゲは死ね
164堕ちた研究者@JHU:2000/10/23(月) 13:54
http://cocoa.2ch.net/test/read.cgi?bbs=hosp&key=972276419
ここがおれのスレだ。
文句があったらいつでもこいや
HAGE
165堕ちた研究者@JHUポスドク:2000/10/23(月) 13:55
おれは>>160のリンク先にいつもいるからこいや。
わかったな
くそHAGE
166堕ちた研究者@JHUポスドク:2000/10/23(月) 13:59
中学の時の柔道部の監督(柳沢)。いつもジャージ姿でギンギンに朝立ちしたまま
朝練に顔出してた。
目がさめてくると部員に稽古つけるんだけど、たとえば背負い投げを
かけようとすると腰のあたりに勃起したティムポが当たる。
寝技の稽古のときなんて・・  襲われそうになった女の気持ちがよくわかる・・

怖い先生だったので誰も文句言えずにいたがオレが三年の時女子部員が入ってきて
最初の朝練で号泣したのでそれ以降は練習を見てるだけになった。
(でも朝立ちはしている・・)

167名無しさん@お腹いっぱい。:2000/10/23(月) 14:01
馬鹿な書き込みは辞めましょうよ。
168堕ちた研究者@JHUポスドク:2000/10/23(月) 14:07
こら柳沢
おれのティムポ強化しろ
HAGE
169堕ちた研究者@JHUポスドク:2000/10/23(月) 14:11
きゅっきゅ
柳沢のカメアタマを磨いてます
170名無しさん@お腹いっぱい。:2000/10/23(月) 14:12
>墜ちた研究者
おまえのせいで、柳沢先生、来れなくなったじゃないか。責任取れ!
171名無しさん@お腹いっぱい。:2000/10/23(月) 14:13
>161
居ても、そんなことここに書けるわけないだろ。
>163ー166
ほんとのキチガイがでてくるあたりが
やっぱり2ちゃんねるだな。
柳沢先生は、どうか毒されないでまたレスください。
172佐藤純:2000/10/23(月) 14:18
こらペコペコするな。
>171
173ある外科医:2000/10/23(月) 14:18
ネイチャーか。すごい。
ねいちゃんにはいっぱいのったことあるけど・・・・。
業績集にねいちゃんってかいねもばれるよな。
だめかな?
174名無しさん@お腹いっぱい。:2000/10/23(月) 14:20
先生、またお待ちしています。
175ポスドク@JHU:2000/10/23(月) 14:22
>173
こら柳沢
おめーは全部売春婦の上だろが。
はげ
176ポスドク@JHU:2000/10/23(月) 14:24
さ〜テレクラでもするか。
研究なんてあほくさくてやってられっか
177ポスドク@JHU:2000/10/23(月) 14:25
>柳沢
おまえは日大でもでたのか?
178名無しさん@お腹いっぱい。:2000/10/23(月) 14:27
Yasui@` Masatoって、優秀な人なんですね。
Natureに論文載ってるよ。
http://www.ncbi.nlm.nih.gov:80/entrez/query.fcgi?cmd=Retrieve&db=PubMed&list_uids=10647010&dopt=Abstract
179通りすがりの部外者:2000/10/23(月) 14:28
>本物のYMさん
日本では、権威に対して過剰なまでに低姿勢を貫き、一方、自分より格下であると
見なした相手に対しては傲岸不遜な態度をとる、という傾向が有ります。
アメリカの研究者にも、このような傾向は見られますか?
ここの書き込みの中にも、この傾向が見られますが、相手をしていて楽しいですか?
180ポスドク@JHU:2000/10/23(月) 14:37
>179
ばーか
柳沢はオイラより格下だよ。
しっかりしゃぶらんかい柳沢
181名無しさん@お腹いっぱい。:2000/10/23(月) 14:39
柳沢先生の誤解を解くために説明させていただきます。
おかしな書き込みを続けている堕ちた研究者は海外生活板と生物板(染色体6番、
イスカリオテのユダというHN)を中心に活動しているやつです。こいつのおかげで
海外生活板は一度壊滅しました。
NIH でポスドクをしている薬学博士(東大)で、よく薬理作用について語っています。
次のポスドク先がUCSF に決まっているにもかかわらず、先生が登場すると「キャラも
かなり良さそうだ。 こういうボスってオモロそう・・  つ、次はテキサスに行きてーー」
とか言ってたやつなので、おそらく先生に憧れるあまり注意を引くためにおかしな書き込みを
続けているようです。御気分を害されたでしょうが、無視してください。失礼いたしました。
182ポスドク@JHU:2000/10/23(月) 14:48
柳沢は産婦人科医だったっけ?
183ポスドク@JHU:2000/10/23(月) 14:49
>181
こらお前は東大じゃなくて、筑波だろが。
死ね
184ポスドク@JHU:2000/10/23(月) 14:52
>181
NIHって日大のことか?
185>181:2000/10/23(月) 14:57
説明サンキューです。理系研究者の悲惨な側面が伝わってきました。
明るくてパワフルなのがひょっとして王道かも、めげずに信じる
道を行きたいと思います。
186ポスドク@JHU:2000/10/23(月) 15:00
安井(筑波卒ジョンズホプキンス大ポスドク)は脅迫罪、
「家まで殴り込みに行くぞ」
「おまえの大学のプロバイダーに嘘のメール出して、個人情報を入手するぞ」」
「アングラHPにおまえのIPをのせて、PCを破壊するぞ」


「大学のサーバーに侵入してLOGIN IDとPASSWORDを盗むぞ」
↑この脅迫に関しては大学警察が馬路で動いた。立派な犯罪だから。

だから安井はなんか日本にいったん帰ったあと、ちゃんとVISAの更新ができて再入国
できるかどうか心配しているみたいよ。
187名無しさん@お腹いっぱい。:2000/10/23(月) 15:01
ポスドク(ださぼーや)は、海外生活板で、他人のIPを抜いて、貼りつけまわった結
果、恨みを買い、本人のそれ
までのカキコによる情報から個人を特定され、それでもその高学歴によるプライドの
高さから、「家まで殴り込みに行くぞ」、「おまえの大学のプロバイダーに嘘のメー
ル出して、個人情報を入手するぞ」「アングラHPにおまえのIPをのせて、PCを破壊す
るぞ」などと狂言を吐きつづけた
また、大学のサーバーに侵入してLOGIN IDとPASSWORDを盗むとも脅した。
その後はコテハン(ζコロ助)を中傷するスレやシモネタスレを乱立させ、板は大混
乱で大削除会がおこなわれた。

大学のシスアドに嘘のメー
ル出して、個人情報を入手するというのは、ポス毒は実行した。しかし、そのし返し
に、大学のサーバーに侵入してLOGIN IDとPASSWORDを盗むという情報は、大学のシ
スアドにおくられ、そして、その情報は、大学警察にまわされ、ポス毒はマークされ
ることになった。

また、彼はハッカーでもなんでもなく、ハッカーというのははったりとも判明。
とくいの強がり、はったり、脅迫だったようだ。
ファイアーウォールがくぐれないと漏らした。


188これもポスドク@JHU:2000/10/23(月) 15:02
500 名前: うんこ野郎 投稿日: 2000/07/21(金) 19:07

よおっ! 欧州からレスだ。
昨日は激しかったんだよな。
友人二人でロンドン歩いてたら、急にそいつが「我慢できない!やろうぜ!」とか言
い出してよお
クソ熱い中、駅の臭い便所で、やりまくったなあ。
まず、俺がそいつの黄色いブリーフ下ろして、しゃぶってやったらよ、
そいつ、ウンコ漏らしやがんの!!
ウヒャヒャヒャヒャヒャヒャヒャヒャヒャヒャ!!
後は、いつもの食糞プレイに突入よ。
夢中でウンコ喰ってるそいつのケツにぶち込んでやったら、
そいつホント、野獣みたいな声で吠えたよなあ。
あれ絶対外に聞こえてたぜ。
最後はそいつのクソまみれの顔に顔射してやったけど、そいつもあん時射精してたの
見たぜ?
俺達ってマジでシンクロしてるよなあ。
でもさ、小便でそいつの顔掃除してるとき、そいつの恍惚とした顔見て思ったんだ。
俺はコイツとは同類にはなれない。もう、ついてけねえって。

ごめんな。 バイバイ


189これもポスドク@JHU:2000/10/23(月) 15:03
海外生活板で、他人のIPを抜いて、貼りつけまわった結果、恨みを買い、本人のそれ
までのカキコによる情報から個人を特定され、それでもその高学歴によるプライドの
高さから、「家まで殴り込みに行くぞ」、「おまえの大学のプロバイダーに嘘のメー
ル出して、個人情報を入手するぞ」「アングラHPにおまえのIPをのせて、PCを破壊す
るぞ」などと狂言を吐きつづけたものの、警察の名前をだされてからは、おとなしく
なり、「おれにはしょうらいがある」などといって、偽者がやったことにしようとし
ています。その後は
MBA以外の文系院の人に質問 (212)  のスレッドで夢遊病者のように独り言をいって
うなっています。
午前9時まで海外生活にいて、その後は正午にふたたびあらわれるなど、重度の依存
症です。21時間そこにいます。また、コテハンのコロ助をシモネタや罵倒で攻撃する
スレッドを50ほど一日でつくり、板は壊滅しています。
そんなポスドクをみて、愉快な反面、こいつガイキチ?とか思い出しました。
人付き合いがよいことの利点はなに?とかいうスレッドもつくりました。
就職がなく、なやんでいるようです。
また日本人の間では異質なやつとみられていて、友人もいず、アメリカで孤独に暮ら
しているそうです。
ハンドル名 ポスドク=ださぼーや=うわじ=NYU=博士号取得後研究員=統計学=
巨人兄さん=フェラチ王=Nature&Science=ヤクザキック=ちんかす=JWA=井沢=
奈々氏=堕ちた研究者 他多数(NET荒らし)
cache-2.sbo.ma.webcache.rcn.net
筑波大学医学部生物化学専攻博士号
Johns Hopkins University Medical School@` Biochemistry@` Postdoc.で働いていま
す。
190これもポスドク@JHU:2000/10/23(月) 15:04
ポスドクの特徴は白人女性に憧れていて、アブノーマルなスレッドをいっぱいたてた。
191これもポスドク@JHU:2000/10/23(月) 15:06
192佐藤純:2000/10/23(月) 15:15
そういえば、柳沢って前から2CHにいたよね。
海外生活板でTEXASがどうこういっていた奴いた。
193佐藤純:2000/10/23(月) 16:01
柳沢〜
194佐藤純:2000/10/23(月) 16:02
Yanagisawaって英語人は発音できるのか?
Yackyと呼ばれているらしいがな。
195これもポスドク@JHU:2000/10/23(月) 16:05
柳沢正史
こいつだろ
196これもポスドク@JHU:2000/10/23(月) 16:05
Masashi Yacky
197ポスドク@JHU:2000/10/23(月) 16:06
きいたとこによると
Macという名前をつかっているそうだ。
198ポスドク@JHU:2000/10/23(月) 16:08
柳沢の顔写真だれかサラシハリにしろ。
コラ
199ポスドク@JHU:2000/10/23(月) 16:09
はやく起きないかな奴
200ポスドク@JHU:2000/10/23(月) 16:09
テキサスは深夜12次か。
明日の朝の奴の反応がたのしみだ。
いまごろ婦人とバイアグラでSEXでもやってんおか?
ハゲ
201偽MY:2000/10/23(月) 16:27
まあな
202名無しさん@お腹いっぱい。:2000/10/23(月) 16:29
さすが2chだな。このスレもそろそろ店じまいか。
案外あっさり尻切れトンボ……
203柳沢の妻:2000/10/23(月) 16:34
今日の正史ははげしかったわ。
204名無しさん@お腹いっぱい。:2000/10/23(月) 17:22
世界の柳沢があいてにするわけないでしょ。
205そうさ:2000/10/23(月) 17:29
自分の基準に当てはめて相手がびびるとでも思ってるんだろうな。
回転が早くシャープな人だろうから心配してないよ。
206みんなの酒豪ちゃん:2000/10/23(月) 17:41
>204,205
そんなこといわれると、ますます柳沢のハゲのチンカス野郎をいじめたくなるな〜
207みんなの酒豪ちゃん:2000/10/23(月) 17:41
柳沢のうんこは何歳?
ハゲ?
208みんなの酒豪ちゃん:2000/10/23(月) 17:42
>>160
おれが作者のスレッドがある。遠足にこいや
209ポスドク@JHU:2000/10/23(月) 17:46
aal@`正史の勃起ティムポが〜〜〜
210ポスドク@JHU:2000/10/23(月) 17:48
中学の時の柔道部の監督(柳沢正史)。いつもジャージ姿でギンギンに朝立ちしたまま
朝練に顔出してた。
目がさめてくると部員に稽古つけるんだけど、たとえば背負い投げを
かけようとすると腰のあたりに勃起したティムポが当たる。
寝技の稽古のときなんて・・  襲われそうになった女の気持ちがよくわかる・・

怖い先生だったので誰も文句言えずにいたがオレが三年の時女子部員が入ってきて
最初の朝練で号泣したのでそれ以降は練習を見てるだけになった。
(でも朝立ちはしている・・)

211Old Friend:2000/10/23(月) 17:49
小学校の時、水泳のスクールで柳沢正史という少年がいつも、準備体操で勃起していた。。
インストラクターの向井に興奮していたのかなあ
でもいつもいつも準備体操で勃起して、ピチピチの競技用海パンにテントを貼らせていたのは不気味だった。
ぜってー病気


212旧友:2000/10/23(月) 17:50
柳沢は中学の水泳の時間、デカイチンポを勃起させて、女子に悟られない様に
はずかしそうにしていたのがバレバレだった。
中2ともなると女子もいい体してるからな〜
厨房ですな。乳首とか透けていたもん
パッドをいれるのをしらなかったんだろね。
一部の女子
213旧友:2000/10/23(月) 17:52

          ___/ 冫   ∧_∧
 ∧_∧  _ __ (_    /   (´∀` )
 ( ´∀`)/ ) )  |   |    //   \
 (  ⊃/ //  ∧_∧    (__(_/`  |
  \__//    ( ´∀`)       _/// <プ
       ̄     ∪ ̄∪      (__(__)

   「レ」       「イ」       「プ」

柳沢の元妻をよくマワシたものだ。

214名無しさん@お腹いっぱい。:2000/10/23(月) 17:57
あらら、一日見なかったらこんな事に...。
まあ、2ちゃんだからなあ...。
215柳沢先生の友人:2000/10/23(月) 17:59
>>135
本当にそうですね。ものすごく責任を感じています。
>>166
柳沢先生は中高は男子校出身です。
このような事実はありません。
>>192
柳沢先生にこちらを教えたのは土曜日です。
それまでは2ちゃんの存在はご存知ありませんでした。

全然フォローになってないと思いますが、
予想されてた展開とはいえ、先生は大変このスレッドを楽しまれていたので
このような形になったのは残念です。
ただ2ちゃんならではなので仕方ないのでしょうね。
先生のことは心配はしてませんが、本当に素直で天然な方なので
煽りに真摯にレスしてしまうでのはないかと危惧してます。

216名無しさん@お腹いっぱい。:2000/10/23(月) 18:06
>215の最後の一文
それをわれわれは期待しているのであった
わくわく
童貞の柳沢をぎゃふんといわせ隊
217名無しさん@お腹いっぱい。:2000/10/23(月) 18:06


         ∧_∧   / ̄ ̄ ̄ ̄ ̄ ̄ ̄ ̄ ̄ ̄
       ( @`@`)< もう来ないように言うしかないな・・・
( ○   ) \_________
       | |   |_
       (__(__)


218名無しさん@お腹いっぱい。:2000/10/23(月) 18:11
>217
おまえボコボコにされるぞ
たぶん
219名無しさん@お腹いっぱい。:2000/10/23(月) 19:36
惨憺たる様相を呈してるな。
極めつけに優秀な人間と語ることのできる幸運を
素直に受け取れないのか?
自らの成長への刺激となるであろうのに。愚かしいことだ。
220名無しさん@お腹いっぱい。:2000/10/23(月) 19:50
厨房ども最低だ。>>219に激しく同意。情けない奴らだ。
22140:2000/10/23(月) 20:09
>219、220
激しく同意に同感。
でも2chていうのは、落ちこぼれ厨房が存在することを前提にしてないとだめなんでしょうね。
彼らの反応は、ひとつの現象であって、観察対象として見るくらいでないとね。
まあでもDr.M.Y.は呆れてもう来ないでしょうよ。
こういうスレがひとつくらいあってもいいと思っていたのだけど。
222名無しさん@お腹いっぱい。:2000/10/23(月) 20:11
アメリカのヤフー掲示板が一度ドキュソだらけになって閉鎖
に追い込まれたような話を聞いたのだけど、あっちには
2chに相当するような板はないのかね?
223閑話休題:2000/10/23(月) 20:26
破壊的なカキコをやめさせるには徹底的に無視すること。真っ向からはもとより、
少しでもそれを読んで反応したと思われてはいけません。
まるでサーバーがコケていたかのごとく、そいつのPCには表示されても
我々のPCには表示されてないごとくに無視しましょう。
他で何をしていたとかを暴露することは、むしろ相手を満足させ増長させるだけです。
無視こそもっとも有効で、かつ他に手立てはありません。
本題に戻って、馬鹿なカキコはあるがままに放置して、
MYからのレスが得られるように待つしかありません。
224閑話休題:2000/10/23(月) 20:27
破壊的なカキコをやめさせるには徹底的に無視すること。真っ向からはもとより、
少しでもそれを読んで反応したと思われてはいけません。
まるでサーバーがコケていたかのごとく、そいつのPCには表示されても
我々のPCには表示されてないごとくに無視しましょう。
他で何をしていたとかを暴露することは、むしろ相手を満足させ増長させるだけです。
無視こそもっとも有効で、かつ他に手立てはありません。
本題に戻って、馬鹿なカキコはあるがままに放置して、
MYからのレスが得られるように待つしかありません。
225名無しさん@お腹いっぱい。:2000/10/23(月) 20:27
>こういうスレがひとつくらいあってもいいと思っていたのだけど。

どこかセキュリティの比較的きつい、医師、医学の板立ててるところは
無いのでしょうか?残念ながらアングラ系以外では私、知らないもので。
226名無しさん@お腹いっぱい。:2000/10/23(月) 20:31
私は待ちます。
こんな機会は二度とないでしょうから。
痛快な知的刺激です。
227名無しさん@お腹いっぱい。:2000/10/23(月) 20:37
>226
待ちたいが、2CHでは些か妨害が多すぎて。
何とか手はないかな。
228120=127 :2000/10/23(月) 21:07
荒れてきたようですね。
まあ避けては通れませんでしたね。
荒らしに対してどう対応するか分かってない「単純過剰反応型」の2チャンネラーも数多く見受けられました。

正直なところ、このスレッドが留学相談板化・媚び売り化しているのに個人的に嫌気がさしましてね。
普段教授に接するようなへりくだりようで情けなかったですね。
「ご趣味は?」「上司との仲はどうすればいいでしょうか?」「英語の勉強はどのようにしましょうか?」「大事にしよう」・・・などなど
まあ、私は人の質問を非難・中傷・中止させたりするような事は言いませんが。
それと提灯持ちの態度が鼻につきましたね。
ちょっと気にくわない質問には「自分で調べろ」「下らない質問するな」「先生答えなくていいです」等々さんざん非難して、まるで検閲ですね。
どういう権限をもって護衛隊の方々は先生の代弁者になったのですか?
質問に答えるか答えないかを判断する能力を先生はお持ちでないと判断しているようで逆に失礼ですよ。
護衛隊気取りも程々にしてもらいたいですね。まあそういうイエスマンがいくらあがいてもオリジナリティのある仕事もできないし現状は変えられないでしょうが。
自分の事も自分で決められず、権威によるお墨付きをもらう事で安心しちゃうんでしょうね。
彼らが忌み嫌った学歴と根本的に一緒ですね。口では学歴は関係ないだの言っても、本心は学歴により安心しちゃったり劣等感持っちゃったりしちゃうんでしょうね。
私が2chらしいprimitiveな質問をした後の提灯持ちの罵声は学歴問題の根深さ(彼の学歴コンプレックスかもしれませんが)の一端を垣間見せてくれました。
>228
それで?
230閑話休題 :2000/10/23(月) 21:50
だから、反応してはいけません。
どのような形でこようと、心理学的にはある種の不快感を催させる言動としては
同じこと。
つられて反応しないでネ。無視です。
231名無しさん@お腹いっぱい。 :2000/10/23(月) 21:53
>228
>私は人の質問を非難・中傷・中止させたりするような事は言いませんが。
充分に言ってるだろう、あんたは。
232名無しさん@お腹いっぱい。 :2000/10/23(月) 22:17
スポーツ番組の中で野球場なのでヒーローインタビューをしていると画面の
後ろに、雲霞のように小学生が集まってきてやたら「うんこ」などと叫んで
いる連中が出てくるように、とんでもないカキコをする連中が登場したのは
うんざりしてしまったけれども、アメリカで若くして第一線で活躍している
最先端の研究者に、純粋に質問したいという思いで、カキコした人も少なく
なかったと思うよ!

日本では、大先生然として、下のものは全く相手にしない権威者が多い中で、
些細な質問にも気さくに答えていただいた柳沢先生の素顔かよく出ていて、
私は逆に好感を抱いたけれどもなあ。ただもうこれに凝りて出てきていただ
けないと思うと残念。
真夏ならぬ、秋の夜長に流星を一瞬、垣間見たような貴重な出来事でした。
233>228 :2000/10/23(月) 22:19
>荒らしに対してどう対応するか

結局おまえが一番の荒らしなんだよ。
消えろ。うざすぎる。
あっ、反応してしまった。>230すまん。
MYを待ちます。
234名無しさん@お腹いっぱい。 :2000/10/23(月) 22:25
すこしでも生化学に携わったこともある人なら、
Yanagisawaのすごさは知っている。
そんな人が2ちゃんにカキコすることがまたすごい、
というか新鮮だった。
235名無しさん@お腹いっぱい。 :2000/10/23(月) 22:28
流星を一瞬 か。
そんな感じだな。貴重な時間だったよ。
脳血管攣縮との関係の話は聞いといて良かった。10年前から注目してたから。
236名無しさん@お腹いっぱい。 :2000/10/23(月) 22:38
>235
そんなこと、ここで知るなって。
おれ脳外科じゃないけどすこしは知ってたぞ。
237>228 :2000/10/23(月) 22:42
媚びているのではないですよ、匿名でなく、正々堂々メアドまで
出されている潔さの前には誰しもこういう礼儀正しい態度を
取るのではないですか?何でも聞いて下さいと言われてまぶしい
訳です。232さん、いい事言います。
238名無しさん@お腹いっぱい。 :2000/10/23(月) 22:44
セカンドメッセンジャーの話でも聞けば良かったかな?
生化の専門じゃないもんで。
惜しい事をした。
239やはり、 :2000/10/23(月) 22:46
Yanagisawa先生は日本医学界の新星であることは間違いない。(現在はUSAですけどね)
今後も、もっと日本からfollowerがでてほしい。

240名無しさん@お腹いっぱい。 :2000/10/23(月) 22:52
才能と努力と巡り合わせ すべてがそろってる。
素直に応援したいとおもうし、もっともっと突っ走ってほしい。
メジャーの佐々木、イチローを応援する心理に通じるものがあるな。
241本物のMY :2000/10/23(月) 23:25
おやおや、ちょっと見ぬうちに随分「煽られて」(というのですね、日本のネットでは)おりますね。なんだか、俺はもう二度と戻ってこないんじゃなかって皆さん心配していらっしゃるようです。ご心配なく、この手の輩どもの所作は全く気になりませんので。Irrelevantでintelligenceのかけらもない煽りがたとえ100項目続こうとも、その中から真面目な質問を見いだすのはyeast two-hybrid screenよりよほど簡単でございます。(爆)

ただ、さすがに俺も(いちおう多忙な!)業務の合間に書いているので、週日はそうは時間が取れないだけです。昨日までは土日だったので、沢山書けたのです。真面目な質問には、時間のあるときに必ずレスしますので、どうぞ続けてご質問下さい。

あ、そうそう、俺の尊敬する研究者(PIにしろポスドクにしろ)の条件:「良い質問が出来ること!」

ではでは、続きはまた時間のあるときに..
242>MY先生 :2000/10/23(月) 23:53
基本的な質問かもしれませんが、皮膚軟部組織の血管(特に細動脈レベル)
に対するエンドセリン受容体遮断薬の血管拡張効果と皮膚血流増大効果は
どれくらい期待できるのでしょうか?
臨床的に興味あります。(plastic surgeonです)
243名無しさん@お腹いっぱい。 :2000/10/23(月) 23:56
うーん、超一流は何かが違うね。
その辺でオナーニリサーチしている連中とは。
244名無しさん@お腹いっぱい。 :2000/10/23(月) 23:59
>241
本当に好きなんです!
245名無しさん@お腹いっぱい。 :2000/10/24(火) 00:08
つうか本音では120みたいなアホらしいが素直な質問は俺もしたかった。
246つまんなくてすまん :2000/10/24(火) 00:18
アメリカは、例えると読売巨人軍みたいだ。
たたき上げもすごいが、各国のトップを
もらって行きさらにパワーアップする。
そして、みなアメリカにいきたがる。
日本は根底から組織変革しないと
とてもたちうちできない。
247名無しさん@お腹いっぱい。 :2000/10/24(火) 00:22
>245
そうそう、柳沢先生でも異性にふられたことがありますか、とか
柳沢先生でもエロビデオ借りたことがありますか、とか米国の風俗事情はどうですか、
とかね。
いかんいかん、書いてて情けなくなってきた。
少なくとも、学歴みたいな過去の栄光にこだわるような方ではなさそうです。
では煽りはここまで、まっとうな質問をどうぞ。
248名無しさん@お腹いっぱい。 :2000/10/24(火) 00:24
柳沢先生には申しわけありませんが、現在日本人の医学者でノーベル賞候補という人は
20人はいると聞いた事があります。また各大学の研究室ごとに一人はいるという噂も
ありました。という事は世界には数百人いや数千人の候補者がいるのではないでしょう
か?
249名無しさん@お腹いっぱい。 :2000/10/24(火) 00:33
>248
>現在日本人の医学者でノーベル賞候補という人は 20人はいる

どれくらい近いかが問題なんだよ。
具体的に名前あげてみてください。
じっくり検討してみましょう。
250>248 :2000/10/24(火) 00:39
レスするまえに、スレッド全部読め。
柳沢先生自身はノーベル賞受賞の可能性は
否定的コメントをしてるぞ。
他人の出世がそんなに悔しいのか?
まあ、どしょぼ研究者にこういうタイプは多いけどね。
251名無しさん@お腹いっぱい。 :2000/10/24(火) 00:42
>現在日本人の医学者でノーベル賞候補という人は 20人はいる

うちの医学部の学部長(地方国立大学)も酒に酔ったら必ず、若い秘書のおね〜に
鼻の下を思いっきり伸ばし、「ワシ、医学生理学賞の候補になっとんや」とのたまっている(爆)。
それに対して秘書達も「せんせ、すご〜い、ノーベル賞の祝賀会の時は、必ず私たちもよんでくださ〜いネ」と
からかいのレスをすることを忘れていない。
日本人のノーベル賞候補者なんて自薦が多い。
誰でも言えちゃうよ!
252先生明るいですね :2000/10/24(火) 00:50
誰か、まじめないい質問、出して下さいよ。
100年後にはお互い、全員いないんだからね、この世には・・。
253>148=228 :2000/10/24(火) 00:56
>>228
>>148
あんたほんとに常識がないな。ネットとはいえ、初対面の人間との会話なら
普通趣味とか休日なにしてるとか、そんなことから話しを始めるのが普通だろうが。
あんた、いきなり「学歴についてどう御考えですか」とか、初対面の人間に
聞けるのか。あんたも医者だろうけど、問診の時でも無難な挨拶から入るのが
普通だと思ってたけどね。学歴問題の根深さを感じたのなら、もうあんたの目的は達成出来ただろう。学歴板で存分に発言してくれ。
254名無しさん@お腹いっぱい。 :2000/10/24(火) 01:06
253に同意。
まあ、学歴はそれなりに大事だとは思うけど、それは我々凡人の支えに
なっているだけで、インパクト・ファクターで勝負できる人はもう、超越
してるって。
柳沢先生には、数少ない日本人のヒーローとして栄光を目指してほしいですね。
255253に追加 :2000/10/24(火) 01:10
もう飛び級の学生だっていますからね。
256名無しさん@お腹いっぱい。 :2000/10/24(火) 01:24
この辺の、学歴うんぬんにやたらこだわる所が、日本人特有の
狭さだね
257名無しさん@お腹いっぱい。 :2000/10/24(火) 01:32
先生、裏ビデオ借りたことありますか?
258名無しさん@お腹いっぱい。 :2000/10/24(火) 01:33
童貞捨てたのはいつですか?
259名無しさん@お腹いっぱい。 :2000/10/24(火) 02:09
>257@`258
シンデクダサイ。
260名無しさん@お腹いっぱい。 :2000/10/24(火) 02:13
>257
アメリカじゃ裏ビデオでもね〜だろ。
>259
同意。
261名無しさん@お腹いっぱい。 :2000/10/24(火) 02:17
柳沢先生の誤解を解くために説明させていただきます。
おかしな書き込みを続けている堕ちた研究者は海外生活板と生物板(J1@` 染色体6番、
イスカリオテのユダというHN)を中心に活動しているやつです。こいつのおかげで
海外生活板は一度壊滅しました。
NIH でポスドクをしている薬学博士(東大)で、よく薬理作用について語っています。
次のポスドク先がUCSF に決まっているにもかかわらず、先生が登場すると「キャラも
かなり良さそうだ。 こういうボスってオモロそう・・  つ、次はテキサスに行きてーー」
とか言ってたやつなので、おそらく先生に憧れるあまり注意を引くためにおかしな書き込みを
続けているようです。御気分を害されたでしょうが、無視してください。失礼いたしました。
262228 :2000/10/24(火) 03:48
私の発言で気分を害された方がいらっしゃるならあらかじめお詫びします。

>253
>ネットとはいえ、初対面の人間との会話なら
>普通趣味とか休日なにしてるとか、そんなことから話しを始めるのが普通だろうが。
そんな会話は2chでは見た事がないですが?
むしろ初対面でいきなり「逝ってよし」とかそんなのばっかですよね。

>問診の時でも無難な挨拶から入るのが
>普通だと思ってたけどね。
ここでのやりとりは問診ではありませんから。

>学歴問題の根深さを感じたのなら、もうあんたの目的は達成出来ただろう
それはあくまで周囲のレスポンスでしたね。

>まあ、学歴はそれなりに大事だとは思うけど、それは我々凡人の支えに
>なっているだけで、インパクト・ファクターで勝負できる人はもう、超越
>してるって。
それはあくまで貴方がインパクト・ファクターで勝負している者としての発言ですか?
それとも貴方がインパクト・ファクターで勝負している者を眺めての発言ですか?
前者でなければあまり意味がないと思いますが。

>256
>この辺の、学歴うんぬんにやたらこだわる所が、日本人特有の狭さだね
個人的にはアメリカの方がもっと学歴にうるさいと思いますが。

>259
>不同意です。
質問:先生、チン毛が生えたのはいつですか?
264>148=228=262 :2000/10/24(火) 04:05
>この手の話題は日常ではタブーであります。
なら先生に聞くなよ。非常識な奴だな、まったく。2ちゃんも日常の世界だろうが。

>国内の事情に疎いだろう先生には答えにくいと思われます。
とことん失礼な奴だな。なんで勝手に疎いとか言うんだよ。

>先生もまだ将来のある方です。 答えない方が賢明な質問かもしれません。
>答えなくても誰も先生を責めたりしないでしょう。
おまえが心配することじゃないだろう。おまえの学歴ワールドに先生を
引きずり込むんじゃないよ。
>>148

こっちの書き込みに対する反論はしないのですか?
それからここが2ちゃんねるだから、まわりが不躾な発言ばかりだから
自分もそうしていいと考えているのなら、子供じみてますね。
まわりがどうであれ、常識のあるマナーを持って振る舞うのが大人だと
私は考えています。
265本物のMY :2000/10/24(火) 05:29
>116 研究費の日米比較
(投稿番号をhyperlinkするには、どうすれば良いのか誰か教えて!)

俺のラボへのHHMIからの研究費は、HHMIポスドク5人、技官4人、秘書、俺自身の人件費込みで、約$1M/年です。その他のソース全部入れても、$1.4〜1.5M/年でしょう。ラボ全体では今20人を超えていますから、決してすごく裕福なわけではありません。特にマウスの仕事をしていると、飼育費の高さに涙がちょちょぎれます(アメリカでは飼育費はどこでもユーザー負担です)。今の日本の比較的大きなラボに来ているお金は、用品物価の差(200〜250円/$ですか?)を考えても、完全にアメリカのレベルに追いついていると思います。

ただし日本の問題は、億円台の予算を取っているラボの層が非常に薄いことですね。その一方で、一部のラボにはPIあたり数10億円という膨大な予算が行っています。うちの大学のGoldstein/Brown研だって、年間$数Mのレベルですよ。これはあくまで私見ですが、一人のPIのもとに30億円奮発するよりは、30歳そこそこ程度の最優秀の人を30人厳選して、それぞれ1億円ずつ配ったほうが、素晴らしい仕事の出る確率と量は全体では高いと思います。

例えば、医学部を卒業してそのまま基礎の院へ行って数年間ポスドク(あるいは助手?)をやったとして30〜32歳くらい。最初に数年間臨床をやっていたとしても35歳くらい。優秀な人間なら、だいたいこの時点でscientificに独り立ち出来るようになるはずです。日本の今のシステムでは、そういった人たちが、まともな規模の(つまり年間1億円規模の)ラボを持つことがまだまだ難しいのではないでしょうか。

このような事を正しく実行するためには、お金を出す官庁の役人が、官僚制度生え抜きの「素人」集団ではダメです。Big name bossたちによる評価シンジケートに完全に依存ぜざるを得ない今の状況ではダメで、自らCell@` Nature@` Scienceなどの原著論文を毎日読むような、本当の「プロ官僚」を官庁が外部から引き抜いて、冷静に、政治的あるいはリニエージ・バイアスのない研究費分配が出来るようにならないと、若い研究者で突出する人がなかなか出てこないと思います。

あと、特に大きな大学の医局から来た日本人のMDポスドクに共通して言えるのが、日本のボスに対する異常なまでの忠誠心です(失礼を承知であえて言います)。自分がそのボスの後を継いで教授になれる確率は競争人数からいっても極めて低いのに、どうしてあんなに「出身講座」に拘るのでしょうかね。そういう、若い人のある意味でのambitionの欠如も、日本の大きな問題だと思います。

もう一点付け加えるとすれば、日本の女性研究者(PIレベル)の極端な少なさです。今の状態では、優秀な人口の半分が完全に埋もれてしまっているわけで、日本とアメリカの人口比は1:2ではなく1:4と考えてよいと思います。日本の研究者人口が上へ行くほど層が薄いのは、当然とも言えるのです。
266本物のMY :2000/10/24(火) 05:35
>140 う〜む、この人はどうやら玄人さん(マスコミ人)ですかね?

言いたいことは一杯ありますが、取り敢えず上記265あたりを読んでいただいて、またご質問下さい。
267名無しさん@お腹いっぱい。 :2000/10/24(火) 05:39
>>116
>>140
先生、出来なかったらこれでも
コピペして使ってよ。数字は
半角でないと駄目だよ。
268本物のMY :2000/10/24(火) 05:48
>242 形成外科の方の質問..

エンドセリン受容体拮抗薬の皮膚微少血管拡張作用は、正常人での実験もpublishされていると思います。
植皮術後回復や創傷治癒における拮抗薬の作用など興味は尽きないですが、まだまだこらからの領域なのではないでしょうか。
拮抗薬が一つでも臨床治験を通れば、そういった研究が爆発的に進むと思われます。
現在開発中の拮抗薬は、数年以内には認可され始めると期待されています。
269本物のMY :2000/10/24(火) 05:59
>247 クソ真面目なレスばかりでもつまらないので..

異性にふられたこと?もちろんありますよー。一番痛かったのは、まだ若かりし高校卒業直後。そのとき付き合っていたのは、知っている方も多いと思いますが櫻蔭高校の同い年の子でした。たしか中3の時からの長いつきあいでした。彼女は、俺が「東京の大学」に行くものとばかり思っていたらしく、筑波に行くことになったとたんに、あっさり切られました。(涙)しかも事もあろうに、彼女は即、東大理学部の俺の高校の一つ先輩に乗り換えたのです(彼女自身はお茶へ行った)。

おぅ、あのときばかりは、「学歴」コンプレックス、大いに感じましたねえ〜。(爆)
彼女、今頃どうしてるんだろなぁ..

エロビデオに関しては、「俺は正常かつ健康な男です」とだけ行っておきましょう。(^^ゞ
先生おもしろいよなー。そのうち(爆)が(藁 に変わるんだろうな。。。
厨房の煽りにも「逝ってよし」とか「オマエモナー」とか。
顔文字も(^^ゞから(;´Д`)へ。。。
271名無しさん@お腹いっぱい。 :2000/10/24(火) 06:25
ああ、このスレが気になって仕事が出来ない。
272コロ助プラスワン :2000/10/24(火) 06:34
素朴な質問
若い研究者ということですが、何歳なんですか?
誰でも知っている人はこたえてください。
わざわざ本人にこたえてもらうことでもないので。
273本物のMY :2000/10/24(火) 06:35
>>117  Howard Hugh Medical Instituteについて。

ひとことで言ってしまえば、「世界最大の私立の非営利基礎医学研究財団」です。年間予算は、現在$1B近いという、尋常な額ではありません。全米の主要な生命科学系キャンパスに、約350のラボが間借りする形で散在しています。研究費は、莫大な永久基金をもとに、株の運用で作り出しています。Wallstreatから敏腕な株屋をリクルートして、内部運用(超低オーバーヘッド)しているのが特徴です。

詳しくはhttp://www.hhmi.org/へどうぞ
274名無しさん@お腹いっぱい。 :2000/10/24(火) 06:37
>>272
ログぐらい読め。1960年生まれって書いてあるぞ。>>63
275名無しさん@お腹いっぱい。 :2000/10/24(火) 06:39
先生が丁寧に答えるので、こども電話相談室みたいになってきた。
276コロ助プラスワン :2000/10/24(火) 06:41
すげ〜若い
277コロ助プラスワン :2000/10/24(火) 06:42
まだ餓鬼だったか
278元祖ポスドク :2000/10/24(火) 06:49
やっぱり白人少女はかわいいね。
あの尻、あの乳
あの髪
たまらん
おいらの勃起ティムポを味わわせたいずら
279元祖ポスドク :2000/10/24(火) 06:51
しかしヤンキースは強いな
クレメンスうざいな
280元祖ポスドク :2000/10/24(火) 06:53
ボルチモアも飽きたな
週末はKorean娘目当てでDCへドライブに逝こう。
愛車のカローラで全開さ。
281元祖ポスドク :2000/10/24(火) 06:53
あ〜白人少女に抱きつきたいじょ〜〜
282元祖ポスドク :2000/10/24(火) 06:54
柳沢慎吾はどうよ
283元祖ポスドク :2000/10/24(火) 06:55
白人のマンコと日本人のマンコはどちらがより臭いのでしょうか?
まあどっちも臭いのは確かだけど。
MoreStinkyを知りたいにゃ〜
284元祖ポスドク :2000/10/24(火) 07:01
まだ餓鬼の正史よ〜TXはいまでもまだ暑いのか?こら
285筑波大病院看護婦一同 :2000/10/24(火) 07:01
正史のチンチンがしゃぶり隊
286元祖ポスドク :2000/10/24(火) 07:02
>マーシー

おまえの給料こたえてみろや

287みんなの酒豪ちゃん :2000/10/24(火) 07:03
またポスドクが荒らしているのか
おまえの趣味だな
荒らしは
288みんなの酒豪ちゃん :2000/10/24(火) 07:03
早く海外生活板へ帰って恋
289外科医です。 :2000/10/24(火) 07:10
>282
それ私も思ってた。
290トレビアーソ :2000/10/24(火) 07:14
>289
おまえシモネタばっかりいってるよな。
291トレビアーソ :2000/10/24(火) 07:15
292[email protected] :2000/10/24(火) 07:34
Fuck yo All yo
293[email protected] :2000/10/24(火) 07:44
[2:330] 外科医です。 ■▲▼
1 名前: 名無しさん@お腹いっぱい。 投稿日: 2000/09/16(土) 07:50

東京近郊で脳外科を開業しております。
既婚者、31歳、岩城晃一に似てます。
愛人希望です。
宜しくお願い致します。
注*(女性に限ります)

>>290

このハゲです


294[email protected] :2000/10/24(火) 07:45
295名無しさん@お腹いっぱい。 :2000/10/24(火) 07:54
>普通趣味とか休日なにしてるとか、そんなことから話しを始めるのが普通だろうが。
学会の質問でも趣味や休日の過ごし方を尋ねるのですか?

少しはTPOを考えたらいかがですか?
296>MY先生 :2000/10/24(火) 08:07
117です。
HHMIってすごいそしきですね。
日本には見当たりませんね。
あえて言うと、理化学研究所の医学版ってとこですかね。
297元祖ポスドク=堕ちた研究者=染色体6番 :2000/10/24(火) 08:25
柳沢先生を見習って・・
ついでにカメレオンも飼ってみようかな(笑
マネたからといってCell級の論文が書けるワケではないが・・
298染色体6番 :2000/10/24(火) 08:38
>>265
何をしてる人か知りませんが、日本のグラント制度をよくご存知ないようですね。
299名無しさん@お腹いっぱい。 :2000/10/24(火) 08:38

http://216.218.192.139/test/read.cgi?bbs=kaigai&key=967616301&ls=50
↑が、ここを荒らしてるウィスコンシン大、学部留学生のガキです。
一時、海外生活版を荒しまくって壊滅寸前に陥れたために、
今ではIP規制くらって海外板にはこれないようになっています。
ここも早めにwisc.eduでIP規制するのが一番だと思います。
300染色体6番@NIH :2000/10/24(火) 08:49
>>268
拮抗薬をいくら突っついたってねえ・・
力技では論文にもならないよ。
301染色体6番@NIH@next UCSF :2000/10/24(火) 09:01
>>299
君、海外生活板の実情に詳しいねえ・・
案外君が荒してんじゃないの(笑
302染色体6番@NIH@next UCSF :2000/10/24(火) 09:26
>>296
なに読んでんだよ。
HHMI 自体は建物がないので間借りしてるって説明されてるじゃないか。
理研は建物がちゃんとあるだろう、所員の間では殺人事件がおこったり
アザイド入れたり無茶やってるけど。
303名無しさん@お腹いっぱい。 :2000/10/24(火) 09:39
質問:何か運動はやっていましたか(ピストン運動以外)?
304名無しさん@お腹いっぱい。 :2000/10/24(火) 09:59
質問:ノーベル賞を受賞された白川先生とはお知り合いですか?
305偽MY>304 :2000/10/24(火) 10:26
お尻愛
306FORCEPS :2000/10/24(火) 10:53
>253,254

Dr. M.Y.が学歴なんか超越してるってのはその通りだろうけど、日本では学閥が
いまだに残っているのは厳然たる事実だ。研究費、人事交流、人材育成などの面で
大きな弊害になっているだろうよ。アメリカで活躍している(た)人達がこの点で
日本の学会・大学組織をいかに効率よいものに変えるかという視点で是非意見は聞きたいね。

もっとも、油がのって研究に没頭している人はそんなことには関心が薄いかもしれないけど、
もう少し年齢が上になったところでは広い視野で後進の育成を考えてくれるといいと思う。
筑波は、江崎氏を学長にするときに教授の間で大変な混乱があったようだが、その結果
研究環境がかなり向上したと聞いている。

307名無しさん@お腹いっぱい。 :2000/10/24(火) 11:19
ここの書き込みって、昔のアイドルの親衛隊か追っかけのような連中が多いね。
308名無しさん@お腹いっぱい。 :2000/10/24(火) 11:30
>307
研究をやった事のない君には関係無いスレッドだよ。
309名無しさん@お腹いっぱい。 :2000/10/24(火) 12:13
僕は今まで、隠れて2chをやっている事に罪悪感を感じていましたが、柳沢先生もやっていると知り、自信が湧いてきました。
310名無しさん@お腹いっぱい。 :2000/10/24(火) 12:36
311 名前: 名無しさん@お腹いっぱい。 投稿日: 2000/10/24(火) 12:33

312 名前: 名無しさん@お腹いっぱい。 投稿日: 2000/10/24(火) 12:51

313 名前: 名無しさん@お腹いっぱい。 投稿日: 2000/10/24(火) 13:19
311名無しさん@お腹いっぱい。 :2000/10/24(火) 12:40
>質問です
論文作成の最重要ポイントは何ですか?
ラボの仲間はbeautifulなストーリーを展開して論文にする。
それが単にspeculationである場合が多いけれども。
自分はある発見から、その証拠を積み上げながら論文に
しようとするのですが、ストーリーに魅力がないです。
どうしたら良いのだろう?
312名無しさん@お腹いっぱい。 :2000/10/24(火) 12:43
取り敢えずこれ以上ばれないようにアクセスする人以外はsageレスで
頼みます。

裏2ちゃんねるの見方は
1.書き込みの名前の欄に http://fusianasan.2ch.net/ と入れる。(裏ドメイン名)
#注意#「fusianasan」です!。”shi”にしないように注意してください。最近ミスが目立ちます。
2.メールアドレス欄に、小学生はlow中学生はmid高校生はhighそれ以上はadul、
   盗撮モノはpeepと入力してください。
3.本文にIDとパスワードの guest guest を入れて、書込みボタンを押します。
#注意#guestの間は半角です。<最近非常に多いようです。ミスしないようにしてください。
4.タイトルが「ようこそ 裏2ちゃんねるへ(笑)」に変わればばOK
5.サーバーが重いと2chに戻ってくるけど、くじけずに何度も挑戦。
  うまく行くと、目的のページにつながります。
6.家庭の電話回線よりも、企業や学校の専用回線からの方がサーバートラフィックの
  都合上つながる確立が高いです
313名無しさん@お腹いっぱい。 :2000/10/24(火) 12:44
しつこいようですが・・・

6.家庭の電話回線よりも、企業や学校の専用回線からの方がサーバートラフィックの
  都合上つながる確立が高いです
guest guest
315名無しさん@お腹いっぱい。 :2000/10/24(火) 12:52
>312
人のリモホ見るのが楽しい?
316名無しさん@お腹いっぱい。 :2000/10/24(火) 12:53
まあな&#9786;
317名無しさん@お腹いっぱい。 :2000/10/24(火) 12:55
○○edu.を待ってるんだね。
318http://www.jikei.ac.jp.2ch.net/ :2000/10/24(火) 12:58
guest guest
319http://www.kyorin.ac.jp.2ch.net/ :2000/10/24(火) 12:58
guest guest
320http://www.harvard.edu.2ch.net/ :2000/10/24(火) 12:59
guest guest
321http://www.chiba-u.ac.jp.2ch.net/ :2000/10/24(火) 13:00
guest guest
322http://www.stanford.edu.2ch.net/ :2000/10/24(火) 13:03
>312
逝ってよし!
323名無しさん@お腹いっぱい。 :2000/10/24(火) 13:04
おまえは誰や?
324名無しさん@お腹いっぱい。 :2000/10/24(火) 13:06
この際、みんな本名を名乗ろうぜ!
325名無しさん@お腹いっぱい。 :2000/10/24(火) 13:11
所属からだね。
326T.T. :2000/10/24(火) 13:20
朝日新聞科学部
327N.U :2000/10/24(火) 13:55
おやおや、こんなところがあったんだ!
柳沢さん、覚えておられますか?筑波と京都でお会いしましたね。
328MY(本物は省略) :2000/10/24(火) 13:55
>>306 学歴(改め)

なるほど、はるかに建設的なご質問だと思います。しかし残念ですが、全くおっしゃる通りで、少なくとも今の俺には、直接、日本のシステムを変えるべくエネルギーを費やす興味も用意もありません。意見は聞かれればいくらでも言いますが(例えば>>265参照)。それより、アメリカで活躍している(た)人に頼るようでは、日本のシステムはいつまで経っても変えられないのでは?ただし、将来、私のラボで一時期を過ごした日本人の方々が偉くなっていった時点で、ダラスでの経験がかれらの視点に与える影響は大きいかも知れないわけで、その点は大きな責任を感じています。

江崎玲於奈氏に関しては、俺は少ししか重なっていないので詳しいことは書けませんが、やはり彼自身がアメリカ生活が非常に長かったせいで日本の学界政治の動かし方を必ずしも良くご存じではなく、絶大なネームバリューの割にはあまり何も出来なかった、というのが学内の者の感想だったようです。旧教育大系とくに文系の教官からの、いろいろな抵抗に遭ったとも聞いています。研究環境が向上した、と言っても、医学から見えてきたのはいわゆるTARAが出来たことくらいで、これも今のスタンダードではちっぽけなものです。母校を悪く言うのはあまり気持ち良くないですが...
329MY :2000/10/24(火) 14:02
>>298 6番染色体さま

こういう反論、大歓迎です。
ただし、俺の見解でどこが間違っている(あるいは無知)と思うのか言ってくれないと、話の進めようがありませんが。
330N.U :2000/10/24(火) 14:04
前の方で書かれていますが、プロの仕事をどう評価するかというのはなかなか難しい問題ですね。
たとえば、白川さんの導電性ポリマーの仕事は、もともと、有機半導体、超伝導の流れで、でてきたもので、ある意味では八〇年代初めに終わっていた研究です。今更再評価されるとはと、専門家にはノーベル賞受賞にとまどいがあるのではないでしょうか。
ノーベル賞をあまり持ち上げすぎるのもなんだとは思いますが、その後の影響は大きいので、無視はできないのですけれど……


たしかに、毎日、ネーチャーやサイエンスを読んで、プロの意見をきっちり取材してという本物ジャーナリストがいないといけないと思います。ガンバらなくっちゃ!
331>>306 :2000/10/24(火) 14:07
名を名乗るか立場を明かして!片一方が匿名のまじ議論は
ちょっと不公平だよ!
友達、頼むよ。
332名無しさん@お腹いっぱい。 :2000/10/24(火) 14:07
でも、遠い将来でもいいので、いずれは日本の研究・教育システムに関するご意見番になっていただきたいです。
333MY :2000/10/24(火) 14:09
>>300 反論です。

いやいや、拮抗薬が臨床認可された暁には、(初期の適応が何であろうと)爆発的に臨床研究が進むと思っていますよ。論文にならないというのも不正確で、エンドセリン拮抗薬を用いた仕事は、Medlineで数100報ではきかないとおもいます。すでにヒトでの仕事も、New Eng. J. Med.やLancetといった一流誌にいっぱい出ていますよ。俺自身は分子遺伝屋なので、自分自身の仕事には拮抗薬は道具程度にしか使いませんけど、拮抗薬というものは(とくに一旦臨床認可されたら)決して馬鹿に出来ないのです。
おいマック柳沢
おまえまだ、39か40だろ?
おめ〜餓鬼じゃねーかよ。
けけけけ
335MY :2000/10/24(火) 14:12
>>303

学生時代は、東医体のスキー部でした。それほど速くありませんでしたが。
おいらは約30歳
柳沢さいこ〜
若いって素敵
そんなに若いとは知らなかったぜ
てっきりハゲじじいかと思っていたずら。
まだに〜ちゃんやんけ
340MY :2000/10/24(火) 14:15
>>304 白川先生について

大変残念ながら、学部(筑波では学系という)も違うし、全く存じ上げませんでした。是非いつかお知り合いになりたいものです。
江崎前学長とは、個人的にも面識があります。
341MY :2000/10/24(火) 14:23
>>311 論文について

自分的には、論文の中でストーリーを展開することはある程度大事だと思います。第一、ストーリーの無い論文は、いい雑誌にはなかなか通らないし。基本的に、データーとその直接解釈においては、過ちを犯すことは絶対に避けなければなりません。しかし、discussionでのspeculativeなストーリー作りでは間違ってもよいのです。大胆にやったらいいんじゃないでしょうか。俺の1988年のエンドセリン発見の論文でも、データでは間違いを冒していませんが、Discussionでは大ポカをやっています(興味があったら読んで探してみて下さい)。

Resultsは慎重に、Discussionは(refereeの批判をくらわない範囲で)大胆に。
yanagi
ティムポってなにかしってるか?
343名無しさん@お腹いっぱい。 :2000/10/24(火) 14:26
感動しています。これほどのカリスマ的存在の大物が、アングラと言われる2chへ乗り込んできて、われわれの腐った性根を叩き直してくれている。人間的にも堂々とされて、こんな素晴らしい研究者は二度と現れないのではないか。
344名無しさん@お腹いっぱい。 :2000/10/24(火) 14:27
>341
有り難うございます。勉強になりました(311ではないですが)。
>343
わしは?わしは?
おまえらFANクラブでもつくれや
一部の予備校講師並の人気だな
348 N.U :2000/10/24(火) 14:33
 こんなに爆発的に増えたスレッドも珍しいですね。
ゆっくり読んでからレスしたいと思います。
349なんか1人無視されてる悲惨な奴がいるな・・ :2000/10/24(火) 14:36
友達になってやろうか♪
350311 :2000/10/24(火) 14:38
>>341
ありがとうがざいました。
現在、論文作成中なのでとても勉強になりました。
慎重にかつ大胆に書きます!
351N.U :2000/10/24(火) 14:44
あははははは
そうですね、ストーリーのない研究は、ジャーナリストとしても大変に紹介しにくいものです。
自分の仕事を位置付けと共に、素人に説明できない研究者は少なくないですね。
その点、柳沢さんは、大学院時代から歯切れが良かったと思います。
352>343 :2000/10/24(火) 14:50
」イナル、ネクス、?、ハ、、、ハ、ノ、ネクタ、?、コク螟ヒツウツウ、ア、?、隍ヲ、ヒシォハャ、ソ、チ、篆ミヘ隍?ネマーマ、ヌ・ル・ケ・ネ、ヤ、ッ、キ、゙、キ、遉ヲ。」ソヘタク、マテサ、、。」
柳沢には素人への説明は無理。
あいつ馬鹿野郎だから
354MY :2000/10/24(火) 14:57
>>343

自己弁護のために言っておきますが、いわゆるカリスマ性の高いタイプのPIではないと自分では思っています(良いか悪いかは別として)。うちにいたことのある日本人に聞けば皆そういうでしょう。日本の医学部(特に一部の臨床系)には、ものすごいカリスマ教授がたっくさん居るじゃないですか。

それから、2chへは「乗り込んで」来たわけではなく、これほどのアングラ劇場とはつゆ知らず、ついついカキコ始めてしまっただけです。(爆)
355名無しさん@お腹いっぱい。 :2000/10/24(火) 15:06
先生は、夜型タイプですか?それとも朝型のタイプですか?
356N.U :2000/10/24(火) 15:07
裏だから我々には、興味があるということもありますね。ただしこれにのめり込むと、表に適応できなくなるという欠点があります。

日本の臨床医学のカリスマ性はもんだいですなー。
357染色体6番薬学博士二世研究者@NIH@next UCSF@年末プロポーズ :2000/10/24(火) 15:07
>>333
エンドセリンアンタゴニストって特異的か?
ライガンド自身が小さなペプチドなので、レセプターの認識部位
も数個のペプチドだろう。他の機能の異なるレセプターをブロック
してる可能性もある・・
そもそもエンドセリンの機能ドメインって決定されてるのか?

>>341
俺は論文のdiscussion は読まないよ。たしかにその論文の弱点とか
次の目標を文章にして浮き上がらせるという点ではいいけど、うだうだ
書く暇があったら新しい実験をやってそのデータを加えてもっと上の
ジャーナルに投稿する・・
実際大家と呼ばれる人のdiscussion でも、そこに書いてあることが
外れていることが多いし、「将来の目標」も放置されてることが多い。
よく出来た論文ってのはアブストと図表だけ追えば、充分にストーリーは
わかるからね。サイエンスはストーリーだという点では、MYと同じ意見かも
しれない。

>>265
金も確かに研究を進める上での大きな要素(エールリッヒのdrei G の一つ)
だけど、金でブレイクスルー出来るんだったら誰も苦労しないって。
日本の特別推進だのCOE だのとんでもない金つぎ込んでも、ノーベル賞が
出てないしこれからも出そうもない。ノーベル賞だけがブレイクスルーの証と
いうわけじゃないけど、ゼニつぎ込んで病気も直せませんでした、国際的にも
評価されませんでしたじゃ、今の日本ならさらし首もんだよ。俺たちの税金を
無駄にしやがってとか暴動が起きるよ。ブレイクスルーは一人の頭脳から
生まれるもんで決して金や組織から生まれるもんではないと考えてるけどね・・
マサキ、エバシの両氏あたりの仕事っぷりを追ってるとますますその感を強く
するし、ボンビーなこっちとしては「よし、俺だって!」と何度もエンカレッジ
されたけどね。
358N.U :2000/10/24(火) 15:08
ところで、そちらは夜の十一時ごろですね?
359名無しさん@お腹いっぱい。 :2000/10/24(火) 15:10
>358
いや、深夜1時を回っているはずです。
360染色体6番薬学博士二世研究者@NIH@next UCSF@年末プロポーズ :2000/10/24(火) 15:12
>NU
MYをヨイショしたいのはわかったから、その細切れの短レスやめてくれ。
361今ダラスは01時14分 :2000/10/24(火) 15:13
日本の時間から2を引いて午前と午後をひっくり返します。
いい質問を待ちましょう。
362ホセ ビスカイーノ :2000/10/24(火) 15:15
やーさんはメキシコ国境の韓国ソープへよく遊びにきてますよ。
363N.U :2000/10/24(火) 15:16
>360
了解。ジャーナリスト的にウォッチングに徹しましょう。これでストップします。
364ホセ ビスカイーノ :2000/10/24(火) 15:16
>361のあほ

テキサス大はAUSTINだよ。
はげ
365ホセ ビスカイーノ :2000/10/24(火) 15:16
やーさん
366ホセ ビスカイーノ :2000/10/24(火) 15:18
Macはよくよく、女遊びに国境まできますね〜
韓国娘目当てでよく、売春してますよ〜
367佐藤純 :2000/10/24(火) 15:20
MACは花の童貞独身だからな〜
今年で40だよ〜
368佐藤純 :2000/10/24(火) 15:20
ある意味記録的だな
40年恋人がいないのも。
これだけでノーベルもにんまり賞
あげたいね
369佐藤純 :2000/10/24(火) 15:21
おっす。
おいらは東工大!の化学工学修士2年の佐藤だよ。
去年の夏から1年ウィスコンシン大学の大学院に留学したんだけど、
おいらは、いまだに素人童貞なんだ。
アメリカに行けば、もてて、白人女と一発できるかと、おもったけど、
結局はエスコの女にテコキしてもらっただけさ。
オイラは東工大だから、頭はいいのに、なぜ、素人道程なのかと
いつもかんがえるんだよな。
ソープにいったときははめそこなったから、実際はヘルスのフェラチオしか経験して
いない、おまんこの感触すらしらないあわれな27歳なんだ。
どうやったら、素人のおまんこ野郎をファックできるか教えてくれ。
まじで。http://www.engr.wisc.edu/groups/fsd/
http://www.chemeng.titech.ac.jp/~fse-grp/OGAWA/index.htm


http://www.chemeng.titech.ac.jp/~fse-grp/OGAWA/member/member.htm

370佐藤純 :2000/10/24(火) 15:22
本籍 東京都中野区
住所 東京都目黒区
氏名 佐藤純
犯罪番号 30−740865 77
指紋番号 左 77967
     右 79997
生年月日 昭和48年7月14日
検挙年月日 平成5年2月17日
罪名 売春防止法違反
検挙警察署 警視庁保安課
処分年月日 平成5年2月25日
処分内容 起訴猶予

総合照会 平成12年5月19日
照会者 東京工業大学
氏名 佐藤純
H5・2・17売春防止法違反 2・25地検起訴猶予
(推定、佐藤純氏が大学2年後期のもよう)


「もうしましぇん若気の至りでして」
371場違いママ :2000/10/24(火) 15:22

 はじめまして、医学とはかけ離れたただの主婦ですが
 書き込みさせてください。
 私の子供は染色体に異常があります。
 13番目の染色体の欠損です。
 現在は知能・運動能力は劣りますが普通学級に通っています
 1歳で染色体の異常が見つかった時に大学病院の先生に
 「寿命は10年ぐらいです」と言われました。
 家庭の医学書などでは13番目の染色体のことは
 詳しく書かれてません。
 ここに来られる方は専門の医学を学ばれているかたが
 たくさんいらっしゃるようなので教えて下さい
 13番目の染色体の異常児はどんな病気にかかりやすいのですか?
 (網膜が細胞種には1歳の時発病してしまったのですが・・)
 もうすぐ10歳になりますが本当に短命なのですか?
 主治医の先生は「こんなに元気に育っている13トリソミーの子は
 まず世界にも例が少ない」と言うのですが本当ですか?
 ここの書きこみに場違いなのはわかってますが
 よろしくお願いします。

 
372元祖ポスドク :2000/10/24(火) 15:24
おっす純
あいかわらずこんにゃくでオナーぶっこいてっか?
いいかげんにしとけよ
こんにゃく買占めだぜありゃ
一日7発してんもんな〜
373>371 :2000/10/24(火) 15:29
まず自分で調べろ
・goo http://www.goo.ne.jp/default.asp?act.expert=1
・infoseek http://www.infoseek.co.jp/
リンク集ご案内用url
http://cocoa.2ch.net/test/read.cgi?bbs=hosp&key=964681615
質問の前に、検索してみましょう。
身体健康板の過去ログを大先生の検索によって調べることができます。
・身体・健康板 http://cocoa.2ch.net/body/index2.html
・大先生の検索http://216.167.29.251/cgi-bin/search.cgi
2ch以外にも医療について調べられるサイトがあります。
・医薬品情報
http://www.pharmasys.gr.jp/index.html
・医療事件について マスコミウォッチ
http://www.tochigi-med.or.jp/~shioya/m_watch/m_watch.cgi
・15万医療機関検索
(医師は地元以外の病院の評判は知らないことが、ほとんどです。
またしがらみもあって、なかなかどこそこがいいとは言いにくいものです。
案外、主治医や最寄りの保健所、ご近所にきいたほうが正確かもしれません。)
http://doctor.nhk-jn.com/
・医学英語 京都ライフサイエンス和英英和辞書
http://zip-translator.pharm.kyoto-u.ac.jp/
・カタカナ医学俗語集
http://apollo.m.ehime-u.ac.jp/~gochi/96/g821zoku.html
・家庭の医学  メルクマニュアル日本語版第17版
http://merckmanual.banyu.co.jp/
・小児科領域
http://www.med.nagoya-u.ac.jp:8888/119/119.html
・調べてわからないちょっとした質問はこのスレッドへ
http://cocoa.2ch.net/test/read.cgi?bbs=hosp&key=971511635
374元祖ポスドク :2000/10/24(火) 15:33
>>371
柳沢はヤブ医者だからわからないよ
375場違いママ :2000/10/24(火) 15:36

>373様
      ありがとうございます。調べてみます
376元祖ポスドク :2000/10/24(火) 15:36
柳沢はベンツ海苔回してオイルらしい
377名無しさん@お腹いっぱい。 :2000/10/24(火) 15:37
ダラスは深夜1時40分。柳沢先生は、お休みになられたと思います。
ちなみに、テキサス大学の本校はオースチンですが、UT Southwesternはダラスです。
378元祖ポスドク :2000/10/24(火) 15:37
柳沢は恐れをなして逃げました
やはり専門馬鹿。
専門医害はわかりましぇんでしゅた
379名無しさん@お腹いっぱい。:2000/10/24(火) 16:10
やっぱり本物だー!記念書きこしよ!
380MY:2000/10/24(火) 16:11
>>357 議論が熱くなってきましたなぁ。こういうの好きだぞ。


>アンタゴニストについて。

んー、もう少しGPCRの一般的なことについて勉強しておいたほうが良いかも。現代の製薬業界が臨床開発にまでもって行くようなGPCR拮抗薬は、尋常な特異性ではありませんぞ。ターゲットGPCRに対するaffinityはsubnanomolarからpicomolarレベルで、その他多数のnegative control targets(100種以上のパネルが使われるのが普通)ではsupra-micromolarまで検証されているのが当たり前です。Signal transduction等の業界で、「特異的」などといわれるprotein-protein interactionなどよりずっと特異的だと思って下さい。つまり、通常のdominant negativeの方法論なんかよりはずっと特異性が高いと思って間違いないです。たしかに、GPCR ligandのmimotopeは普通せいぜいアミノ酸側鎖数個分ていどですが、だからこそ、orally activeな小分子で、かつhighly specificな拮抗薬が作れるんです。あ、それから、endothelin - endothelin receptorsの機能ドメイン解析、ないしはstructure-activity解析の論文数は、100報や200報では済まないです(しかも皆、非常にまともな論文)。Industry research / drug researchを侮ってはいけない。

...と、ここまでは薬理の出としての弁護ですが、一方でアンタゴニストを使った仕事に限界があるのも、百も承知。多くの場合、in vivo geneticsの切れのよさには勝てません。

>discussionについて。

同感。俺も人の論文のdiscussionは、まず読みません。よほど近いものでない限り。でも、「うだうだ書く暇があったら新しい実験をやってそのデータを加えてもっと上のジャーナルに投稿する」という言い切りは、じつに時と場合によりけりです。あなたが次のcareer stageに進んだ段階で、それだけでは少々naiveだったと気付かされるでしょう。

>金について。

金でブレイクスルー出来るんだったら誰も苦労しない、というのは100%同感。しかし残念ながら現在の生命科学では、あるレベルの研究費はブレイクスルーのための必須条件になってしまっていると思います。江橋節郎先生(俺は彼の孫弟子)のスタイルが通用した時代は、悲しいけれど、とうに過ぎ去っています。いくら銭つぎ込んでもダメなものはダメ、であることは確かだが、現代では、ある一定のfundがないところからは、ブレイクスルーの出現はほぼ絶望的でしょう。ブレイクスルーが一人の頭脳(そう、数人ですらない!)から生まれるものだっていうのも100%同感だけれど、そのアイデアをexecute出来るだけのfundは必須です。今の日本のシステムでは、若い研究者にそのチャンスが与えられてないんじゃないの、と言いたかったのです。自分は貧乏だ、貧乏だって研究は出来るんだ、とヘンな諦めをしないで、必要十分な金の来る立場に自分を置きつづける努力・能力も、研究者としての資質のうちだと思う。

江橋先生といえば、彼の筋肉におけるintracellular messageとしてのカルシウムの発見は、真にノーベル賞に値するとおもいます。カルモジュリンの柿内(spell?)先生が亡くなられたのが非常に残念ですよね。でも、今年あたりのスタイルを踏襲して、そのうちBerrigeあたりと一緒に貰えるといいんだが..


ところであなた、aquaporinのことやってる人?
381内科系院生:2000/10/24(火) 16:12
(基本的な質問で失礼します)))

通常の免疫組織でエンドセリン受容体(特にB)を染めた論文が結構あるのですが、
感度が高いと思われる凍結切片でもなかなかきれいに染まりません。他のモノは
とても上手く染まるので、手技上の問題ではないと思います。何種類もの抗体が
販売されていますが、どれがお勧めでしょうか。希釈倍率などもできれば教えて
頂ければ幸いです。エンドセリン関係の論文を書きたいのです。
382うっす! 自己紹介だ!:2000/10/24(火) 16:20
14−3−3=染色体6番=イスカリオテのユダ=元祖ポスドク@海外生活板=
堕ちた研究者=流浪の・・=スイスブラック と多数のHNを持つ俺です。

研究者の家庭にうまれ、東大で薬学博士を取得したクリスチャンです。
最初のファーストデビューはJ. Bacteriology です。
今は、ベセスダのNIHでポスドクしてます。ボスはユダヤ人で、ここでは
中枢神経系に発現する遺伝子のノックアウトマウスの開発を
していますが、アメリカに来てほぼ1年たったので次は
UCSF に行きます。NIH での仕事でファーストオーサー論文が
だせるかどうか心配です。昼食はカフェテリアでラボメイトと
取っているので、見つけたら声をかけて下さい。白人の集団の
中、一人あいまいな笑顔でいる日本人、それが俺です(笑。
今、大陸横断の準備のために愛車のカローラを整備中です。
NIH には日本人が多いのですが、俺は何故かそのグループからは
嫌われていますが、たまにELT のCDを貸してくれる日本人もいます。
それから年末にJ1ビザの更新に日本に一時帰国します。
そのときに、彼女(元彼女かも・・)に告白してオーケーが出たら
アメリカで一緒に生活したいと思います。シアトルに親戚がいるので、
式はシアトルかサンフランシスコかで挙げたいと思います。
海外生活板の友人の酒豪に友人代表のコメントを頼むと思います。
酒豪、また例の紫のスーツでキメてよ。
38318番染色体:2000/10/24(火) 16:30
>357
君、痛過ぎ。

認識部位が数個のペプチドなんて例を挙げたら
きりがない。それらが複合体を形成して機能するのも別に
目新しいことではない。

論文読む楽しみはDiscussionだね。書き手の考え方がわかる。
下手な小説読むより面白い。君はアブストしか読まないのか。
論文作成したこともないようだな。

研究は金。予算がなければ人を雇うことも出来ない。
潤沢な予算がなければ、革新的な研究より堅実な結果が絶対に
出るような研究しか出来ない。
どんなプロジェクトでもそれなりの成果は問われる。
結果かゼロで許されることなど額に関係なく、あり得ないことだ。
予算を貰う立場になって下さい。

それから染色体6番とはどういう意味でつけたのかな?
特徴があったかな?
384名無しさん@お腹いっぱい。:2000/10/24(火) 16:35
荒らしと混在して何が何だか。
本物でも何でも良いけども。
学会で講演でも聞きにいけばいいか。
385名無しさん@お腹いっぱい。:2000/10/24(火) 16:42


178のリンクにあるアクアポリンの安井さんはここで荒らしてる変なポスドクとは
別人だと思いますが・・・・。彼は筑波出身ではなく慶應です。
38618番染色体=染色体6番:2000/10/24(火) 16:44
>>380

なるほどね・・
鎌田-江橋-真崎-MYというラインか・・
サラブレッドだね。あと正確には垣内だよ(藁。

まあ、金のことを言い出せば切りがないからやめとくけど、
言いたかったのは金よりもアイデアで勝負したいということ。
俺も、日本でははぐれ者だったから自分でかなりの額を集めざるをえなかった
し、アメリカでもそれなりの国際フェローシップももらってるよ。
まさか、どん底のボンビーだと思ってたわけかな?
まあ、コネで取ったHHMIの大先生にはゴミ扱いされても無理ないけどね(藁。
それから、エバシの装置は当時としてはかなりのもんだったと思うけどね。
イトウも真似してカリウム除去の装置を作ったというけど、それにしても
当時なら結構な金をつぎこんだはず。

アンタゴニストについては、わざとナイーブに煽ったわけだけど
まあ、優等生の回答が返ってきたんでほっとしたよ・・

あと俺のフィールドとプロファイルだけど、>>382で簡単に
述べた。NIHにいる奴ならすぐにわかるよ・・
UCSFにあきたら、MYラボにでもアプライしてみるかな(藁
決して損はさせるよ(藁
387名無しさん@お腹いっぱい。:2000/10/24(火) 16:49
UT Southwestern?

i ve never heard of it.
thats a Mother fuckin univ.

i thought Mother fuckin Yanagi was from UT-Austin.
388名無しさん@お腹いっぱい。:2000/10/24(火) 16:52
`Yanagi Mother fuckin JAP
389>383 18番染色体:2000/10/24(火) 16:54
おまえの話し、具体的でないんで面白くない。
MDじゃないんだろ(藁
厨房ががまんしきれなくて、飛び出してきたって感じで笑えるけど
あんまりスレ汚しするなよ、ぼうや。もう寝ろ。
研究は金と言い切るあたりがMY先生とは違ってまだアオイね。
390名無しさん@お腹いっぱい。:2000/10/24(火) 16:54
>Motherfuckin Yanagi

Dont be confused as if your became a mother fukin celebrity.
Aint no shit.
U aint no motherfuckin nobel prize mother fuckers

Fuck yo.
391名無しさん@お腹いっぱい。:2000/10/24(火) 16:56
Mother fuckin Yanagi

You aint give us no shit.
mother fuckin garbage men are more mother fuckin productive.
Mother fuckin jack off.

392名無しさん@お腹いっぱい。:2000/10/24(火) 16:57
all you mother fukin doctors.

you got mother fuckin Players hate Degrees?
393名無しさん@お腹いっぱい。:2000/10/24(火) 16:58
Go and get fucked.

Mother fuckin Yanagi
394名無しさん@お腹いっぱい。:2000/10/24(火) 16:58
i hate mother fuckin brain bustards like yo
395名無しさん@お腹いっぱい。:2000/10/24(火) 16:59
By the way I m a fuckin nigger bitch.
396名無しさん@お腹いっぱい。:2000/10/24(火) 17:01
.mother fuckin Yanagi

You aint no ignore 371?
Mother fucker
397名無しさん@お腹いっぱい。:2000/10/24(火) 17:08

>他人の経歴を騙って意見を書いている人

そういう事を続けていて虚しくないですか?
どんなに良い意見?を書いたところで自分の意見にはならないですよ。
自分の経歴で堂々と書き込みをして下さい。
貴兄も研究者を目指す者であるのならば。
39818番染色体:2000/10/24(火) 17:12
え? 俺のこと?
399酒豪:2000/10/24(火) 17:29
        
        
        
         よ〜!ひさしぶりポスドク
            早く海外生活に戻ってこいよ
400名無しさん@お腹いっぱい。:2000/10/24(火) 17:31
>Fuckkin yanagi


Mother fucker is fuckin his wife.
3AM
401コロ助@お休みver.1:2000/10/24(火) 17:37
おや博士殿、こんなところにいらしたとは。
オイラもさびしいよん。
402ベルタン男爵:2000/10/24(火) 18:06
こんな糞スレしか立てられないお前は即刻タンクトップと短パンに着替えて
公園で走って来い。ボケ。
日頃感じているストレスを忘れるほどに無心で走れ。ボケ。
5kmほど走った時、お前はある人物と出会う。ムロさん(自由人)だ。ボケ。
挨拶もそこそこに貴重な酒が振舞われる。絶対飲め。ボケ。
飲んだ者だけがムロさんの家(ムロハウス・ビニール)に招待されるんだからな。
ボケ。
そこでムロさんによるありがたい説教をうけろ。ボケ。
お前はそこで気付く。知らず知らずに涙が頬を伝っていることにな。ボケ。
お前はそこで気付く。自分がいかに愚かだったのかということにな。ボケ。
説教をうけたら、感謝の気持ちをこめてタバコを差し出せ。ボケ。
言葉は要らない。それがムロさん(フリーマン)への礼儀だ。ボケ。

ムロさんの言葉を噛み締めながら家まで走れ。ボケ。
大きくなって帰ってくるお前をここで待ってるぞ。

403>398 その他の18番染色体:2000/10/24(火) 18:09
君、読解力にかけるね
404>402:2000/10/24(火) 18:13
コピペうざい
405Weibel-Palade命:2000/10/24(火) 18:32
第3回エンドセリン研究会で私の発表に柳沢先生が質問を
してくださったのを今でも光栄に思っているものです。
その後もずっとエンドセリン関係の研究をしています(形態屋です)。
形態学関係の質問が出ていましたので甚だ僭越ながら、
381さんに横レスさせてください。

下記論文が参考になると思います(ただし、ラット)。
ヒトなら北里研メディカルセンターの内科が頑張ってます。

Fukushige H@` et al.
Synthesis and receptor sites of endothelin-1 in the rat liver vasculature.
Anat Rec. 2000 Aug 1;259(4):437-45.
406名無しさん@お腹いっぱい。:2000/10/24(火) 18:44
>>397の書き込みはMY先生をまだ偽と疑ってるのかな?
しつこいやつだ。
407FORCEPS:2000/10/24(火) 19:31
>328 Dr. M.Y.
 日本人の医学・生物学系研究者は相当な数が欧米の超一流のラボに留学し、そのうちのかなりの数が指導的立場になっているはずですよね。それを考えると、日本のシステムの変化は歯がゆいほど遅い。他人を評価することが苦手、かつ/または評価基準の多様性に欠けるのかなあと思います。研究費を取るためもありますが、日本中の大学で流行に遅れないように同じような系統の研究を追いかける傾向にあるように感じます。これはアメリカでもある程度は見られる現象でしょうが、日本の方が画一化がはっきりしているようです。

 もっとも、アメリカでは日の当たらない研究を細々と続けている人を根気強く置いておく鷹揚さも併せ持っているように思います。動く遺伝子でノーベル賞をとった畑大好きな女史もそんな一人ではないでしょうか。公務員の身分保障をたてに、IFの低い論文さえ書かず大学に残っている人間が沢山いる日本の大学(主に基礎)から、あのようなユニークな研究が出ないのが不思議です(藁。アメリカでは、見ている人はちゃんと見ているってことなんでしょうかね。

 論文を書かない教授がいるということでは、筑波はまさにその典型例だったようですが、日本の大学の中では第一段階の改革にかなり成功した方じゃないかと思います。江崎氏が学長に就任させることは、その運動自体が重要なきっかけになったように聞いています。

(注)(藁、苦笑のことです。
 例:日本語の総説で使ったら受けるかもしれません(藁。

さて、ムンテラしてこようっと(>331、身分は……ということです。名を名乗るほどのもんじゃござんせん)
408内科系院生:2000/10/24(火) 19:45
>405
ありがとうございます。さっそく取り寄せてみます。もしかして産業医大の先生でしょうか。
論文ができたら引用はWeibel-Palade命 et al. 2 channel: 405@` 2000なんちゃって。
409名無しさん@お腹いっぱい。:2000/10/24(火) 19:57
>>405

Anat Rec.ってインパクトファクターどれくらい?
ネットで調べても出てなかった。1未満?
410元祖ポスドク:2000/10/24(火) 20:02
おまえらみんな皆殺しじゃ
411元祖ポスドク:2000/10/24(火) 20:05
柳沢はやろうのティムポしゃぶってるらしいな。
412名無しさん@お腹いっぱい。:2000/10/24(火) 20:09
413元祖ポスドク:2000/10/24(火) 20:10
おいらのIF3程度
414元祖ポスドク:2000/10/24(火) 20:10
おいらはIF3程度ドキュン
415元祖ポスドク:2000/10/24(火) 20:11
柳は朝鮮人
416名無しさん@お腹いっぱい。:2000/10/24(火) 20:11
スレッド名が呼びつけってのも…
417元祖ポスドク:2000/10/24(火) 20:11
柳沢もたぶん朝鮮人
418元祖ポスドク:2000/10/24(火) 20:11
>柳沢
おまえチョン公だろ?
419元祖ポスドク:2000/10/24(火) 20:12
柳と言う苗字は朝鮮人。
柳沢はその亜流
420元祖ポスドク:2000/10/24(火) 20:12
おまえはやくピョンヤンにかえれよ
ワカハゲ
421名無しさん@お腹いっぱい。:2000/10/24(火) 20:37
Yanagisawaを偉そうに誰だっ!ていってるあたり、
1は間違いなく“悲惨な1”認定だな。
422名無しさん@お腹いっぱい。:2000/10/24(火) 20:48
基礎って、成功すれば柳沢先生のように一躍スターで華やかだが、
その影で芽のでない敗北者がごろごろしてるんだよな。
ここ荒らしてる奴等とかみてると悲惨な人生なんだろうな
としみじみ感じてしまう。外科医で良かった。
423ベルタン男爵:2000/10/24(火) 20:48
>>421でも面白くないので悲惨な1スレには
乗せられません。
424名無しさん@お腹いっぱい。:2000/10/24(火) 20:50
>>421
それは違うと思う。先生のことを詳しく知ってる人ならまだしも、
知らないなら普通じゃないか?本人が来るなんて思ってもいないし。
1は単に知りたかっただけだろ。
425名無しさん@お腹いっぱい。:2000/10/24(火) 20:55
1はともかく、のこのこ出てきた嵐どもが悲惨な人生送っていることは
間違いないね、哀愁漂ってるし。
426レプチンの心:2000/10/24(火) 21:43
ここで柳沢先生を執拗に煽っている方々って一体なにがしたいのですか?」
全然見えてきません。個人的な恨みなどをお持ちなのでしょうか?
427>427:2000/10/24(火) 21:45
嫉妬。
428427:2000/10/24(火) 22:11
ううっ、>426の間違い。

打つ打指嚢
429潜伏中:2000/10/24(火) 22:13
>426
「柳沢先生」を煽っているのではなく、海外生活板で
「ポスドク=堕ちた研究者」という固定ハンドルネームを
名乗っていた者を煽るために、
「そのハンドルネーム」を「使用」して多くのクダラナイ書き込みを
繰り返しているようです。

「責任の一端」はそのハンドルネームを使用していた(海外板で)
自分にあるかもしれません。
ちなみに自分がここに「煽り」の書き込みをしたことは一度もありません。

スレを汚してすみません。また潜伏します。
430>429:2000/10/24(火) 22:20
ご愁傷様です。2chで個人を特定されるような投稿はやはり
タブーですね、小生も気をつけます。
431名無しさん@お腹いっぱい。:2000/10/24(火) 22:24
>>397はこのスレで大学名を書き込みかつ荒らしている人に対してです。

432名無しさん@お腹いっぱい。:2000/10/24(火) 22:38
医学の話と全然関係ないですが、柳沢先生は日本の教育についてどう思われますか?
日本の大学にはモラルに欠けた教授が多いような気がしてなりません。地位だけで
頭の硬い無能な教授が多い。日本の国立大学の昇進システムがおかしいと思います。
私は柳沢先生のような若い先生に日本の教育を引っ張っていって欲しいと思います。
ありがとうございました。
433堕ちた研究者:2000/10/24(火) 22:52
>Fuckin Korean Yanagi

Why did you pretend to be a professor of UNIVERSITY OF TEXAS--AUSTIN?

The reality is U are the professor of Mother Fuckin University of Texas--something (Dallas TX).
I ve never ever heard of such a mother fuckin university.

U SUCK
434堕ちた研究者:2000/10/24(火) 22:55
>432

U bet
435名無しさん@お腹いっぱい。:2000/10/24(火) 23:00
103 名前:名無しさんの初恋 投稿日:2000/10/20(金) 17:04
http://www.engr.wisc.edu/groups/fsd/
http://www.chemeng.titech.ac.jp/~fse-grp/OGAWA/index.htm
http://www.chemeng.titech.ac.jp/~fse-grp/OGAWA/member/member.htm
佐藤純
HN=Dr.DRE
東工大化学工学修士2年
小川研究所所属
27歳
[email protected]

104 名前:名無しさんの初恋 投稿日:2000/10/20(金) 17:06
佐藤純
東京都中野区野方2−15−12
[email protected]
[email protected]
436 潜伏中 :2000/10/24(火) 23:04
submarine pitching
437潜伏中:2000/10/24(火) 23:04
SUBWAY SERIES GOGOGO!!!
438潜伏中:2000/10/24(火) 23:05
sou kougeki
DO OR DIE
maximum attack
439潜伏中:2000/10/24(火) 23:06
fuck up otiken

here comes a chance
440潜伏中:2000/10/24(火) 23:08
9;00 AM in Texas
441名無しさん@お腹いっぱい。:2000/10/25(水) 00:08
Yasui Masato
Yanagisawa Masashi
どっちもY.M.やね。
なんか因縁でもあるんとちゃうか?>マイケル
442http://133.87.77.77.2ch.net/:2000/10/25(水) 00:13
guest guest
443エンドセリン拮抗薬、変換酵素阻害薬の研究開始を笑った元研究者:2000/10/25(水) 00:30
>柳沢先生はじめまして(100まで読んだところで)
87年秋の薬理学会関東部会は??自治医大でしたっけ??
忘れてしまいましたが、エンドセリン発見の発表を聞きましたよ。
あ、、、しばらくは薬にならないなと、、、
それが私の感想でしたね。

EDCFがペプチドでEDRFが短命な無機低分子であったことが
エンドセリンを発見できた理由でしょうか?
でも、in vitroではアセチルコリンが収縮反応を起こすが
in vivoでは弛緩反応を起こすことから始まる
血管内皮細胞の生理機能の探索から考えると
エンドセリンの発見はhANPの発見と
そう大きい違いが無いのではないでしょうね。
そうなると、ご自身のおっしゃる通り100のレスは本物かなと??
学問的にはノーベル賞は遠い。
444>442:2000/10/25(水) 00:31
;; AUTHORITY RECORDS:
87.133.in-addr.arpa. 86400 SOA nameserv.sys.hokudai.ac.jp. postmaster.nameserv.sys.hokudai.ac.jp. (
1000004358 ; serial
3600 ; refresh (1 hour)
900 ; retry (15 mins)
3600000 ; expire (41 days 16 hours)
86400 ) ; minimum (1 day)
445MY:2000/10/25(水) 00:55
>>381

エンドセリン受容体の免疫組織染色は、少なくとも我々は成功したためしがありません。単にウデが悪いだけかも知れませんが。エンドセリン受容体の免疫組織を用いた一部のpublicationの信憑性は、大いに疑うべきものがあります。とにかく、抗エンドセリン受容体抗体で染めに使える物はあまり見あたらない、というのが私の観察です。もっとも、ここ数年は探してすらいないので、新製品がどこかから出ているかも知れませんが。お役に立てなくて御免。
446MY:2000/10/25(水) 00:59
447MY:2000/10/25(水) 00:59
448だめドクター:2000/10/25(水) 01:05
>ブレイクスルーが一人の頭脳(そう、数人ですらない!)から生まれるものだっていうのも
100%同感
と、ありますがもしも自分よりも優れた、到底かなわないと思わせるような若造(20〜30歳)
の人間が自分のラボに入ってきたらどうしますか?
また、年齢が上下に関わらずそういう人にはどのくらいあったことがありますか?
また、そう思わせるのはどういったときでしょう?いい質問ができること意外に、もし
あれば宜しくお願いします。
指導者というものが自分よりも優れた人間に出会ったときにどう対処するか知りたいです。
あまりいい質問ではありませんね。
449MY:2000/10/25(水) 01:11
>>382 自己紹介ありがとう。

やはりどういう人と話しているのか判らないのは時につらいものがありますな。で、現在KOマウスを作っておられるようですが、それだと、普通1年ではとてもとてもプロジェクトを完遂できないのでは?その仕事が終わる前にUCSFに移ってしまわれるのですか?おそらくいろいろな事情があるのでしょうが、何かそれだけ聞いていると、一寸もったいないようにも見えます。特にそれが自分の論文になりそうにないのなら..一つのラボに1年というのは、いかにも短いと思いますが、いかがでしょう。

それから、貧乏だと自分で言っておられる、と申し上げたのは、もちろん個人的な経済状況のことではなく、研究費のことを指していると解釈しましたが。気に障ったのなら御免。

最後に、クリスチャンとのこと。奇遇ですが、俺もそうです(どうだ、皆さんこれは知らなかっただろ)。どうぞよろしく!
450MY:2000/10/25(水) 01:15
>>405 >>381

おぉーっと。形態のプロの方が既にレスしておられましたね。俺の上記>>445の回答は取り消しまーす。
451薬学博士@二世研究者@NIH@next UCSF@年末プロポーズ :2000/10/25(水) 01:45
>>449 MY先生
早速おれのケチな自己紹介にレスどうも。いまはランチタイムです。
おれの実験の心配までしていただいて恐縮です。
おっしゃる通りです。狙いの遺伝子の局在マッピングから
GFP付きのコンストラクトまで一人で作って進めてたんですが、
ボスの方針でファーストオーサーになれない可能性大です。
しかも、今のラボには自分のフェローシップで滞在しているのに(涙。
そのため気分がすさんで毎晩サミュエルアダムスをあおりながら、
2ちゃんで憂さを晴らしていました。その気分がおれの書き込みに
反映されて、MY先生の気分も害したかもしれませんが、御容赦下さい。

>>445で話題になっている受容体の染色ですが、お家元でも成功していないというのは驚きました。コマーシャルの抗体ならFitzgerald IndustriesやChemiconから出てたと思いますが、試していないので性能の方は不明です。

それから10月30日に合わせて御帰国(来日?)されるそうですが、受賞おめでとうございます。あのメンバーだとMY先生は若いほうから二番目だと思います。ほんとうにすばらしいことだと思います。ではまた。

堕ちた研究者
452薬学博士@二世研究者@NIH@next UCSF@年末プロポーズ:2000/10/25(水) 02:04
>>449 MY先生
思い出したんで追加です。
受容体のアミノ酸配列が種間でconservative なときには、
特異性の高い抗体って作りにくいですよね。エンドセリン受容体が
どの程度まで種間で保存されているのかわかりませんが、生体に
基本的に必要な分子なので、かなり種間での相同性は高いと思ったのですが、
見当外れかもしれません。
453>382:2000/10/25(水) 02:42
まさかprion関係の方ですか?
14−3−3なんてCJD以外ではあまり聞かないですよね。
UCSFということはいよいよ大御所Prusinerのところに乗り込むのでしょうか?
454薬学博士@二世研究者@NIH@next UCSF@年末プロポーズ:2000/10/25(水) 02:50
>>453
niyari

>>382
455>382=453:2000/10/25(水) 03:00
prionのKO mouseでは“Doppel”が話題ですよね。
もしや、それ関連の仕事をしているのですか?
456名無しさん@お腹いっぱい。:2000/10/25(水) 03:22
>薬学博士
non M.D.は逝ってよし!
457名無しさん@お腹いっぱい。:2000/10/25(水) 03:33
>443 エンドセリン拮抗薬、変換酵素阻害薬の研究開始を笑った元研究者

なんて失礼な事言うんだ、あんたは!
先生 非礼をお詫びします。なにかあったら、ここの医者みんな
でフクロにするので、どうぞご寛容に。
458MY:2000/10/25(水) 04:06
>>451 NIH →UCSF

う〜む。自分で給料持ってきているのに、ですか。それは一寸ひどいですねぇ。まぁ、特に東洋人のポスドクには良くある話、といえばそれまでなのかも知れませんが。お察し申し上げます。しかし、そこから先のKOプロジェクトを「横取り」する人は一体どういう御仁なんでしょうね。どうやってボスを丸め込んだ(?)のでしょう。
459MY:2000/10/25(水) 05:05
>>448

「ブレイクスルーは一人の頭脳(そう、数人ですらない!)から生まれる」と言いましたが、それは俺のラボから出る仕事のアイデアが全て俺自身によるものだ、などという不遜なことを意味しているのではありません。誤解があるといけないので念のため。例えば、99年に発表した、オレキシン欠損がナルコレプシーを起こすという発見は、論文の第一著者であるアメリカ人M.D.ポスドク(Rick Chemelli)の独創的なアイデアに端を発しています。彼はその後、うちの大学の小児科のtenure-track assistant professorになり、10% clinical duty / 90% researchの保護のもと、俺のラボで精力的にオレキシングループを牛耳り、semi-independentな仕事を続けています。いま申請中のNIH RO1が通れば、彼も晴れて完全な「PI]になって行くのです。それでも共同研究は続けたいですし、続けるでしょうが。

前に>>98で言ったように、論文のストーリーの根幹をなすアイデアを自分で出せる人は、アメリカでは必ずやがてPIへの道を進めるのです。

いままで、明らかに自分より優れた所のある「部下」に出合ったことは数回あります。前出の桜井君(現筑波大学助教授)とは、90年にエンドセリンB受容体の同定(Nature)、98年にorphan GPCRリガンドとしてのオレキシンの発見(Cell)、という二つの大きな仕事を一緒にする機会がありましたが、彼の実験センスやアイデアへの鼻の良さは、時に本当に敵わないなと思わせられます。また、上記のRick Chemelliも、彼のobservationの精緻さは、俺が自分で実験していた頃のことを思い出すと恥ずかしくなるくらいのものがあります。彼の「物を見る眼」が無かったら、ナルコレプシーの仕事は絶対に出なかったでしょう。

科学者には、よく言われるように個々色々な「タイプ」があります(例えば、有名な『成功する科学者』参照)。一人一人、長所と欠点がありますから、ある特定の事項に関して明らかに自分より優れている人をsuperviseする立場になっても、俺としてはdefensiveになったりしないし、なる必要もないのです。彼らに与えることの出来そうなもの(経験に基づく判断・哲学など、有形・無形を含めて)を出来るだけ伝授するのが、PIとしての責務ではないでしょうか。その上で、お互の能力を補い合えるような関係が築ければ、最高ですよね。
460MY:2000/10/25(水) 05:31
>>432 日本の(大学院)教育について

言いたい意見はたくさんありますが、敢えて一つに絞るとすると、「日本の生命科学系大学院は、教育システムとしては有名無実である」となります。俺自身の過去も含めて、日本の大学院生は、将来PIとしてやって行くためのサバイバル技術を正式に教わる機会がほとんどない、と言えると思います。まぁ、優秀な学生は、所詮放っておいても自然に自分でサバイバルを修得して行くものですが、そういう恵まれた少数の背後に、きちんとトレーニングされないで野放し状態になっている平均的学生が沢山いる、というのが現実ではないでしょうか。

アメリカの大学院生は、実によく勉強します(強制的にさせられます)。まず最初の1年〜1年半は、フルタイムで授業・レポート・テスト・抄読会の連続ですし、2年目の夏頃に「Qualifying Exam」といって、フルサイズのグラント申請とそのoral defenseの練習のようなことをやらされます。つまり、ラボでの自分の実験だけでなく、幅広いknowledge baseの取得とともに、口頭・文書によるformal presentationの訓練を徹底的に受けるのです。このようなトレーニングが、後にポスドクを終えPIにならんというときに大いに物を言います。それに比べ、日本の平均的大学院生は、ただラボでの実験を遂行し、自分に与えられたプロジェクトを確実にこなせば100点満点と思っている人がほとんどだし、教官の側も、その認識に完全に寄り掛かっていて、大学院生は実質的に実験手技のトレーニングしか受けていない、というのが実状だと思います。PI予備軍ではなく技官としての訓練です。

そういう教育しか受けて来なかった人が、やがて教授になり、ボスとしての「既得権」(学生を実験の手足としてコキ使うことも含め)を得るわけですから、このシステムがなかなか変化しないのは残念ながら当たり前です。この辺にも、日本の研究者の「層の薄さ」の原因があるのではないでしょうか。
461MY:2000/10/25(水) 05:34
>>451

ひとこと言い忘れました。ストレスが溜まった時のビールは、やはりギネス・ドラフト(例のピンポン玉入り缶)でしょう!是非おためしを。値段も、ほとんど変わりません。
462名無しさん@お腹いっぱい。:2000/10/25(水) 05:42
先生のストレス解消法はは何ですか?
また,研究がうまくいかないときの気分転換法は?
463薬学博士@二世研究者@NIH@next UCSF@年末プロポーズ:2000/10/25(水) 06:03
>>455
niyari
>>456
iyan

>>382
464だめドクター:2000/10/25(水) 06:12
 >>448
 どうも、大変有り難うございました。聞きたかった満足のいく答え(>一人一人、長所と欠点がありますから、ある特定の事項に関し
て明らかに自分より優れている人をsuperviseする立場になっても、俺としてはdefensive
になったりしないし、なる必要もないのです。etc・・・)が返ってきまして喜んでおります。僕と
してもそういう指導者があたりまえと思いますし、そういう指導者のもとで研究を進めて
いきたいと考えておりますがなかなか・・・(日本には帝王気取りのボス多い?)僕自身の性格も災いしているのでしょうけど。
僕自身の研究分野では、日本にはなかなかいないのかもしれません(有名どころは
悪いうわさが・・・真偽は別として)。次のボスあたりは本当に将来を決めそうなので慎重に・・と
アメリカへの脱出を試みたいと思っております。実現可能か、不可能かは別として。貯金0ですし。
もっとも、だめドクターと名乗っておきながらまだドクターですらないのですが。
来年三月薬学博士取得予定?
やっぱり、金のことなんて問題にするべきではないのでしょうけど。以前、TVで
日本人ノ−ベル賞受賞者のT博士が「しがらみがあってより研究に適した環境に移れない
という人は本当に研究がしたいとはいえないと思います。」とおっしゃっていたの
が耳をついて離れません。はー、耳が痛い。
というわけで?どうもありがとうございました。だめドクターは博士論文作成に
戻ろうと思います。なんか文章下手ですね。でもそのまま載せます。

465MY:2000/10/25(水) 06:33
>>452

それもあると思いますが、やはり何と言っても、「多くのGPCRは何故かとてもpoor antigenである」という多くの方が同意する大原則がここでも働いてしまっているのでしょうね。
466名無しさん@お腹いっぱい。:2000/10/25(水) 06:44
88年の先生のNatureのcitationをISIで調べてびっくり、
目が・になりました。
6686

因みに98年の桜井さんのCellは
255、

ホットペーパーで有名な長田先生の98年のNature Articleは
663

蛇足ですが同時期に出版された小生のPaperは5です。トホホ!

どうすればホットペーパーがかけるのでしょうか?
ホットドッグを毎日かじっていてもやっぱり無理でしょうね。

ところでそのTM先生は実力とは反比例して、どうして英会話があれほど下手なのでしょうか?
国際学会の発表の質疑応答でこちらまで恥ずかしくなってしまった。
467MY:2000/10/25(水) 06:45
>>462 ストレス解消法

ひとつは、眠りこけることですね。俺には、必要とあらば10数時間にわたる惰眠を一気に貪る能力があるのです。
気分の波は結構大きい方なので(外には必ずしも出さないが)時に苦しくなります。自分では、完全に躁鬱気質だと思います。

今は、研究そのものはうまく歩んでいます(つもりです)が、ちとラボ内の人間関係の調停ストレスが溜まっておりまして、ここへのカキコも良い気分転換になっていますから、ご安心下さい。こうして、普段文章にしたりしない自分の意見を書いてみるというのも時にはいいもんですな。
468薬学博士@二世研究者@NIH@next UCSF@年末プロポーズ:2000/10/25(水) 06:50
>>466
who is Dr. TM?
469名無しさん@お腹いっぱい。:2000/10/25(水) 06:53
先生はgraduateやundergraduateのstudentのeducationの義務はあるのでしょうか?
もしあるならば年間何ヶ月くらいですか?
授業のための下準備は?
日米の講義でここが違うという事は、それと学生のレスポンスと質はどうでしょうか?
学生に発音がひどすぎて理解できないと言われた経験は?

この病院・医者の板を「柳沢」専用板にすることを提案します。
470466:2000/10/25(水) 06:56
TM was Yanagisawa's teacher !
471薬学博士@二世研究者@NIH@next UCSF@年末プロポーズ:2000/10/25(水) 06:59
>>470
You might use a word mentor in that context.
Any way@` thx.
472名無しさん@お腹いっぱい。:2000/10/25(水) 07:01
>469
柳沢先生は英語の発音はネイティブ並みだ。
473466:2000/10/25(水) 07:03
Thank you for good suggestion.

Mentor means supervisor.
474薬学博士@二世研究者@NIH@next UCSF@年末プロポーズ:2000/10/25(水) 07:05
>>470
I was really impressed with his work on the myosin isoforms during development.
475名無しさん@お腹いっぱい。:2000/10/25(水) 08:19
>親愛なるMY先生

よろしければこちらにも遊びに来て下さい。
お待ちしております。(´ー`)y-~~

「柳沢先生の話がわからない人のスレッド」
http://cocoa.2ch.net/test/read.cgi?bbs=hosp&key=972281677
476名無しさん@お腹いっぱい。:2000/10/25(水) 08:22
>475
先生は忙しいんだ。ったく、そんなもん自分たちで解決しろ!
477名無しさん@お腹いっぱい。:2000/10/25(水) 08:23
今、食事中です。
478名無しさん@お腹いっぱい。:2000/10/25(水) 08:24
>>466
>88年の先生のNatureのcitationをISIで調べてびっくり、
目が・になりました。
6686

因みに90ー96年までの世界のトップ5は、
S. F. Altschul: 4100
S. Moncada: 3889
T. A. Springer: 3400
R. O. Hynes: 2642
J. Schlessinger:2615でした。

>蛇足ですが同時期に出版された小生のPaperは5です。

お前のはCold paper言うんじゃ!日本語で「凍るどペーパー」じゃ!
しかし、日本から出されるサイエンス論文のcitationの平均は
確か1.4なので平均より大幅に上を行っている。
479名無しさん@お腹いっぱい。:2000/10/25(水) 08:36
>>478
じゃあ、90年代に出た論文で、柳沢先生の論文を上回ったものはないということ?凄すぎる!
480名無しさん@お腹いっぱい。:2000/10/25(水) 08:36
どうしてHHMI研究員になると論文の所属が最初にHHMIになっちゃうんですか?
元々の所属が2番目になるなんて、ちょっとずるいような気がするんですが。

だから機関別IFがHarvardを大きく抜いて、1位になっちゃうんですね!
481478:2000/10/25(水) 08:59
上のトップ5は90ー96年の足掛け7年の数値で
88年のYnagisawaのNatureは足掛け12年の数値
なので残念ながらトップ5ではないと思います。
しかし確実にトップ20くらいは行っているのではないのでしょうか?
しかしこれは出版後、何年以内を見るかでcitationは変わってきます。
最初、すごくても5年で尻つぼみのものもあるし、逆に後から評価されて
じわじわ伸びていくものもあるし、

因みに90ー96年間でcitation300以上のホットペーパーを持つバイ
オ研究者の世界のトップは2位のKinzlerを10報以上引き離して22報の
p53のVogelsteinちゃん!
われらが阪大のゴリラ総長・岸本天皇が8報で堂々の17位、
論文1報あたり446.9回引用されている。

482名無しさん@お腹いっぱい。:2000/10/25(水) 09:06
>481
おまえは何報もっとんねん?
483自分たちで調べようよ:2000/10/25(水) 09:06
>>475
申し訳ないけど「話が解らない人たちの」スレで解りやすい方の話を読みたい者です。
Expert discussionをRead Only Memoryするのに徹した方がいいと
思います。書き方のみ真似して見ました(爆)。
484薬学博士@二世研究者@NIH@next UCSF@年末プロポーズ>>382:2000/10/25(水) 09:24
>483
君の日本語もわかりにくいけどね・・
485名無しさん@お腹いっぱい。:2000/10/25(水) 09:34
引用される数が多い論文が重要な論文であることは認めるけど、
単なる数字で論文や研究者を評価するのには抵抗を感じるな。
単に論文数で評価するよりはましだろうけど。

確かに数字はわかりやすいけど、
科学をとしてどう評価するかというのは、
それとは別個になされなければいけないんじゃない?

少し話は違うかもしれないけど、大学の個性を無視して、
偏差値で序列をつけてしまうのと似ていると思う。
アメリカだったら、Harvard と MIT と Swarthmore College の内どれが
一番かというのは学生によって違うんじゃない?

柳沢さんはどう思います?
486チャネラー:2000/10/25(水) 10:54
柳沢先生、楽しく拝見させていただいています。
私は38才の循環器医です。今は市中病院にて臨床に明け暮れています。
大学院の時に基礎研究をおこなっておりました(主にイオンチャネルですが)。
現在の医学研究において、臨床医が大学院の仕事、学位の仕事、などで比較的いい仕事を
しても、その後医局人事で地方病院へ派遣(学位のお礼奉公)されたり結局臨床へ戻って
しまい、せっかくのいい仕事が尻すぼみになるケースが多々あります。また基礎の教室でも
PhDが人生をかけて仕事に没頭しているところへ、臨床系の人間が片手間で研究をかじるという
ケースも多いと思います。

アメリカでのMD(臨床医)のbasic reserchに対する取り組みかた、また現在の日本のあり方に
ついて先生のご意見を教えてください。

まえいた研究室ではずいぶん薬学の先生に医者が片手間に研究やられてもと煙たがられました。
487名無しさん@お腹いっぱい。:2000/10/25(水) 11:59
>>458
論文は多分、大丈夫です。
形勢が変わって来ました・・というか、自力で変えたんですが。
研究はかけた時間に比例するものでは無いと実感しました。
488475:2000/10/25(水) 12:40
>>475は先生に説明を求めて書いたわけじゃないよ。
何気ない気楽なことでも書いて欲しいなって思っただけ。
こっちでは学術的なことが中心になるからね。
だから「遊びに」って書いたの。(´ー`)y-~~
489名無しさん@お腹いっぱい。:2000/10/25(水) 12:43
   Λ_Λ   / ̄ ̄ ̄ ̄ ̄ ̄ ̄ ̄ ̄ ̄ ̄
  ( ´∀`)< 記念カキコだモナー
  (    )  \___________
  | | |
  (__)_)
49014-3-3@NIH to UCSF:2000/10/25(水) 13:05
   Λ_Λ   / ̄ ̄ ̄ ̄ ̄ ̄ ̄ ̄ ̄ ̄ ̄
  ( ´∀`)< me too
  (    )  \___________
  | | |
  (__)_)
491名無しさん@お腹いっぱい。:2000/10/25(水) 13:07
やっぱり、偉大な学者になるには体力もなきゃいかんのではないかな。
やなぎさわせんせいは、徹夜で車を運転して筑波から新潟へスキーにでかけ、
現地でスキーブーツを忘れたことに気がつき、再びそのまま筑波に
とりにかえり、それから再びスキーをやったとさ。
(ところで、やなぎさわせんせ、ほんとうですか?このいつわ)
492堕ちた研究者@海外生活板:2000/10/25(水) 13:09
>>456

675 名前: 堕ちた研究者 投稿日: 2000/10/25(水) 13:05
MDなんて満足に実験出来ないヤツが多いよ・・
493名無しさん@お腹いっぱい。:2000/10/25(水) 13:32
    /∵∴∵∴\
   /∵∴∵∴∵∴\
  /∵∴//   \|
  |∵/   (・)  (・) |
  (6       つ  |    / ̄ ̄ ̄ ̄ ̄ ̄ ̄ ̄ ̄
  |    ___ |   <  よくしまってるよ、父さん
   \   \_/ /   ζ \_________
     \____/   / ̄ ̄ ̄ ̄ \
  /⌒  - - ⌒\/\   / " \|
/ /|  。   。丿 | |   (゜)  (゜) |
\ \|    亠  | (6-------◯⌒つ |   / ̄ ̄ ̄ ̄ ̄ ̄ ̄ ̄ ̄
  \⊇  /干\| |    _||||||||| |  < さよう、エンドセリン飲んだからな!
    |       |  \ / \_/ /    \_________
   ( /⌒v⌒\_ \____ /
パンパン|     丶/⌒ - - \
    / \    |  |     / |
    /  ノ\__|  |__三_ノ|  |
   /  /パンパン|  |      |  |
  /__/     |  |      |  |
          ⊆ |     | ⊇
494元祖ポスドク:2000/10/25(水) 13:54
柳沢は5流大学教授
しかも詐称
Texas大学Austinを詐称
しかし
その実はTexas大学なんとか(Dallas、TX)というカス大学の主任教授程度のうんこ野郎
わらってやろうぜ
495元祖ポスドク:2000/10/25(水) 13:55
>>441
安井さんは慶応だからまったくの別人だ。もう名前はだすな。失礼だ。
496内科系院生:2000/10/25(水) 13:58
>493
飲んでもきかんだろう
497元祖ポスドク:2000/10/25(水) 14:00
>柳沢の朝鮮野郎

おまえ素人にわかるように説明でけへんのかや?
氏ねや
ぼけ
498元祖ポスドク:2000/10/25(水) 14:01
>>441
おいらは筑波卒だよ
BOKENAS
特定してみそ?
けけけけ
499内科系院生:2000/10/25(水) 14:03
>445
元祖エンドセリン先生が受容体の組織に懐疑的とは、なんだか不安になってきました。
それにしても、日本の大学院教育が技官教育だっていう意見にはもっと不安になりました。
自分で考えるしかないですね。上の先生も基礎や他の教室に行ってこいとアドバイスは
してくれているのですが……。
500名無しさん@お腹いっぱい。:2000/10/25(水) 14:10
>498
筑波大の近くにある女子大の名前は?
メドの図書に一番近いショッピングセンターの名前は?
501元祖ポスドク:2000/10/25(水) 14:11
オマンコ女子大
マンカスショピセン
502ふ〜ん:2000/10/25(水) 14:14
日本でノーベル賞に一番近い?少なくともそれは嘘です。
このスレッドは自作自演ですか?
503結局、1の答え:2000/10/25(水) 14:15
柳沢先生について:
大学院生時代に血管収縮因子、エンドセリンのクローニングを行い
一躍世界の注目の的に。その後、ノーベル賞受賞者、ジョー・ゴールドシュタインに
引き抜かれて、ハワードヒューズ主任研究員(ノーベル賞級の研究者のみに与えられる名誉ある地位)
としてテキサス大学サウスウエスタンメディカルセンター
(全米トップレベルの医学研究・教育機関)で研究室を開く。
その間、東大医学部第二生理学の教授に最年少として選ばれるが、辞退する。
最近は、睡眠をつかさどる因子、オレキシンの研究でも大成功を収め、
今や日本を代表する医学研究者である。
ノーベル医学・生理学賞の受賞が大いに期待される。


504名無しさん@お腹いっぱい。:2000/10/25(水) 14:16
>501
「大角豆の交差点」←読める?
505お前らひがむな:2000/10/25(水) 14:17
折角、聞きたいことがあったのに、来なくなったらどうするんだよ。
506名無しさん@お腹いっぱい。:2000/10/25(水) 14:20
>502
全部レスよめ。
それに関するMY先生のコメントがあるだろ。ボケ。
507名無しさん@お腹いっぱい。:2000/10/25(水) 14:22
>501
東大通りをトウダイ通りとかいうなよ。ボケ。
508名無しさん@お腹いっぱい。:2000/10/25(水) 14:24
>507
北大通りもあるぞ。
509名無しさん@お腹いっぱい。:2000/10/25(水) 14:26
一の矢で飲みましょう。
510名無しさん@お腹いっぱい。:2000/10/25(水) 14:27
>501
宮ランチしってっか?ボケ。
511名無しさん@お腹いっぱい。:2000/10/25(水) 14:29
>501
パチンコ大学に入学しろ
512元祖ポスドク:2000/10/25(水) 14:40
>おまえら
なめんなよ

「ちょっとむかついた。
今夜はソーダが妙に胃にしみるな
バックスのラテはうまいね。」

(オチケン名言集より引用)
513元祖ポスドク:2000/10/25(水) 14:43
つーか
ソーダが胃にしみるようなら、病気の疑いがあるな
514元祖ポスドク:2000/10/25(水) 14:54
Q
515元祖ポスドク:2000/10/25(水) 14:55
yanagisawaは躁鬱病らしい。ここ100件以内にそうカキコしていた
もっと煽って研究の邪魔してやろーぜ
516元祖ポスドク:2000/10/25(水) 14:55
おいらはミーンミーン蝉
517名無しさん@お腹いっぱい。:2000/10/25(水) 15:32
エンドセリンやオレキシンは製薬会社が絡んで来ているそうですが、特許の方はどうなってるのですか?
518名無しさん@お腹いっぱい。:2000/10/25(水) 15:37
エンドセリン、およびオレキシンをクローニングするに至った動機(経緯)を差し支えなければ教えていただければと思います。
519元祖ポス毒:2000/10/25(水) 15:38
元祖ポス毒
ばかみたいなので、消えてください。
私たちは柳沢先生と話したいのです。
あなたは不要です。抹消!ぴーーーーーーーー
520名無しさん@お腹いっぱい。:2000/10/25(水) 15:50
私の現在のボスもHHMIのInvestigatorで、その道の大物ですが、あまり論文を読みません。自分の仕事にかなり関係の深い、ライバルの出したCellなどの論文ですら読んでないこともあり、他人の論文を読む事をそれほど重要視していないようです。逆に、日本にいたときのボスは、ものすごく論文を読んでいて、分野外の事でも主要な論文の内容は殆ど知っていました。先生はどちらのタイプでしょうか?日ごろから文献を読んで勉強する事に関して、どのようにお考えでしょうか?
521名無しさん@お腹いっぱい。:2000/10/25(水) 16:11
age
522520:2000/10/25(水) 16:13
お返事がいただけるまで固唾をのんで何度でもリロードしてお待ちします。
523本物の520:2000/10/25(水) 16:15
522は偽者です。どっちでもいいが・・・。
524名無しさん@お腹いっぱい。:2000/10/25(水) 16:15
もう、寝てんじゃないの?
525潜伏中:2000/10/25(水) 16:23
>>467

He confessed Fuckin KOREAN Ynagisawa has mental disorder.
526潜伏中:2000/10/25(水) 16:24
uckin KOREAN Ynagisawa confessed he has mental disorder.
527名無しさん@お腹いっぱい。:2000/10/25(水) 16:26
>525
いいかげんにしろよ、マイケル!!
またアクセス出来ない板が増えるだけだぞ。
わからないのか?
528YM:2000/10/25(水) 16:26
Fuck you
529Yanagisawa--&gt;insane:2000/10/25(水) 16:27
気分の波は結構大きい方なので(外には必ずしも出さないが)時に苦しくなります。自分では、
完全に躁鬱気質だと思います。
530jons hopkins university:2000/10/25(水) 16:41
shut up!!!!
531JHU:2000/10/25(水) 16:41
>Yanagi

please shut the fuck up
532JHU:2000/10/25(水) 17:23
yanagi fuck you
533JHU:2000/10/25(水) 17:30
i wamma eat white girls shit
by otita kenkyuusha
534Weibel-Palade命:2000/10/25(水) 17:56
>499(内科系院生さんへ)

前述の論文は足掛け4年以上掛けたものだそうです。
ま、それでIFが1.2は確かにトホホですが、
形態屋として再現性には自信があるとも聞いています。

悲観する前に「切って切って染めまくる」で頑張りましょう!
形態学的研究はいろいろな意味で効率が悪いのは事実ですが
「美しい」データが得られたときの醍醐味もあります。
きっと「美しい」データが貴方を待っていますよ。
535JHU:2000/10/25(水) 20:03
http://cocoa.2ch.net/test/read.cgi?bbs=hosp&key=972471119
This might affect our nation's existence.
536JHU:2000/10/25(水) 20:04
That will ceratinly influence korean's attitude toward our nation.
537JHU:2000/10/25(水) 20:05
The existence of Yanagisawa just means KOKUTI of our nation
538JHU:2000/10/25(水) 20:06
national shame
539JHU:2000/10/25(水) 20:07
dont let Yanagisawa enter our nation.
540JHU:2000/10/25(水) 20:07
The race of Yanagisawa is inferior to us.
541名無しさん@お腹いっぱい。:2000/10/25(水) 20:07
基礎系いって、芽が出なかった人って、就職どうすんの?
臨床に戻ったって話は聞かないし。
大学教官だって当然無理だし、企業の研究員といっても
売り込めんでしょ。
臨床医からの質問です。
542JHU:2000/10/25(水) 20:08
Wake up Yanagisawa it passed 6AM
543名無しさん@お腹いっぱい。:2000/10/25(水) 20:20
>541
就職のこと気にするようなひとは、基礎ではmotivationを
保てないでしょう。
まあ、柳沢先生のようなスーパー成功者は例外的ですが、
みんなそこそこ生活してるけどね。
やっぱり、柳沢先生のような研究者がもっと日本からでれば
基礎系の魅力がもっと浸透するかとも思うけど。
544JHU:2000/10/25(水) 20:21
柳沢先生のような研究者がもっと日本からでれば

oh yeah
He is not a pure japanese.
he must do so
545巨人優勝:2000/10/25(水) 20:25
>motivationを 保てない
そして、542みたいなキチガイを生むのか・・・・
546MY:2000/10/25(水) 20:27
>JHU
Yes I am a Korean japanese.
I am proud of it.
547名無しさん@お腹いっぱい。:2000/10/25(水) 20:31
自作自演か。むなしい。
548>JHU:2000/10/25(水) 20:34
だから何なんだよ。日本の環境で育って、世界のために
活躍している先生だろ。日本人&韓国人でいいじゃねえかよ。
在日の同級生なんてそこら辺にいるし、出世早いヤツ
多いだろ。他人の能力は素直に認めろよ。
549>518:2000/10/25(水) 20:36
me too.
先を越されましたか。
私もエンドセリンを見つけるに至った経緯をぜひ訊きたいです。
エンドセリンは我々臨床医(臨床の循環器)の片手間の研究の場ですら、一時は話題の頂点でした。
こんなところで発見者にお会いできるとは。2チャンも捨てたもんではないですね。
もうひとつ、スミスク**ビー**社のorexin antagonistの臨床応用薬の実用性について。
先生のpaperにもありましたが、食欲を制御する複雑なsystemの一端を担う物質としてのorexinですから、
肥満やエネルギー過剰摂取のいわば先進国の食行動のゆがみを是正する”画期的な医薬品”にはなりえない、
つまり、
いくら素晴らしい薬剤が出現しようとも、個体としての人間が自己の欲望を制御し平安を得られるという結果にはなりえないという永遠の命題が
を感じざるを得ませんが、う〜ん、科学は進む、それでも地球は回っているんでしょうね。
軽率かも知れませんが、一度先生とギネスを一杯やりたいもんです。
550名無しさん@お腹いっぱい。:2000/10/25(水) 20:39
ここで本人の名前を出して誹謗中傷している人はもう犯罪の域に達しています。
もうかなりの事を言っています。
このスレッドが警察の目に止まれば、話題性もあるスレッドなので一発です。
このスレッドが管理人の手を離れ司法に委ねられたら、皮肉ですがY先生より有名になります。
551名無しさん@お腹いっぱい。:2000/10/25(水) 20:41
>MYセンセ
非常に撫しつけな質問で誠に恐縮ですが、あえて。
ずばり、HHMIのサラリーはいくらですか?
日本みたいに定期昇給あるんですか?
歩合給ってあるんですか?
退職金ってあるんですか?
副収入ってあるんですか?
日本とくらべ税金高いですか?
繰り返し、恐縮です。
552>551:2000/10/25(水) 20:50
参考までに、私
33歳、国立大学助手 勤続2年
教育職(一)2級 12号俸
で、手当てこみ手取り32万程度ですが。
(当直なし、扶養手当てあり、住宅手当てなし)
553名無しさん@お腹いっぱい。:2000/10/25(水) 20:59
>552
日本の大学教官の安さは異常です。
HHMIはしらんが、日本よりかなーり良いことは間違いない。
554MY:2000/10/25(水) 21:11
>>469 Teaching duties

ウチの大学(UT Southwestern)は、graduate-only campus(medical schoolとgraduate schoolのみ)ですので、undergraduate studentとの接触は、インターン(いわゆるsummer student intern)としてラボに廻ってくる学生以外、全くありません。

現在、俺はmedical schoolのクラスは一切教えていません(見返りがほとんどゼロなので、断りました)。Graduate school(Ph.D.コースおよびMSTPのPh.D. phase)では、現在年間7〜8コマのクラスを受け持っています。その他に、毎週1回、Integrative Biology Graduate Programというコースの論文抄読会を受け持っております。教官の数が学生数の数倍以上ですから、教官ひとり当たりの負担は、この程度です。その他に、qualifying examの審査や、博士論文審査はしょっちゅうやらされます。

大学院の授業のやり方は、これまでいろいろ自分なりに試行錯誤してきましたが、結局、一番学生のウケが良くて、自分自身も楽なやり方は、徹底的に「学生参加型」のクラスにすることだと気づきました。例えば「G蛋白供役受容体」に関する2時間のコマがあったとすると、最初の40分間くらい、GPCRに関する基礎知識を「講義」するのですが、それも、出来るだけ学生に質問を廻す(一人一人順番にどんどん廻す)形で、学生が自分たちで自分たちに講義しているような形にしてしまうのです。で、コマの残りは、GPCRについて予め選んでおいた最近の良い雑誌に出た論文を、3〜4報、抄読会形式で発表させます。学生をサブグループに分けて、ひとグループ1報担当させるのです。俺は、ゆっくりと腰掛けて彼らの議論を聞いているだけです(うひひひ)。もちろん、実際にはかなり口を挟みますが。

結局の所、このような学生参加型授業のほうが、学生の評価も高い、というのが教官皆さんの一致した意見です(そうです。アメリカの大学院は、学生が教官を逆評価する機会があるのだ)。

よく言われる、「medical school では『知識』を教えなくてはいけないが、graduate schoolでは、『ストーリー』を教えろ!」というのは、本当であると実感しています。

英語に関しては、この業界には外国人PIも学生もウヨウヨいるわけで、訛のある英語には皆さん非常に寛容です。Fluencyは大切ですが。英語そのものに関して学生に苦情を言われたことはありません。むしろ、お陰様でとても明快なクラスであったという評価をもらうことが多いです。俺自身も、アメリカに来てから(日本以外の)訛のある英語を聞き取る能力が増しました。今では、その人の訛から大体お国を言い当てることが出来るまでになりました。
555名無しさん@お腹いっぱい。:2000/10/25(水) 21:32
>554先生
あの、サラリーの件はやぱっりお答えできませんか?
556名無しさん@お腹いっぱい。:2000/10/25(水) 21:37
>>555 ビンゴ
ボケ茄子。質問が腐るほど溜まってるんだ。もう少し待て。
557名無しさん@お腹いっぱい。:2000/10/25(水) 21:41
>>553
17年前にWashington Universityのassociate professorから東大へ教授として
赴任した細胞生物学の先生の場合、サラリーはほぼ1/2になったという。
当時の円$レートなどはわからないが、アメリカでの生活物価の安さを考えると
この先生の場合だとかなりの賃下げで米国のサラリーの30-40%になったという
感覚ではないだろうか(本人でないので想像)。また日本の大学の教員の講義や
実習、試験監督などの業務を考えると実力がある人なら多くはアメリカにとどまって
研究を続けたいと感じるだろう。問題は、英語とグラントの獲得のみであろう。
558名無しさん@お腹いっぱい。:2000/10/25(水) 22:05
>>555
(日本語)サラリーの件は、やはり お答えいただくのは 無理でしょうか。
559内科系院生:2000/10/25(水) 22:27
>534
ありがとうございます。
がんばって切片切りします!教えて頂いた論文といくつかの会社の抗体でやってみます。
先生は、やはり凍結切片が一番抗原性の維持がよくて免疫組織には良いとお考えですか?
560名無しさん@お腹いっぱい。:2000/10/25(水) 22:29
>546

おまえ、文法が2カ所違ってるぜ。人のHNを使うんだったら、もっとまともな英語かけ。
561内科系院生:2000/10/25(水) 22:33
このスレ、柳沢先生が時々書き込んでいただければ、
ネオむぎスレ以上にのびるかもしれません
(内容が全然違うので比較してはいけないですが)
反響大きさは論文のインパクトだけじゃないですね。
562名無しさん@お腹いっぱい。:2000/10/25(水) 22:43
師はただいまお仕事中ですので、しばらく待ちましょう。
563そうか、時差がねえ:2000/10/25(水) 23:01
あの柳沢先生とチャットができるって聞いて
来きたんだけど、残念。
564医学の徒:2000/10/25(水) 23:17
MY先生、はじめまして。
このような場ですが、先生の気さくな人柄を知ることができ、喜ばしく思っております。
私はとても自分のことを明かす勇気はないのですが。
ところで、私の知人のK医師が、MY先生のところに留学して、青い目の女医さんを連れて
かえってきました。結婚して、アメリカで暮らしているようです。けっこうお似合いの
夫婦でした。MY先生のラボが縁とのことです。
こういうことは多分あまりないと思うのですが、これから留学して留学先でパートナーを
見つけようとする人もいると思いますので、アドバイスを宜しくお願いします。
565このスレは:2000/10/26(木) 01:06
ageとくね&hearts;
566>電波妨害センセ:2000/10/26(木) 01:15
これはブラウズ環境に依存のものなので余りお使いにならないよう>&hearts;
567電波妨害:2000/10/26(木) 01:19
>566 ごめん。てへっ
568田舎脳外科:2000/10/26(木) 01:23
>566
ど、どうして565みて電波妨害女史ってわかるんだ?
おれはまだまだ厨房か、、、
569電波妨害:2000/10/26(木) 01:29
>568
いや、違うんだけど・・・なんとなく謝っちゃった。
柳沢先生オンしてるけど忙しいみたいよ。
570MY:2000/10/26(木) 01:46
>>480 HHMI Investigatorの「所属」

人によって、HHMIを所属の最初に持ってくる人と、後に持ってくる人が居ますよ。俺はたいてい最初に持ってきていますが、この順序には特別にルールはないはずです。

ただ、HHMIが単にその人のgranting sourceではなく「所属」になることには、れっきとした理由があります。つまり、たとえばNIHとNIH grantをもらっているPIの関係とは異なり、HHMIとそのInvestigatorは、真に雇用関係にあるのです。俺の給料は全額HHMIから出ており、他のソースから給料を補充することは税法上できません。ちょっとした、裏話でした。
571564:2000/10/26(木) 01:49
>>564 の質問ですが、お答えがいただけるまで固唾をのんで何度でもリロードして
お待ちしております。もちろん、先生も御将来のあるお方です。
私の質問に答えていただくことで将来のグラントプロポーザル、他大学への移動
に影を落とすようではいけません。ですが私も金髪女性を是非ともモノにしたいと
存じます。何かコメントを頂ければと存じます。
572名無しさん@お腹いっぱい。:2000/10/26(木) 01:51
いや、マジ本物とは。びっくり。
やっぱ、ナルコの今後の展開が気になります。
睡眠も含めた脳波の分子メカニズムっていう流れになっていくのでしょうか?
今の脳の高次機能をやっている主流の人からは、無視されてい雰囲気があるんで。
けど、意外に20年ぐらい前の人は、睡眠関係をみんな一度は通過していたりします。
これは、脳波の生成のメカニズムが分からないっていうことに起因していると、
感じました。ナルコの仕事で、この辺が分子の問題でごちょごちょしていく
ような気がするのですが。お考えを聞かせていただけたら・・・
573名無しさん@お腹いっぱい。:2000/10/26(木) 01:51
528 名前:本物のMY 投稿日:2000/10/23(月) 11:54
>126 あ、これ良い質問!

もちろん、個々のPIのスタイルによって「良いポスドク」の条件はいろいろ違うと思います。だから、これはあくまでも俺自身の基準です。ハードワーキングとか、勉強家(文献に通じている)とか、実験センスがいいとか、当然の事を別として一つだけ挙げれば、やっぱり、「PIを放っておかない」じゃないかなぁ。PIは(少なくとも俺は)寂しがりやです。毎日のように、うるさいくらいに、あーだこーだと議論を吹っかけてくるタイプが俺は好きですねぇ。あと、PIが何か大事なことをsuggestionしたとき、黙って何となく無視されてしまうタイプは、そのうち逆にPIに無視されるようになります。反対意見があるならあったで、ハッキリ言って欲しいですね。


574名無しさん@お腹いっぱい。:2000/10/26(木) 01:58
あと、せんせのノックアウトの仕事と犬の仕事、
いったい、どっちが先だったのか、聞きたい。
最初、Cellの表紙に圧倒されただけど、いろいろ調べたら、
orphan recepterからのストーリがあっって、こちらのに、
すっごく感動したです。
575電波妨害:2000/10/26(木) 02:06
>574 発表は同時
576名無しさん@お腹いっぱい。:2000/10/26(木) 02:09
>575
それはオフィシャルにはそうだけど、実際には発表前に
データが飛び交ってたはず、皆はその裏話やゴシップを
聞きたいのさ。
577MY :2000/10/26(木) 02:27
>>485 論文引用件数

良いこと言われますね、まったくその通りだと思います。評価基準としての引用件数の最大の欠点は、数字がその分野の人口をダイレクトに反映してしまう事です。俺の1988論文の引用件数が多いのも、心血管系の生物学、はたまたペプチド性ファクター一般に携わる人の数がすごく多いからです。雑誌のインパクト・ファクターについても、同様の批判が出来ると思います。ISIなどでは、この"population effect"を修正したいろいろな適量的基準を作ろうとしているみたいですが、なかなか難しいらしいですね。

アメリカの大学でPIをリクルートする際には、論文総数とか、インパクト・ファクター総数や平均値とか、そんな数字そのものは、ほとんど評価基準になりません。まぁたしかに、最低限度の論文総数や、トップ雑誌に最低1,2報は出している、とかの「足切り」基準はクリアしていないと始まらないかも知れませんが。ほんとうの業績評価は、数字ではなかなか表すことの出来ない、その人のストーリー性とか発見のインパクトとかに基づいてなされるのです。

大学の序列についても、同じ意見ですね。Harvardだから手放しで一番なのではなく、Harvardの何先生に教わりたいとか、Stanfordのこのdepartmentは最高とか、そういう物言いになりますよね。俺自身も、UT Southwesternの、Pharmacology (Gilman) + Molecular Genetics (Brown&Goldstein) のフロアは、全米でも最高だと思っていますよ。
578MY:2000/10/26(木) 02:28
>>487

よかったよかった!! やっぱ、「自力で」がキーワードですよね。
579MY:2000/10/26(木) 02:28
>>491 体力というより、単なるアホ

本当です(爆) しかしどーしてそんなこと知ってんの?
580MY:2000/10/26(木) 02:29
>>499

必要以上に懐疑的になることはありませんが、正直いって、in situ hybによるサポート無しのGPCR immunohisto(エンドセリンに限らず)は、信じないことにしています。

大学院教育については、少なくともその「不安」を自覚している人は大丈夫です。
581MY:2000/10/26(木) 02:52
>>485 >>487 >>491 >>499 の方々..

すんません。不注意で、話がわからない人のほうへ回答書いてしまったようです。そちらの44からご参照くださいな。m(_ _)m
582MY:2000/10/26(木) 03:21
>>564 >>571 YK君の友人へ

あの、水を差すようですが、アドバイスって一体何をすれば?彼女の見つけ方とか結婚までの持って行き方なんて、んなもん知るかよっ。ここのいっぱいある「どうすれば医者と結婚できますか?」的なスレッドを熟読してはいかがでしょうか。金髪女性を是非ともモノにしたいと存じます、って言われても..それって本末転倒なのでは? YK君だって、たまたま俺のラボで出合って惚れた女が金髪碧眼の女医さんだったわけで。

ちなみに、この女医さん(元うちのM.D.ポスドク)も、無事ウチの大学で小児科のassistant professorになり、しかも初めて出したNIH RO1が、一発で何と7パーセンタイルの高得点を獲得しfundされました。I am proud of her!

さてと、YK君の論文を仕上げなくては...(^^ゞ
583MY:2000/10/26(木) 03:30
>>582

あ、ちなみにこのYK君っていうのは、東大医学部を卒業して、半年だけ内科研修やって医局にも入らず、院にも行かず、いきなり俺のラボにM.D.ポスドクとして来てしまったという大変オモロイ奴です。今ほぼアクセプトされつつある論文の他に、結構でっかい仕事を2つほど完成させつつあります。金髪碧眼の女医さんとの結婚では、最初反対していたご両親を、勘当をも恐れず、かつ粘り勝ちで説得した奴です。I am pround of him@` too!
584MY:2000/10/26(木) 03:34
>>486 >>517 >>518 >>520 >>549

皆さん、良い質問されますね。しかしいずれも重い質問なので、もっと時間のある時に..
気長にお願いしますね。
585MY:2000/10/26(木) 04:07
551>> 給料のこと、別に不躾な質問とは思いませんよ。

HHMIの給料は、原則としてhost institutionでその人がもらうべき給料と同じです。しかし、fringe benefits packageが大学のものより圧倒的に良いので、実質的には同格のnon-HHMI PIよりも数割高い給料になっていると思います。そうですね、やはり日本の(国立)大学の同格教官と比べたら、アメリカの大学のPIは2倍以上はもらっていると思います。実際、多くのばあい、ポスドクからPIになると、給料が一気に2〜3倍跳ね上がります。日本で、科技庁グラントなどで高給ポスドクをしていて、大学教官になると給料が実際に減ることがあるのとは随分違います。

定期昇給はウチの大学は原則としてありません。しかし、きちんとやっていればメリット昇給が来ますので、俺の場合はだいたい今まで毎年、5〜10%程度ずつ上がってきました。歩合給(例えば、論文一報いくらとか?)のようなものは、一切ありません。退職金はもちろんあります。Pre-taxのfringe benefit moneyから、自動積み立てし、自分の好みのマーケットに投資するようになっており、年々増えて行きます。副収入としては、もちろんセミナー・講演などの謝金の他に、製薬会社のコンサルタント料や、パテントが売れた時のロイヤリティーなどがあります。(最近までの?)日本のどこかの医局の教授たちみたいに、部下の仲人料で儲ける、なんてのは一切あり得ません。所得税は、州や地域によっても総額が違ってきますが、日本よりはやや高いかも知れませんね。最近の日本のことを知りませんので、正確には判りません。
586名無しさん@お腹いっぱい。:2000/10/26(木) 04:16
日本の場合、45歳の教授では、早慶クラスで1000万円、
東大クラスで大体800万円位です。

ということはアメリカンのPIは、15万ドル位貰っているのでしょうか?
凄い!
ポスドクの給料は大体アメリカでは2.3-4.0万くらいですが、
日本では最近350-550万くらいです。
587名無しさん@お腹いっぱい。:2000/10/26(木) 04:22



588だめドクター:2000/10/26(木) 04:48
>>586
なんと!!早慶クラスではそんなにもらっているのですね。
東大で800万円ですか。国立大学独立法人化したらどうなるのでしょうね。
日本の場合、建前社会で阿吽の呼吸で水面下で色々決まってしまうからますます
変な感じになりそうな気がします。まー、なってみらなきゃ分からないですが
日本で若手と呼ばれる30-40歳の人に有志はいるのか?
博士までとって、ある年齢までいってもう引き返せないから上に媚びて出世を目論見、
上の人間はそういう人たちのいいなり。そういうラボがいっぱいありそう。
上を見ていると日々の生活が不安です。立ち上がれ!!もっと若い人よ!!
やっぱ、薬剤師のバイトやって金ためて国外脱出するしかないかな。
あと、医者から見ると薬剤師および薬学部ってどうなんでしょうね?地位向上断固反対?
ちょっと、スレが違いますか。
医者じゃなくてすみまそん。
589堕ちた研究者:2000/10/26(木) 05:03
>>588

こんにちは。俺と同業のようだね・・
簡単に自己紹介させてもらうとこんな感じだけど>>382
意外にどっかで会ってるかも・・
日本だとMDが幅をきかせてるんで、医学部で研究してる
ノンMDの日常は悲惨だよ・・
俺もここに書き込んで薬学博士を名乗っただけで出ていけコールを
喰らった。
これも何かの縁、海外留学を考えているのなら少しはアドバイスできるかもね。いちおう東と西の情報なら提供できると思う・・
ここだとなんだから、海外生活板に書き込んでくれたら出来るだけの
返事はする・・
海外生活板でオチケンと言えば、有名だよ。
ここでも無理矢理有名にされたけどね・・(笑
590堕ちた研究者:2000/10/26(木) 06:19
suck my dick
>nigger bitches
591堕ちた研究者:2000/10/26(木) 06:19
then i suck your fuckin tits
592堕ちた研究者:2000/10/26(木) 06:23
http://cocoa.2ch.net/test/read.cgi?bbs=hosp&key=972276419
I m this motherfuckin' idiot.
be nice to me
593堕ちた研究者:2000/10/26(木) 06:28
大学院の授業のやり方は、これまでいろいろ自分なりに試行錯誤してきましたが、結局、一番学
生のウケが良くて、自分自身も楽なやり方は、徹底的に「学生参加型」のクラスにすることだと
気づきました。例えば「G蛋白供役受容体」に関する2時間のコマがあったとすると、最初の4
0分間くらい、GPCRに関する基礎知識を「講義」するのですが、それも、出来るだけ学生に質問
を廻す(一人一人順番にどんどん廻す)形で、学生が自分たちで自分たちに講義しているような
形にしてしまうのです。で、コマの残りは、GPCRについて予め選んでおいた最近の良い雑誌に出
た論文を、3〜4報、抄読会形式で発表させます。学生をサブグループに分けて、ひとグループ
1報担当させるのです。俺は、ゆっくりと腰掛けて彼らの議論を聞いているだけです(うひひ
ひ)。もちろん、実際にはかなり口を挟みますが。


I am going to try to use this strategy in my presentation next time.
Thanks
594堕ちた研究者:2000/10/26(木) 06:29
Yanagisawa came to say more practical thimgs.
Thats nice
595堕ちた研究者:2000/10/26(木) 06:39
596教官の逆評価は日本にも必要ですか?:2000/10/26(木) 06:42
>そうです。アメリカの大学院は、学生が教官を逆評価する機会があるのだ

日本の大学の教官には、研究を全くしていない人、指導能力に欠けた人、
恐喝まがいの方法で業者から試薬をくすねている人・・・・、さまざまな
不適格者がいますが彼らを追い出す方法はありません。そして、
運悪くそれらの不適格者が主宰する教室を選んでしまった大学院生は一旦退学するか
他教室に移るしかありません。日本の研究環境の向上のためにも、
教官を逆評価するシステムが必要だと思うのですが、先生はどうお考えですか?
時間に余裕があればご教示願います。
597堕ちた研究者:2000/10/26(木) 07:39
hage
598堕ちた研究者:2000/10/26(木) 07:40
>yana

How many niggers are there?
599名無しさん@お腹いっぱい。:2000/10/26(木) 09:27
いかくさいのが、わしのティムポじゃ。
600名無しさん@お腹いっぱい。:2000/10/26(木) 09:44
600番ゲット
601名無しさん@お腹いっぱい。:2000/10/26(木) 10:24
ティムポ
602名無しさん@お腹いっぱい。:2000/10/26(木) 10:25
ティムポ
603名無しさん@お腹いっぱい。:2000/10/26(木) 10:38
551です。亭々なお答えありがとうございます。
>やはり日本の(国立)大学の同格教官と比べたら、アメリカの大学のPIは
>2倍以上はもらっている

その他いろいろ含めると、日本の比じゃありませんね。
やはり、ここに出てくる先生のお話を総合的にみると、
すべての面でアメリカの研究環境は優れてるわけですね。
日本に帰れないのがよくわかります。
今後も日本の頭脳流出は続きそうですね。
まあ、流出したからなんなんだという意見もあるでしょうが、
そういう環境を放置してる日本の役人ってなんなんだろうと思ってしいます。
まだ高度成長期の幻影をみてるのだろうか。

>MY先生
あえて、日本のほうが良かったことってありますか?
604名無しさん@お腹いっぱい。:2000/10/26(木) 10:41
先生、ぼくは一日中2chに没頭して、研究に身が入りません。どうしたらいいでしょうか?
605名無しさん@お腹いっぱい。:2000/10/26(木) 10:49
MY先生がまちがえて他のスレにレスったコメントです。

>>485 論文引用件数
良いこと言われますね、まったくその通りだと思います。評価基準としての引用件数の最大の欠点は、数字がその分野の人口をダイレクトに反映してしまう事です。俺の1988論文の引用件数が多いのも、心血管系の生物学、はたまたペプチド性ファクター一般に携わる人の数がすごく多いからです。雑誌のインパクト・ファクターについても、同様の批判が出来ると思います。ISIなどでは、この"population effect"を修正したいろいろな適量的基準を作ろうとしているみたいですが、なかなか難しいらしいですね。
アメリカの大学でPIをリクルートする際には、論文総数とか、インパクト・ファクター総数や平均値とか、そんな数字そのものは、ほとんど評価基準になりません。まぁたしかに、最低限度の論文総数や、トップ雑誌に最低1,2報は出している、とかの「足切り」基準はクリアしていないと始まらないかも知れませんが。ほんとうの業績評価は、数字ではなかなか表すことの出来ない、その人のストーリー性とか発見のインパクトとかに基づいてなされるのです。
大学の序列についても、同じ意見ですね。Harvardだから手放しで一番なのではなく、Harvardの何先生に教わりたいとか、Stanfordのこのdepartmentは最高とか、そういう物言いになりますよね。俺自身も、UT Southwesternの、Pharmacology (Gilman) + Molecular Genetics (Brown&Goldstein) のフロアは、全米でも最高だと思っていますよ。


45 名前: MY 投稿日: 2000/10/26(木) 02:15

>>487
よかったよかった!! やっぱ、「自力で」がキーワードですよね。


46 名前: MY 投稿日: 2000/10/26(木) 02:17

>>491 体力というより、単なるアホ
本当です(爆) しかしどーしてそんなこと知ってんの?


47 名前: MY 投稿日: 2000/10/26(木) 02:23

>>499
必要以上に懐疑的になることはありませんが、正直いって、in situ hybによるサポート無しのGPCR immunohisto(エンドセリンに限らず)は、信じないことにしています。
大学院教育については、少なくともその「不安」を自覚している人は大丈夫です。
606名無しさん@お腹いっぱい。:2000/10/26(木) 11:07
>603
先行投資っていう概念が日本(特に役人)には希薄すぎる。
とくに長期的なね。
さらに、そういう考えがあってもできない組織構造になってるね。
607名無しさん@お腹いっぱい。:2000/10/26(木) 11:18
>583
>今ほぼアクセプトされつつある論文の他に、結構でっかい仕事を2つほど完成させつつあります。

おれもめちゃくちゃテキサスへ行きたくなってきた。
608名無しさん@お腹いっぱい。:2000/10/26(木) 11:22
次のポスドク先を真剣に再考中・・
609名無しさん@お腹いっぱい。:2000/10/26(木) 11:28
>583
いいなー。(涎
610学位取得見込:2000/10/26(木) 11:28
研究から足を洗うことも考えてたんだけど
このスレ読んでたら,もうちょっとがんばりたい気がしてくるなー。
611名無しさん@お腹いっぱい。:2000/10/26(木) 11:53
MY先生、ブッシュテ・キサス州知事をどう思いますか?
612名無しさん@お腹いっぱい。:2000/10/26(木) 11:54
ゴア副大統領をどう思いますか?
6132chの住人:2000/10/26(木) 12:29
2chから足を洗うことも考えてたんだけど
このスレ読んでたら,もうちょっとがんばりたい気がしてくるなー。
614医学の徒:2000/10/26(木) 12:30
564の医学の徒です。
MY先生、お返事ありがとうございます。YK君も元気で活躍しているとのこと、
喜ばしく思います。
ちなみに571は私ではありません。私はちょっとした世間話のつもりだったのに
「固唾を飲んで」だなんて無粋なことを言わないでほしい。まあ、ネット上ですから
こういうこともあるんでしょう。
615名無しさん@お腹いっぱい。:2000/10/26(木) 13:00
好きなテレビ番組は何ですか?
616名無しさん@お腹いっぱい。:2000/10/26(木) 13:10
結婚されてますか?ご家族は?
お休みの日はどう過ごされますか?
617一庶民:2000/10/26(木) 13:12
柳沢先生、はじめまして。

もう、チャットにはウンザリなさっておられる頃でしょうか?
読んで頂けるといいのですが…

 いつもはyahooの掲示板を楽しんでいる者ですが、
そこで評判を聞きつけてここへ辿り着きました。
こんなことを申し上げると皆さんの袋叩きに遭いそうですが、
先生にはここから撤退して頂きたいと思います。
先生のここへの存在は、
多くの人に希望や夢を与えると同時に同じくらい多くの人の心を傷つける。
私の想いを寄せる人は、違う分野の研究者なのですが、心穏やかでないようです。
頭では己の分をわきまえて頑張れば良いと理解は出来ても、
心の問題は一筋縄ではいきません。

 先生は幼い頃から今日に至るまで、陰で日向で有り余るほどの称賛を浴びてこられたはず。
先生の罪ではないのですが、これは高い所得税だと思ってください。
眩しすぎるが故の、無垢なるが故の残酷さに気付いて頂けたら、
私は先生を同じ日本人として心から誇りに思います。
失礼な事を申し上げてご免なさい。

PS
 でも、いろいろと参考になるお話し、ありました。
出来たら、ご自分のHPを作って、その中でチャットなさっては如何でしょう?
どうしてもここを楽しみたいなら、身分を伏せて匿名で、ネ…
618名無しさん@お腹いっぱい。:2000/10/26(木) 13:13
>616
「既婚です。」という本人のレスがある(>>85)。奥さんも同業者。娘さんがいる(結構美人)。
619名無しさん@お腹いっぱい。:2000/10/26(木) 13:17
匿名じゃないから、マジに感動してんのに!!
ここでレスもらってから、迷いが消えたっていうか・・
すごく励まされて研究の進みもいいんだけどな。
620名無しさん@お腹いっぱい。:2000/10/26(木) 13:19
>617
ここはみんなが学術的な内容を含めて、先生と情報交換している貴重な場です。これくらい高名な先生にいろいろ質問することは、他の場では不可能です。ぜひともこれからも続けていただきたいと思います。あなた個人の勝手な意見で、みんなの語らいの場を潰すのはやめてください。
621名無しさん@お腹いっぱい。:2000/10/26(木) 13:20
柳沢先生、これに懲りずに、どうか書き込みを続けてください。よろしくお願いします。
622元祖ポスドク:2000/10/26(木) 13:23
おれは筑波の後輩だちょ〜ん
柳沢が大嫌い。
JHUのポス毒しゃ〜
623名無しさん@お腹いっぱい。:2000/10/26(木) 13:27
>多くの人に希望や夢を与えると同時に同じくらい多くの人の心を傷つける。
私の想いを寄せる人は、違う分野の研究者なのですが、心穏やかでないようです。

ようは>>617の想いをよせてる人が、柳沢先生に嫉妬して心穏やかでないということですね。
ここに書き込まないで617がその心を鎮めてあげなさい。
自分たちの問題は自分たちで解決するように。
624名無しさん@お腹いっぱい。:2000/10/26(木) 13:28
アンフェアーな巣窟、匿名掲示板「2ch」に、敢えて本名を名乗って現れた柳沢先生のフェアープレーに本気で感動しています。これこそ、まさに尊敬できる姿だと思います。これからも、ぜひ書き込みを続けてください。
625名無しさん@お腹いっぱい。:2000/10/26(木) 13:30
先生はこの時間寝てます。
現在ダラスは23時半過ぎです。
626名無しさん@お腹いっぱい。:2000/10/26(木) 13:32
先生、ここに書き込むの楽しいって自分で言ってたじゃん。
レスも丁寧につけてるし、荒らしは完全無視してるし、
>どうしてもここを楽しみたいなら、身分を伏せて匿名で、ネ…
なんて言われてもねぇ・・・
場違いは617に決定!
627名無しさん@お腹いっぱい。:2000/10/26(木) 13:33
>617
あんた、自分の思いを寄せている人が個人的に、他人を妬んでいるからと言って、なんでみんなを巻き込もうとするんだ。その嫉妬心の固まりの研究者を精神病院へ連れていって、薬物療法でもしてろ。
628名無しさん@お腹いっぱい。:2000/10/26(木) 13:34
俺のところは24時過ぎてる。まだ起きてるけど。
先生は早寝なんだね。
629sage:2000/10/26(木) 13:40
>>628
アメリカの研究機関,大学はだいたい朝が早いよ.
8時ぐらいにみんな来てるし,駐車場も結構埋まるよ.
ところで先生は一日何時間ぐらいを研究に費やしてるのでしょうか?
630元祖ポスドク:2000/10/26(木) 13:42
http://cocoa.2ch.net/test/read.cgi?bbs=hosp&key=972534030
柳沢君にぜひよんでもらいたい
631名無しさん@お腹いっぱい。:2000/10/26(木) 13:43
>>629
駐車場・・(笑
チケット貰ってキー預けます。
632元祖ポスドク:2000/10/26(木) 13:45
BY THE WAy、FuckinKorean説はどうなったのか?
立証者はいないのか?
633名無しさん@お腹いっぱい。:2000/10/26(木) 13:47
>617
>いつもはyahooの掲示板を楽しんでいる者ですが、
ならば、お前こそくるんじゃねえ。これだけの人か意見交換を楽しみにしてるっのに
ひょこっときて出てけとはなんなんだおまえ。いってることわかってるのか?
そんな歪んだ嫉妬心しかもてないんだったら、まずお前が消えろ。

634MY:2000/10/26(木) 13:52
では期待にこたえて答えましょう。
はい、私は朝鮮人です。
祖父は奴隷として、祖母は慰安婦として下関につれてこられました。
わたしはいわゆる3世ですので、あまりわかりません。
635MY:2000/10/26(木) 13:53
君は筑波の後輩にあたるんですね?
よく覚えておきましょう。ふふふ
>おちた研究者(元祖ポスドク君)
636元祖ポスドク:2000/10/26(木) 13:55
>柳の院ポ
ついに小隊をあらわしだしたな。
おうよ。
おいらは今はジョンズホプキンス大だぜ。
98に筑波博士卒だ。
よくおぼえとけ。
愛車はカローラだ。
こんどはSFでポス毒だ
BOKE
637名無しさん@お腹いっぱい。:2000/10/26(木) 13:56
だれだよ。先生は荒らしには無視して偉いとかいってたのは?
まんまと策にはまってるじゃないか?
638名無しさん@お腹いっぱい。:2000/10/26(木) 13:57
しかし下等市民の末裔を先生と呼ぶのも気が轢けるな
真実がわかった以上・・・
639名無しさん@お腹いっぱい。:2000/10/26(木) 13:59
>617=634=635
先生に、出て行けと言ったかと思うと、今度は偽者になって、いいかげんにしろ。

こんな屑レスはほっといて、本物先生、よろしくお願いします。
640名無しさん@お腹いっぱい。:2000/10/26(木) 13:59

一連の一人芝居は、そぞろ哀れを誘いますね(ふぅ
剥き出しの劣等感は見ているほうが辛い。
641名無しさん@お腹いっぱい。:2000/10/26(木) 14:05
634−639
こいうみえみえの自作自演って、久しぶりにみたよ。
ほんとのキチガイだな
642名無しさん@お腹いっぱい。:2000/10/26(木) 14:06
641
634−638に訂正。
643MY:2000/10/26(木) 14:10
いえ、ボクですよ。(にやにや)
644名無しさん@お腹いっぱい。:2000/10/26(木) 14:17
ダラスは午前0時過ぎ。
先生は寝てる時間です。今 現れているのはニセモノです。
645名無しさん@お腹いっぱい。:2000/10/26(木) 14:19
634−638
あまりの幼稚な自作自演に、
屈折した日常生活が容易に想像できて目頭が熱くなります。
せいぜい人並みの人間に更正してね。
646>誰だって庶民:2000/10/26(木) 14:32
>>617 一庶民さんへ
気持ちはわかります。同じ領域の研究者の方、あるいは自分の様に、研究職より臨床を選んでずっと
歩いてきて、もう戻れないところでまだ迷いを捨てきれずにいた時に、このようなスレで
柳沢先生のような存在に触れてしまった時の、打ちのめされたような気持ち。
でも、その類の傷つくことって、それは己の現実を知ることであって、心の問題が一筋縄ではいかないからって、
誰が自分を傷つけたっていうのとは根本的に違いませんか?
無垢なるがゆえの残酷さっていう表現、わかるけど、見当違いだと言いたい。そのくらい自分達で乗り越えて生きていきましょうよ。
なんだかモーツァルトとサリエリの話みたいですが。才能に恵まれた成功者と周りから見える人だって
そういう瞬間が人生にいくつもありますよきっと。
名のろうと名のるまいと、身分も比較も超越して互いに会話できる場って貴重だと思います。
647名無しさん@お腹いっぱい。:2000/10/26(木) 14:32
617が思いを寄せている人は、今偽者として暴れまわってる奴じゃないか?(藁
648名無しさん@お腹いっぱい。:2000/10/26(木) 14:53
>647
それが事実だったらそれはそれですごい話だ。
ドキュンな彼氏を救うためにYahooからわざわざやってきたなんて。
649MY:2000/10/26(木) 15:14
>>603 日本vsアメリカ?

俺はアメリカの環境が好きで敢えてそれを選んできた人間ですから、俺の書き方はどうしてもアメリカびいきになりますよ。あくまでも、そういうバイアスのかかった意見としてお読み下さい。おっしゃるように(日本からアメリカへの)「頭脳流出」っていうコンセプト自体、俺は嫌いだし、これからは時代錯誤になってゆくんじゃないでしょうか。オリンピックじゃあるまいし、科学者が仕事を地球上のどこでやったって、構わないと思うのですが。俺は何処にいようと、たとえアメリカ市民権を取ろうと、中身は最期まで日本人なわけだし..

日本の方が良いところ:何と言っても、食い物です(爆) あー、旨いラーメンが喰いたいっ!
650名無しさん@お腹いっぱい。:2000/10/26(木) 15:27
よかった、来てくれて。嬉しいです。
651MY:2000/10/26(木) 15:30
>>614 留学先で伴侶を見つける..

それは失礼いたしました。お詫びに、一言だけ「アドバイス」らしきものを..
アメリカ人女医と結婚したYK君ですが、やはり、彼は日本人男としては例外的にanti-chauvinisticですよ。アメリカに居ると、(俺自身も人のこと言えませんが)日本の男共がいかにchauvinisticであるかが、嫌というほど見えてきます。国際的感覚(死語か?)を身につけたいなら、ニッポン男児は、その辺から自分を変えてゆくべきなのでしょうね。
652名無しさん@お腹いっぱい。:2000/10/26(木) 15:34
やっぱり起きてるじゃん・・先生。
夜型かな?
653名無しさん@お腹いっぱい。:2000/10/26(木) 15:41
chauvinistic=男尊女卑
654名無しさん@お腹いっぱい。:2000/10/26(木) 15:42
超無名の自分。
アメリカの研究環境の素晴らしさに感激してる。
何故かって日本に居た時より、研究上のストレスが少ないから。
日本の職(助手)を辞めて、こっちで武者修業を続けようか思案中。
655名無しさん@お腹いっぱい。:2000/10/26(木) 15:48
>653
逝ってよし!

正解)chauvinistic =愛国主義的
656名無しさん@お腹いっぱい。:2000/10/26(木) 15:58
657MY:2000/10/26(木) 15:58
>>616 >>618 家族のこと (ちなみに、>>85は偽物でした。実害ないので言わなかったけど)

奥方は同業者(筑波の医学の1年後輩)で、今Eric Olson研で2度目のポスドクしてます(彼、今年米国アカデミー会員になりましたね)。将来は、なんとか独立したいそうです。Olsonの所へ行く前には俺とも共同研究していたので、俺と共著の第一著者ペーパーが3報ほどあります(H. Yanagisawaです)。

俺は結婚が早かったので、長女はもう13歳(アメリカでは中2)です。その下に小3と小1の、3人姉妹です。

休日は...ご存じのように、先週末はネットしてました。(爆)
658名無しさん@お腹いっぱい。:2000/10/26(木) 16:04
>655

651の文章では
chauvinistic = 男尊女卑・性差別主義
659名無しさん@お腹いっぱい。:2000/10/26(木) 16:04
>in situ hybによるサポート無しのGPCR immunohisto(エンドセリ>ンに限らず)
初めまして柳沢先生失礼します。
少し質問したい事がございましてカキコさせてもらいます
免疫染色に際して上記は一般的にあてはまる事なのでしょうか?
サイトカイン等の微量物質の免疫染色は個人的経験的にも疑問があり
先生のおっしゃる通りだと思うのですが、これは一般的な細胞内蛋白
の免疫染色にもあてはまるのでしょうか?
ISH併用すればかなり実験の精度が向上が望めるって事なのですか?
またRT-PCR、alley等でmRNAを調べた後で免疫染色を行えば精度としては充分と考えてよいでしょうか?
話が抽象的・あいまいで申し訳ないのですが、
時間・その他にさしつかえない範囲でお答えくだされば光栄です。
(ちなみに僕は形態屋・病理医なんでちょっと免疫染色というもの
 をどこまで信じてよいものかということに興味があるのです)
660659:2000/10/26(木) 16:07
僕はエンドセリンの研究者ではないので申し訳ないのですが
ちょっと興味が湧いてきたのでよければお願いします。
661659:2000/10/26(木) 16:09
>必要以上に懐疑的になることはありませんが、正直いって、in
>situ hybによるサポート無しのGPCR immunohisto(エンドセリン
>に限らず)は、信じないことにしています。
たびたびすみません、このカキコに対する疑問です。
662MY:2000/10/26(木) 16:20
>>653 >>655

Chauvinisticという言葉ですが、確かにこの両方の語義があります。でも、アメリカでこのようなコンテクストで使ったら、「男尊女卑」の意味にしかなりません。念のため..
663名無しさん@お腹いっぱい。:2000/10/26(木) 16:30
筑波は何もないからってことで、子作りですか?
664MY:2000/10/26(木) 17:25
>>486 臨床医の基礎研究、いわゆるphysician-scientistについて。

これは、日本でもアメリカでも、別の意味で実に根深い問題だと思います。

まず、日本での現状については、>>265でもちらっと私見を書かせていただきました。全くチャネラーさんのおっしゃる通りで、日本の臨床医(ないしは臨床の医局に属する人々)で基礎研究を目指す人たちにありがちな最大の問題点は、『不必要で時に卑屈とも思えるほどのボス(主任教授)への忠誠心』だと思っています。もちろん、彼らの忠誠心に完全に寄りかかっている教授の方にも根本的問題があるのでしょうが、それを言い出すとキリがないので、ここではあくまでも若い先生方の側からの問題として捉えましょう。最初からそれほど真剣にM.D.研究者としてやって行く意志はなく、ただ「博士号」が取りたいために一時期研究をする方も多いと思いますが、そういう人はそれでいいのです(ただし、そうでない、基礎研究に本気なM.D.までがそういう方々のためにPh.D.の人たちから煙たがられたりするのは可哀想ですが)。

しかし、真剣に臨床医&基礎研究者になりたいと欲している若いM.D.の方々には、この、医局への忠誠という「まやかしの安全弁」に惑わされない勇気とambitionを是非とも持っていただきたい、というのが俺の心底からの希望です。アメリカに来て9年、これまで、こちらの研究室で本当に素晴らしい仕事を成し遂げていながら、日本に帰った途端に、「医局の意向・ルール・年功序列配慮・留学お礼奉公」等々によって、まともな基礎研究の全く続けられない立場に追い込まれてしまう人をイヤというほど見てきました。彼(彼女)らに、客観的に見て研究の才能があまりなさそうなのなら、気にしませんよ。でも、彼らの多くが、実に素晴らしい研究成果を挙げて来た人たちなのです。[幸い、俺自身のラボから日本に帰って行かれた先生方々は、今のところ全員、きちんとした研究が続けられる(時間的にもプロテクトされた)立場に戻られています。本当に幸いな事で、本人および関係者全員に心から感謝です。]

俺に言わせれば、やはりこの「医局への忠誠心」は、その人のambitionの不足の顕れでしかないのです。医局・白い巨塔にべったり奉公していれば、いつか必ず教授になれるわけではないのでしょう?どうして、そういったバックアップと逃げ場を全て自分から外して、自分を解放して、背水の陣の精神をもって、基礎研究なら基礎研究が好きな自分に100%賭けてみることが出来ないのでしょう。もしそれで本当にダメなら、その時はその時でいいじゃあないですか。M.D.なんだから、いざとなればいくらでもツブシは効きますよ。思い切って医局を離れて、成功すれば大いにハッピー。たとい成功しなくたって、自分の信念に従って一生懸命やったのだという自覚がもてるのなら、アンハッピーになるわけないと思いますよ。


転じてアメリカでは...こちらは、人事システムについても徹底的に個人主義の社会ですから、このような「一生をかけたボスへの忠誠心」のような現象は起こり得ません。しかし、physician-scientist、特に、真に臨床的な基礎研究を目指す者は、こちらでも色々な問題に直面します。それを書き始めるとキリがないので、今回はひとつだけ文献を引用して失礼します。うちのChairmenが書いたエッセイで、彼らいろいろ良いこと言ってますので、是非、時間のある時に読んでみてください。

Goldstein@` J.L.@` Brown@` M.S. The clinical investigator: bewitched@` bothered@` and bewildered -- but still beloved (editorial). J. Clin. Invest. 99:2803-2812@` 1997.
665名無しさん@お腹いっぱい。:2000/10/26(木) 17:40
>柳沢先生
2ちゃんねる嫌いの主人も、楽しみにしています。
666名無しさん@お腹いっぱい。:2000/10/26(木) 17:43
柳沢先生の話面白い。
弱いモノいじめして威張り腐っているうちの教授に
送ってやりたい。
667名無しさん@お腹いっぱい。:2000/10/26(木) 17:53
先生はブラインドタッチできるのかな。
あれだけの文章を書いてるし、書き方も上手だから相当慣れてるね。
2ショットチャットで技を磨いたのかな?
668名無しさん@お腹いっぱい。:2000/10/26(木) 17:56
みんなブラインドタッチじゃないの???
669668:2000/10/26(木) 18:00
そう言えば、ブラインドタッチとは言わないんだよね。
なんつったっけ。
670N.U:2000/10/26(木) 18:21
タッチタイプとかいうんじゃない?
671堕ちた研究者:2000/10/26(木) 18:29
>669
タッチタイプだよ・・
672TS(MY先生の弟子のひとり):2000/10/26(木) 18:29
MY先生がいらっしゃるとはびっくりです。すごいもりあがりですね。
全部読むのが一苦労です。
論文の方もよろしくお願いしますm(._.)m
673668:2000/10/26(木) 18:44
>670@`671
ありがとう。
動 touch-type キーを見ないでタイプを打つ
名 touch-typing
名 touch-typist
でした。
勉強になりました。
674TS:2000/10/26(木) 18:50
しかし、ここにはあまりに下品な書き込みが多いことにもビックリです( ゜_゜;)
675名無しさん@お腹いっぱい。:2000/10/26(木) 18:56
603です。お返事ありがとうございます。
664のレスはほんと眼からうろこが落ちる思いです。
先生のおっしゃるとおりどこに所属してようが、個人は個人であり、個のレベルではそのとおりだと思います。
ただ現在、特許(知的所有権)の問題が外交問題にまでなっています。
日本は医学生物産業の分野では完璧に出遅れて、‘playerですらない’とアメリカに言われています。
社会レベルからみると“頭脳流出”も度が過ぎると経済的・商業的に問題になりそうです。
まあ、会社の国籍なんてどうでもいいじゃないかといえばそれまでですが。
今後は先生のfollowerがもっと出て、‘外圧’で日本をかえて欲しいと思います。
676堕ちた研究者(14-3-3):2000/10/26(木) 19:12
>675
followerはあくまで二番煎じ。MY(他人)に期待しているようでは何も変わらないよ、
自分も、まわりもね・・
俺は自分の力で変えてやるっていう位の気構えでいるけどね・・

>674
大部分は俺に恨みをもつ人間が、俺の名を語ってやってること。
君も自分から「弟子」と名乗ってるようでは、二番煎じ路線だね(笑。
677名無しさん@お腹いっぱい。:2000/10/26(木) 19:21
675です。
>676
follower ってのは、“柳沢先生のような研究者”
という意味でつかったつもりです。
私は臨床、しかも外科系ですんで、組織の力が強すぎてどうも・・・。
678TS:2000/10/26(木) 19:36
>676
何が二番煎じなのかわからないし、なにが笑えるのかもわかりませんね。
きょう、ここを見ただけだから、よく勝手がわかりません。
僕にはMY先生のような力はありませんし、弟子でもいいです。
「元弟子」「後輩」でも、いいです。
いまでは、曲がりなりにも半分独立できましたので。
14-3-3を研究してる方ですか?
いろいろ教えてください。
いいデータでてますか?
679ぴねる:2000/10/26(木) 19:40

 ちなみに私自身は学生時代にちょっと本庶研など覗いたことありますが
何かと関心が持続しないのと享楽的な性格なのを自覚して結局神経内科にしました。
要するにそんながんばる人間でもないし。

 ところでMDの基礎研究の話、私自身は「学位だけ取ったらいいや」派ですが
身も蓋もないのですが、皆さん、ほんまに研究好きでやってるんでしょうかねえ?

個人的には次のようなパッシブなファクターが大きいと感じておるのですが。


#:臨床(特にいわゆるメイジャー(藁)では)にいった場合の生活が次の2通りしか
  選択肢がない(→どっちも気が進まない)。

 a.地域の中核病院で病棟に縛り付けられて自由時間のない奴隷のような生活

 b.老人病院や保険医などで、ヒマではあるがdullな仕事をする生活
  (たまにバイトでのぞきますが自分の気力とか時間とか脳みそをシュレッダー
   にかけたらチャリンチャリンとコインが出てくるような生活ですね)

##:良くある偏差値意識=将来のゴールとして以下のようなランキング
  (旧帝大教授>国立、、>私立、、>市中病院院長)
  個人的には適当な大学の教授になってもそんな楽しいとは思えないのですが

 というわけで本気で研究好きなわけではないので、いろいろと研究の阻害要因があっても
それを改善しようという力学があんまりなかったりしてと疑っております。それにしても
大学病院って医者の労働は無料だという経済原則で動いていて(よって医者がナースや技師や
事務官の仕事をする)、一方でその労働に対する評価は全くないので、臨床的なパフォーマンスの
低いこと低いこと。科あたり50床程度だったら本来、せいぜい5人くらい医者が臨床に専念して
働けば充分回るはずなのに、、。


 個人的には臨床好きなのですが選択肢のaとbどっちも嫌だし、更に大学病院の臨床は大嫌いだ
し(これを改善するためにエネルギーを使う気もないし)、基礎研究を続けるのは柄じゃないし、、
でちょっとヤクザな路線を目指そうかなと思ってます。
そのヤクザ路線には学位あった方が便利そうなんですよねえ(笑)。
(ヤクザ路線とはもっともらしい所に籍をおいて、南の離島や北海道の奥を巡回するとか、
やや怪し気な私費診療に一枚噛むとか変化に富んだことをしたいなと)
680ぴねる:2000/10/26(木) 19:42
あれっ、679に冒頭が抜けてしまいました

 どうも初めまして
なんかすごい勢いでレスが増えてますね
私も便乗してちょっと質問してみようかな
(それと独り言もぼそっと書いてみたりして)

Q:先生は進路で臨床方面行こうとかは全く迷われなかったのですか?
681名無しさん@お腹いっぱい。:2000/10/26(木) 21:16
あげます
682>679:2000/10/26(木) 21:59
他人の芝生は青く見えるってホントだとおもいます。
いろいろとね。
683名無しさん@お腹いっぱい。:2000/10/26(木) 22:38
あげておこう。
684悲しき天使:2000/10/26(木) 22:43
柳沢先生、はじめまして。宜しくお願いします。

在米の医学研究者です。MD. PhD.(日本)です。東海岸のある有名大学でポスドクをしています。将来は臨床に戻る予定で、こちらでの臨床も考慮しています。また現在はやや基礎的な研究もしています。先生のお名前やお仕事は日本で博士過程をしていたころから存じております。実際の所先生と同世代で年齢はそうそう離れていないようにも思われます。

研究をしていてつくづく思うのですが、成功を収めようと思えば、やはりどこかで「ブレーク」しなければいけないような気がします。そしていざ大ブレークして、自分の道筋が作れれば、先生のエンドセリンから始まった仕事のように、脈々と自分の城を作れるのではないかと思います。

しかしそれには、先生が述べておられたように、実力もさることながら、そのような機会を作ってくれた上司や、実験の運にもめぐまれなければいけないだろうと想像します。今の私にはまだその状況が難しいです。そこで、失礼ながらひとつお訊きしたいのですが、のちの膨大なお仕事に繋がっていく最初のペーパー(1988?)を出されたとき、どの程度ご自身のお力で、ペーパーをまとめあげられたのでしょうか?上司の先生の恩恵とはいかなるものであったのでしょうか?

それと研究とは関係ありませんが、アメリカでメジャーな科(内科、外科)で教授になっておられる日本人の方をどの程度ご存じでしょうか?

失礼な質問の連続で申し訳ありません。また個人メールを出すことがあるかもしれませんが、その節は宜しくお願いいたします。
685柳川定食:2000/10/26(木) 22:53
>684
ブレイク・スルーね。だれでも巡り合いたいよ。
これに関して、師はこのレスのなかで少しコメントしてたぞ。
ボスはパワフルなほうがいいといってます。
686どぜう鍋:2000/10/26(木) 23:22
>685
いい研究を論文にするのが、もちろん一番だけど
その後の評価は他人がするもの。やはり大物の目にとまると
何かと得な面が多いだろうね。
欧米のビッグボスは、若い才能を育てるのも自分達の義務だと
考えているみたいだ。日本では、出る杭は打たれる。
687名無しさん@お腹いっぱい。:2000/10/26(木) 23:29
>659
私も柳沢先生の意見を聞きたいと思います。ただ、形態の専門家がどういう意見を
持っているのかも合わせて聞いてみたいですね。in situというのもかなりartifact
が出やすい手技ですから。ポジコン、ネガコンをしっかり取るとか、吸収試験を
しっかりやるなどで、免疫組織もバカにしたもんではないと思います。
(ハイレベルなmolecularの話が出ている中で失礼しました)
688生姜焼き定食:2000/10/26(木) 23:35
ボスは一度なると、一切他人から評価・査定されないうえに、
終身雇用だから始末が悪い。ほんと変なのいるよな。
689ランランの肉豆腐丼:2000/10/26(木) 23:40
>688
アメリカでは学生等による教官の評価があるそうですね。
690悲しき天使:2000/10/26(木) 23:51
柳沢先生へ

他のスレッドで「東大医学部はあほばっかり」というあいもかわらぬ東大是非論がでていました。ここで柳沢先生は、東大卒もしくは関係者なら同等の仕事ができたか?・・・というような議論がありましたが、一言。

わたくしも、一応同じ穴の狢(他大学ですが)で、自分の首を絞めるような発言ですが、最近変わってきたとはいえ、日本の知識偏重型の入学試験はやはりその人の本当の能力を反映していない面があると思います。

端的に言えば、すべて予め「答え」がある問題だからです。そしてそういった問題に対しては、いかに普段から類型の問題をやって対応能力をつけられるのかという、努力、勉強総量が問題となるのです。ですからそういう能力は、従来の入試でも確かに見ているのかも知れませんし、そういった能力が重要であるとかんがえられるprofessionにつかれる方にはいい評価基準なのでしょう。しかし、本当に世の中に貢献できる基礎的な仕事とは、いままで誰もやっていない、「独創的な仕事」や「従来の既成観念をうちやぶる仕事」だと思います。

例えば、「世界で未解決の難問、謎」などと同等な問題を朝9時から夕方5時まで考えなさい。と言った問題形式で、参考書、本いっっさい持ち込み可にして、一日目数学、二日目理科などとやったらどうかと思うのですが。。。そしてその人の施行プロセスを徹底的に検証して、判定する(評価や点数かは難しいですが)

東大合格者のかなりが入れ替わるでしょう。柳沢先生も「東大合格者に彼女を奪われて悔しい思いをされる」こともなかったのではないでしょうか。。。(今のような仕事が出来て有名になっていたかどうかは知りませんが)

ただし文部省がやろうとしている教育改革で、英語の時間も減らす、必須単語数も減らすというのは誤りですね。こういう語学関係こそ「努力の総量」です。それは留学してみて感じます。

今までの日本の英語の学習法が誤っていたのは、「文法作文の時間が多すぎた」のではなく、「会話など実用的な部分が少なすぎた」だけであって、入れ替えるのではなく、はっきりいってプラスしなければいけない、単語などは高校卒までに従来の5000どころか1万にまで増やさなければ、国際人として通用しない、ということです。

以上長々と述べました。柳沢先生、この辺のご意見はいかがなものでしょうか?
691チャネラー:2000/10/27(金) 00:01
先生、わざわざレスありがとうございました。
今後も色々お話を聞かせてください。
現在やっているclinical reserchに対するファイトが
でてきました。意外とbasic reserchをやっていた頃の事って
clinical reserchでも役立ってるんですよね(内容とは別に色々なことが)。
692うに丼:2000/10/27(金) 00:53
あげておこう。
693MY:2000/10/27(金) 01:09
>>659 免疫染色

申し上げたのは、あくまでもGPCR(膜7回受容体)に関して、です。GPCRの免疫染色は、伝統的に、タグ無しでは非常にトリッキーなことが多いからです。その他の抗原に関しては、判断はもうケース・バイ・ケースでしょう。例えば、免疫染色を行って、既に確立されている組織分布あるいは細胞内分布が再現性よく得られた場合は、まず善しとしてよいのでは?でも、これまでに言われていなかった、以外な場所に発現していることを見つけたような時は、やはりmRNAレベルでのサポートが必須と考えます。最低限RT-PCRやNorthern、できればやはりin situ hybを併用することが望ましいと思います。

これは免疫染色に限らず、全く独立な2つ以上の手法で再確認する、というのはデータの信憑性を高めるための必須の手段ですよね。

694MY:2000/10/27(金) 01:22
>>672 >>674

うわっ、これはどうやら本物のTS君のようですね(当スレッドに既出です)。こんな所でカキコしているのを見つかってしまいましたか...

ホントだ。ここで油売ってる暇があったら、彼の論文早く返さなくちゃ..(^^;;
695うに丼:2000/10/27(金) 01:27
>MY先生
675について、どう思いますか?
696MY:2000/10/27(金) 01:47
TS君のために弁護しておきます。彼は謙遜して「弟子」という言葉を使いましたが、それはあくまでもlineageという観点からであって、彼は俺の「子分」や「2番煎じ」などでは全くありませんぞ。俺だって、lineageとしては江橋の孫弟子、眞崎の弟子ですが、自分が彼らの2番煎じだと思ったことはないですからね。あと、正確に言うとTS君は俺の兄弟弟子と言った方がよいと思います。
697堕ちた研究者@どぜうの丸煮@浅草:2000/10/27(金) 01:56
うるわしい師弟愛ってやつかな(藁
フーッ、煎じ茶がうまいわい・・
698うに丼:2000/10/27(金) 02:24
これをあげて、今日はねる。
699堕ちた研究者@どぜうの丸煮@浅草:2000/10/27(金) 03:06
>>678
うわさの助教授殿の登場とはね・・
こんなとこで遊んでていいのかな、医学生に「2ちゃんの○○ちゃん」
とか揶揄されるよ。あの大学は周りに娯楽がないから、中傷やうわさ話しが
大好きだからね・・
まあ、イニシャルで登場するくらいだから、皆に気付いて欲しいんだろうけどね。
700名無しさん@お腹いっぱい。:2000/10/27(金) 04:06
>684@` 690
謙遜を鵜呑みにするバカ発見!
701だめドクター:2000/10/27(金) 06:43
 今日は、主役の方があまりお見えにならないですね。もうすぐ、日本にこられるという話を噂でお聞きしたので
準備などがお忙しいのか、もう飽きたか、日本中に広まりすぎて出づらいなど。
僕はもう博士課程を終わる(含願望)のですが、結局満足の行くような研究しか出来ませんでした。
上のほうで述べられていたように、
アメリカのgraduate schoolでは、『ストーリー』を教えろ!」
とありますが、それは授業などだけではなく教室などのセミナーや普段の研究のdiscussionなどでも
そうなのだと思います。現在もそうなるように普段から努めていらっしゃるのだと想像しております。
具体的には、どのようにして、MY先生の場合は身に付けられたのでしょうか?
日本の大学院出身でいらしたと記憶しております。
やはり、勉強あるのみ?優秀なボスとの会話?もってうまれた素質など?
このくらいしか考えつかないです。
ご自分のが答えづらいようなら周りにいたそういう人たちの話でもいいので
お聞かせください。
non-MDですが、お聞きしたいです。non-MDですがってとこが卑屈になってますが実は自信家です。

702名無しさん@お腹いっぱい。:2000/10/27(金) 06:43
記念カキコ
703だめドクター:2000/10/27(金) 06:45
>>701 「満足のいくような研究しか」じゃなくて、 「満足のいくような研究が」
です。すみません。お馬鹿でした。
704名無しさん@お腹いっぱい。:2000/10/27(金) 07:08
柳沢先生に「国民栄誉賞」を希望!
705名無しさん@お腹いっぱい。:2000/10/27(金) 07:25
おれも有名になったら、本名を名乗ってカキコしてみたい。今名乗っても、誰も知らないので寒いだけだ。
706堕ちた研究者:2000/10/27(金) 07:37
最初はかしこまってたレスが、段々2ちゃんらしくなってきていいね・・
707700レス突破記念! イタイ奴ベスト3!!:2000/10/27(金) 07:59
1. 堕ちた研究者(荒しとわけわからん自己紹介)
2. 「固唾をのんで…」「先生にも将来が…」の勘違いオヤジ
3. MY 先生に連絡するのならメールですむものを、自分が話題になっているのが
嬉しくてイニシャルでのこのこ出てきた助教授君
708TS:2000/10/27(金) 08:09
>>699
僕は掲示板という場を良く知らないのですが、ここはそれほど有名な場なのですね。
知人に教えてもらってのぞいてみたのですが、書き込みをしたのは間違いであったようです。
堕ちた研究者さんは、面と向かってはなしているときでも、人の話をバカにすような発言をする人なのでしょうか?
僕は掲示板は良く知らないのですが、すくなくとも直接あって話すときと同じくらいの節度をもったことしか書くつもりはないのですが・・・
ですから、正体がわかろうが、問題ありません。
いずれにしろ、アドバイスありがとうございました。
ここでは、無理なようですが、他の方とは、どこかでまともな研究の議論ができるといいですね。
709サイコロ:2000/10/27(金) 08:13
柳沢先生こんにちは。ただの一臨床医です。
中学の頃臨床医を志し、目標がちょっとずつ変化はしたものの、これまでの人生も仕事も
半分くらい思っていたあたりに来ています。でもまだやり遂げたいことが他にあり、片手間ですが手をつけ始めています。

先生はいつ頃から研究者を目指されましたか。筑波の医学部に決めた経緯は?高校当時でもう
ボスの文献あたってたんですか?
ブレイクなんていうと漠然としてますが、何かあるのは間違いないという感触からエンドセリンに至るまで、
いつ頃からどんな風につかんでいったのでしょうか。
今の仕事はどのくらい楽しいですか。将来設計はどんなでしょうか。
710名無しさん@お腹いっぱい。:2000/10/27(金) 08:22
柳沢先生は、高校時代にすでにScienceに投稿したことがあると聞きました。さすがにアクセプトはされなかったようですが。
711TS:2000/10/27(金) 08:22
>>707
話題になっていたのですか?それは知りませんでした。
出てきてすみませんでしたね。申し訳ありません。
712MY:2000/10/27(金) 08:26
>>159 だいぶ以前の質問ですが..

英語のfluencyを鍛えるしかないですね。それも、言語としての流暢さだけでなく、なんつーかな、言い回しの日米文化差、みたいなものが分かってくると、「渡り合う」のが苦痛でなくなります。
713名無しさん@お腹いっぱい。:2000/10/27(金) 08:30
>TS
ひがまれているのよ。実力のある証拠。気にしない気にしない。
714電波妨害:2000/10/27(金) 08:35
>711
雲の上の話がきけてうれしい。
http://www61.tcup.com/6124/surgeons.html
わたしが持っている管理しない掲示板つかってもらっていいけど、
2chでやってくれたほうが面白い。
715だめドクター:2000/10/27(金) 08:40
 こちら側からの都合ですがTS先生とMY先生がこういった場で臨場感のある話をしていただけると大変、
ためにもなると思います。ある質問に対して、MY先生はそういっているがTS先生は俺はちょっと
違うぞみたいな。一方的な意見を聞くだけでなくいろんな考えがあるもんだみたいに
思えるような気もします。
 いまや、掲示板、インターネットはもはや一つの文化といえます。内容さえきちんとしていれば
非難される筋合いはないと思いますが。
716名無しさん@お腹いっぱい。:2000/10/27(金) 08:47
>715
そうかもしれんが、一流の研究者の中でそういう議論が早く出来るように
自分が精進する方が、自分のためにもなると思う。自信家のわりには、
他人にあこがれてるんだね。
717名無しさん@お腹いっぱい。:2000/10/27(金) 08:52
こんにちは、医学部志望の受験生です(国公立大学)。柳沢先生って凄い方なんですね、知らなくてすみません。私の成績ではさすがに筑波の医学部は無理ですが、一応医学研究者を目指しています。今は質問も思い付きませんが、そのうちご質問させていただければと思います。それではまた。
718だめドクター:2000/10/27(金) 09:14
>>716
なるへそ。ためになるという意味がおわかりにならなかったようだ。
僕のど素人的な考えでは研究をやっていく上で最も大事なことの一つは議論
ができること。その議論とは人のあげあしとりなどではなくきちんとした
論理の展開。それができる人同士の意見の交換というのは時に僕など博士課程の人間
からみて、上の先生たち同士はもちろん学部生の後輩同士の似非ディベートもためになる。
そういう意味もあってMY先生の意見だけを聞くのも面白いが・・・ということです。
議論の楽しさも知ったほうがいいのではないかというのが僕の意見です。
別に権威主義に陥っているわけではないし、まったく鵜呑みにするわけでも俺とは
違う意見だ、じゃーだめだこいつってのでもない。色々な意見を聞かずに自分にのみ
自身を持ち人の意見を聞き入れないのではどこかのドキュソ研究室のボスではないでしょうか。
すみません。眠くてかっとなりました。この性格がいかんのだ。

719うに丼:2000/10/27(金) 09:15
>708 TS氏
文面から判断するに、2ちゃんねるにはなれてないようですね。
ここは匿名が保証されたとてつもなく無責任な掲示板なんです。
よって、荒らしとよばれる無法者による煽り、中傷なんでもあり
とういうとこなんです。そういうのに反応しても無意味です。
徹底的な無視が一番なんです。匿名だから本心が覗けるということも多いようですが
最低限のマナーすらもってないのが結構いますし、ネット犯罪者まででているところです。
そんなところに、実名・メアド付きで登場してくれたMY先生は、ほんとに
器の大きさを感じてしまいます。TS先生もあまりその辺こだわらずに、また
意見交換してくれると、うれしいとおもいます。
720名無しさん@お腹いっぱい。:2000/10/27(金) 09:20
おれも、もうちょっと有名だったら、実名で登場して意見をかっ飛ばしてたのに・・・。
721MY:2000/10/27(金) 09:22
おいらはヤリチン
17歳で子供産んだ。
長女は中2
722MY:2000/10/27(金) 09:23
おいらはうんこ人間
17歳で餓鬼つくちゃっちゃのじゃ。
できちゃった結婚しゃ。(もち中だしがたたった。)
中だしってドキュンしかしねーよな・・・・
とほほh
723MY:2000/10/27(金) 09:24
長女は13歳
オイラは40歳
724名無しさん@お腹いっぱい。:2000/10/27(金) 09:24
たしかに。きみだれ?詳細きぼーん!なんてかかれたら恥ずかしいしね。
725MY:2000/10/27(金) 09:24
あ、やべ
計算まちがった。
AHOなことが悟られてしも^た
手屁・・・。
726MY:2000/10/27(金) 09:25
>719
あほ
レス嫁
おいらは2CHがこんなアングラ劇場とは知らなかっただけで、乗り込んで北わけじゃねー
727名無しさん@お腹いっぱい。:2000/10/27(金) 09:25
>718
自分で研究者を相手に討論できるような場をさがせばいいのに。
自分で体験したことは、必ず自分の肥やしになるぞ。
ここで他人のやり取りみてるより面白い。
討論するときは、自分より上の人間を選ぼう。小学生と討論しても
得られるものはないかもしれんが、実力が上の人間との討論は、必ず自分を上に
引き上げてくれる。
728名無しさん@お腹いっぱい。:2000/10/27(金) 09:27
>721−723
他人のメアドを名乗っての中傷。とうとう、やってしまいましたね。
これであなたは立派な犯罪者になりました。
729MY:2000/10/27(金) 09:28
屁?
おいらだけど・・・
730MY:2000/10/27(金) 09:28
2CHじゃ常識
731MY:2000/10/27(金) 09:29
爆〜〜〜ばく〜〜〜
バクは哺乳類
732MY:2000/10/27(金) 09:30
今、ちょっと悩んでる。 自分の
今までの人生を考えると・・・・・ まるで絵に描い
たもちのような、そんな空虚感にさいなまれるんだな。 安らぎの瞬間、というのが
ここにカキコしてる時なんだな、今は 。いい朝だ。Bonnie Ridgeを車で回ってみよ
う。 美しい韓国女性に会えるかもしれない‥ そして、目が合えばニイハオと話しか
けてみよう。 バックスのラテは美味いね。睡眠最低4,5時間は取る主義だけど
ね。


733MY:2000/10/27(金) 09:30
・ソーダが妙に胃に染み渡るな・・・・・今日はなんだか自分
じゃないみたいだ。この掲示板の存在を知らなければ、俺の人生
は虚構のハリボテだったろうな・最近、時間がたつのが妙に早い。俺も歳をとってき
たのかな最近ガソリンが高い‥ 明日は、ガスをいれたついでに洗
車して、部屋の掃除もしよう‥ヘアカットにも行こう‥
734MY:2000/10/27(金) 09:31
いつからだろう‥‥やらなければい
けないことを、先送りにするようになったのは‥集中力の無さという
か、落ち着きのないところがある。ふと、「おい!」といわれてハッとする事が 挙
動不審というのかも知れないが
実はこの板で一心不乱にカキコしている時が一番安心できる時なのかも しれない 。
どこか遠くへ逃げてしまいたい、と思うのだが・・・・また同じ日々の繰り返しか・
・・・ 最近は、米の買い出しにも行ってないな‥
韓国店で買うと高いし、ハナンルーまでのドライブが 楽しかったのも最初の数回だ
けだった‥‥部屋のカーテンを閉め切っているのがまずいのかな?でもできればあけ
たくないな 。それにしても、ハナンルーで見た韓国女性は、美し
かった‥独特の化粧のせいかもしれないが‥ 。化粧は、男をだますためというが‥
一生だまされたままなら、それはそれで‥ 一つの真実だろう‥ 。一見水商売風だ
が、妙に刺激される。五感が本当に洗われるようだ 。コーヒーでも飲むか・・・・
・・・自分にとって、
女とはなんだろう?ふとそう思ったな。だまされたままで死んでいく「文系」の連中
は、幸せだろうな‥
現実というものを知らずに‥今夜も‥ この板で、最後の一人になった‥しかし、
「現実」を知ったところで、何になるのだろう? 世の中全て、「架空の世界」みた
いじゃないか。皆、疑う事も無く働き、家庭に戻る。 それなら、だまされたままの
方が幸せかもし
れないな・・・・・・ソーダが妙に胃に染み渡るな・・・・・
735MY:2000/10/27(金) 09:31
最近金魚蜂の金魚を眺めてもの思いにふける日が多い 。頭の中の自分と実際の自
分とのギャップに戸惑い、妄想に逃げるのはまずいと思いつつも、この掲示板に書き
なぐることが多い ・・自信が無いのだろう
か?それにしても、静かな週末だ‥思いきって明日は休むか‥かといって‥これと
いった趣味もないし‥ 。
736名無しさん@お腹いっぱい。:2000/10/27(金) 09:32
>730
世間では犯罪者。
2ちゃんで人生棒に振った人ってけっこういるんだよね。
737うに丼:2000/10/27(金) 09:38
>730
また2ちゃんから逮捕者がでるのか・・・・・。
千葉大事件ですこしはましになってきたと思ったもに。
またここたたかれるのかな。
738MY:2000/10/27(金) 09:39
みんにゃ〜
あたまナデナデしてくだしゃいね。
おいらこれが好きなんでしゅ。
初対面でもいいから、オイラにあったら、まずは「いいこいいこ」
っていいながらナデナデしてくだしゃい。
朝鮮人の娘のママをおもいだしましゅのじゃ
739MY:2000/10/27(金) 09:40
おまえが死ね
はげ
740本物のMY:2000/10/27(金) 09:41
まあボクはいいですけどね。
楽しいじゃん?
741本物のMY:2000/10/27(金) 09:41
Let's Have A Partyみたいなね。
742MY:2000/10/27(金) 09:42
やあ、やなやな
元気でシュか?
743堕ちた研究者:2000/10/27(金) 09:43
師ねはげ
744堕ちた研究者:2000/10/27(金) 09:43
おいらがJHUのポスドクのオチケンです
745堕ちた研究者:2000/10/27(金) 09:44
あ〜しんどい
746堕ちた研究者:2000/10/27(金) 09:44
今日はつかれた
747堕ちた研究者:2000/10/27(金) 09:45
筑波卒DEATH
748堕ちた研究者:2000/10/27(金) 09:45
CHAtしたい
749名無しさん@お腹いっぱい。:2000/10/27(金) 09:45
>729〜735
先生は明日から来日します。休んでるひまはありません。
あなたが韓国女性に憧れを抱く気持ちは理解できますが
書いてあることが実際の先生のライフスタイルとかけ離れてます。
なによりその三文ポエムは勘弁して。
750MY:2000/10/27(金) 09:46
>>>>701d
751MY:2000/10/27(金) 09:48
>>701

ちょっとクソ忙しくてカキコの暇がないだけです。
回答すべき質問はリストアップしてあり、もう1ダースほども溜まっております。(^^;;
752堕ちた研究者:2000/10/27(金) 09:51
和紙和紙
753堕ちた研究者:2000/10/27(金) 09:52
玉ってそう
754堕ちた研究者:2000/10/27(金) 09:52
チンポは元気?
>ヤナヤナ
755MY:2000/10/27(金) 09:58
あ〜今夜も暇だ
756MY:2000/10/27(金) 09:59
>オチケン
元気だよ
昨晩も女房をヒィヒィいわせたやったさ
757MY:2000/10/27(金) 10:00
758MY:2000/10/27(金) 10:00
バクチーチ
759MY:2000/10/27(金) 10:01
あ〜キムチくいてぇ〜
760MY:2000/10/27(金) 10:01
キムチいいことしてぇ〜
761MY:2000/10/27(金) 10:02
762MY:2000/10/27(金) 10:02
ぶひ〜
763MY:2000/10/27(金) 10:02
ホログラム効果ですね
764MY:2000/10/27(金) 10:03
屁をポラロイドでパシャパシャ
765MY:2000/10/27(金) 10:03
にゃんにゃんしてぇ〜1
766MY:2000/10/27(金) 10:04
読み方
ニャンニャンしてぇ〜ワン
767MY:2000/10/27(金) 10:04
今夜は鼻くそが格別にうまいわい
768MY:2000/10/27(金) 10:04
**********終了*************
769MY:2000/10/27(金) 10:09
かえすがえすも
*************終了*************
770MY:2000/10/27(金) 10:10
しつこいようだが
*******************終了*****************
771MY:2000/10/27(金) 10:16
名残惜しくも
**********終了*********
772名無しさん@お腹いっぱい。:2000/10/27(金) 10:21
とりあえず管理人に通報。その後、警察に告発しておこう。
ほかの実名名指しのスレもまとめてROM。
773名無しさん@お腹いっぱい。:2000/10/27(金) 10:24

おやくそく。
近頃、初心者さんが増えてきたので、お約束をつくってみました。
基本的には、ユーザーの良識に任せたいのですが、
「明文化したルールがない」ということを逆手にとってなにをしてもいいと誤解する人が多いので、、、
はぁ、、めんどくさ。。


他人に迷惑をかけるのはやめよう
芸能人や公人ではない一般人の誹謗中傷、プライバシー暴露は禁止してます。
一般人の電話番号や住所などを見かけたら削除依頼にどうぞです。

もし、プライバシーが公開されてしまった場合は、ぜひ最寄の警察にも相談してください。
警察、裁判所から要請があれば、捜査協力してそのような書き込みをした人をつれてってもらってます。
警察か裁判所以外の問い合わせでは一切動きませんので、警察に行くことをお勧めします。

また、2ちゃんねる内では、「誹謗中傷板」以外の場所では、固定ハンドルさんを叩く行為は禁止してますです。
もし、誹謗や叩きがあったら削除依頼板にお願いします。


774元祖ポスドク:2000/10/27(金) 10:24
チンポ 
775元祖ポスドク:2000/10/27(金) 10:26
柳沢って一般人じゃないんじゃないか?
776柳川定食:2000/10/27(金) 10:26
ところで、MY先生は就寝中かな?
777元祖ポスドク:2000/10/27(金) 10:28
柳沢にもプライバシーってあんのか?
778元祖ポスドク:2000/10/27(金) 10:28
パパラッチか?
779>772:2000/10/27(金) 10:29
そろそろ、良いタイミングじゃないか?
これだけあれば動いてくれるよ。
なんなら、おれ告発しておこうか?
780元祖ポスドク:2000/10/27(金) 10:31
>779
がんばれよ
781堕ちた研究者:2000/10/27(金) 10:33
わっしょいわっしょい
782名無しさん@お腹いっぱい。:2000/10/27(金) 10:33
ねえ、良い子だからその辺で煽っている子供たち、
どこか別の板で遊びにいっておいでよ!
誰もここではボクタチのこと相手にしないから、もう無駄だよ!
783堕ちた研究者:2000/10/27(金) 10:33
祭りだ祭りだ
784堕ちた研究者:2000/10/27(金) 10:34
          ___/ 冫   ∧_∧
 ∧_∧  _ __ (_    /   (´∀` )
 ( ´∀`)/ ) )  |   |    //   \
 (  ⊃/ //  ∧_∧    (__(_/`  |
  \__//    ( ´∀`)       _///
       ̄     ∪ ̄∪      (__(__)

   「レ」       「イ」       「ク」


785堕ちた研究者:2000/10/27(金) 10:35
そうだそうだ
よそに逝け
786>772さん:2000/10/27(金) 10:35
よろしくお願いします。ありがとうございます!
ところでMY先生はファーストクラスの飛行機の中でおもむろに
ノートを出し、1ダースたまってる質問にレスを書くのかな、
先生の自己管理は心配要らないとは思うけどちと勿体ないような気がします。目も心配です。
787堕ちた研究者:2000/10/27(金) 10:36
今夜は週末なので、一晩ここでAHO達とCHATじゃ〜
祭りだ祭りだ
788堕ちた研究者:2000/10/27(金) 10:36
今日はHARDだったので、今夜はここでAHOとCHATじゃ
789告発関係スレはこちら:2000/10/27(金) 10:37
790堕ちた研究者:2000/10/27(金) 10:37
GO METS
791堕ちた研究者:2000/10/27(金) 10:38
ワールドシリーズみてるか?
ポスドク
792名無しさん@お腹いっぱい。:2000/10/27(金) 10:40
mannkohihi
793名無しさん@お腹いっぱい。:2000/10/27(金) 10:40
柳沢のアナル攻め
794名無しさん@お腹いっぱい。:2000/10/27(金) 10:42
柳沢の悪口はこちらに相談
http://www.fbi.com
795名無しさん@お腹いっぱい。:2000/10/27(金) 10:49
ほいほい
ついでに1000になるとここ閉まるから。
発言できなくなるからさ
796名無しさん@お腹いっぱい。:2000/10/27(金) 10:49
サーバーの負担軽減のシステムで名
797名無しさん@お腹いっぱい。:2000/10/27(金) 10:50
おーい
798名無しさん@お腹いっぱい。:2000/10/27(金) 10:50
あと241
799名無しさん@お腹いっぱい。:2000/10/27(金) 10:50
だれかセーブの仕方教えてくれ。
800名無しさん@お腹いっぱい。:2000/10/27(金) 10:51
裏技はSAGEをつかうこと
するとカウントされないからいつまでたっても1000はこない
801名無しさん@お腹いっぱい。:2000/10/27(金) 10:51
>795
それ本当?その後どうすればいいの?
802名無しさん@お腹いっぱい。:2000/10/27(金) 10:51
現在763
うちSAGEは39
803名無しさん@お腹いっぱい。:2000/10/27(金) 10:52
書きこめなくなるだけで閲覧はできる。
また新規にPART2をつくればいい。

>801
804名無しさん@お腹いっぱい。:2000/10/27(金) 10:56
最近オナに-やってる?
805659:2000/10/27(金) 10:57
実は現在免疫染色のノンスペを逆手にとったような実験してまして参考になりました。
お忙しい中お返事ありがとうございました。
806名無しさん@お腹いっぱい。:2000/10/27(金) 10:57
>804
やってるよ
807名無しさん@お腹いっぱい。:2000/10/27(金) 10:57
はげ
>806
808名無しさん@お腹いっぱい。:2000/10/27(金) 10:58
>807
はげ
809名無しさん@お腹いっぱい。:2000/10/27(金) 10:59
>805
はげ
810元祖ポスドク:2000/10/27(金) 11:08

>先生
MAILして
811HNを騙られています:2000/10/27(金) 11:33
HNを騙られています。
このスレの「元祖ポスドク」「堕ちた研究者」のHNの書き込みは
「全て」、某板の固定HNの「騙り」です。
大体、自分の経歴を喜んでこんなとこに書くアホはいません。

>管理人様
これら(おそらく2名)のIPを調べてアクセス禁止にして下さい。
法的手段を取るつもりであれば証言します。
812HNを騙られています:2000/10/27(金) 11:39
無名人は個人情報がもれると悪戯されるだけですね。
813N.U:2000/10/27(金) 11:51
匿名でなくきちんとした議論をしている掲示板には、たとえば東北大学数学科の黒木玄さんが主宰している掲示板などがあります。
ここさえ、たまには、問題児があらわれます。
814名無しさん@お腹いっぱい。:2000/10/27(金) 11:55
>811
ここに依頼しよう。

http://teri.2ch.net/saku/index2.html
815MY:2000/10/27(金) 11:57
>>265 で言ったことについて..

日本の官僚も、一応いろいろと考えていますね。ええこっちゃ。今日のAOLニュースから:

専門知識持つ民間人登用に道
 政府は27日の閣議で、高度な専門知識を持つ民間人を5年以内の期限付きで国家公務員に登用するための「任期付任用法案」を決定した。一般公務員よりも高い特別の俸給体系を設けるのが柱で、特に有能な人材には最高で東大学長と同額の俸給を支給する。給与面などで優遇することにより、民間との人材交流拡大を目指す。
816名無しさん@お腹いっぱい。:2000/10/27(金) 11:57
<dd><a href="../test/read.cgi?bbs=hosp&key=971904362&st=716&to=716&nofirst=true" target="_blank">&gt;&gt;716</a> <br>
817名無しさん@お腹いっぱい。:2000/10/27(金) 12:04
818名無しさん@お腹いっぱい。:2000/10/27(金) 12:17
>>265
>特にマウスの仕事をしていると、飼育費の高さに涙がちょちょぎれます・・・

「涙がちょちょぎれる」というのは、現在日本では死語となっておりますので、よろしくお願いします。
819某国立研究所研究員:2000/10/27(金) 12:24

独立法人化により、自由な研究が保証されるのかと思いきや、
丸で逆な結果になりそうです。
以前より自由度がなくなり、部門ごとに少数のテーマを挙げ、
部門員全員がそのテーマに沿った研究を余儀なくされます。

良いテーマなら、そういうやりかたも有効でしょうが、
海外の後追いの無難なテーマですから。
若手の発言権はないでしょう。
国のやることなんてそんなもんです。
820某国立研究所研究員:2000/10/27(金) 12:27

独立法人化により、自由な研究が保証されるのかと思いきや、
丸で逆な結果になりそうです。
以前より自由度がなくなり、部門ごとに少数のテーマを挙げ、
部門員全員がそのテーマに沿った研究を余儀なくされます。

良いテーマなら、そういうやりかたも有効でしょうが、
海外の後追いの無難なテーマですから。
若手の発言権はないでしょう。
国のやることなんてそんなもんです。
821名無しさん@お腹いっぱい。:2000/10/27(金) 12:40
MY先生
先生の語り野郎がいます。大変を手数うですが、ハンドルネームに以下の処理を
していただければ、まことに幸いです。

「キャップをかぶせる」って、どういうこと? top age sage ▲ ▼

固定ハンキャップ(なんちゃってニセモノキャップ)を設定することです。
匿名掲示板である為に、人気のあるハンドルネームを持つ人のニセモノが出た際の混乱を避けるためにある機能です。

○○○さんのニセモノは、"○○○"といった風に、名前が"括弧"でくくられます。
ニセモノ被害でお困りの方は、ひろゆきへのメールで申請してください。

その際には、利用プロバイダのメールアドレスと判るかたちお願いします。
リメーラ、転送メール、webメールなどでの申請はお断りする場合があります。
使いかたは、申し込んだ方本人にメールで説明します。
822名無しさん@お腹いっぱい。:2000/10/27(金) 12:42
821続き

ひろゆきへのメール
mailto:[email protected]
823名無しさん@お腹いっぱい。:2000/10/27(金) 12:43
824>初心者:2000/10/27(金) 13:33
1000でstopって本当?
700超えたあたりから誰かが回数だけ増やしているのはそのため?
先生が書き込めるようになるまでに終わっちゃう。
場所を移しておきませんか。でもやり方知らないし。
825元祖ポスドク:2000/10/27(金) 13:34
767 名前:MY投稿日:2000/10/27(金) 10:04
今夜は鼻くそが格別にうまいわい


826名無しさん@お腹いっぱい。:2000/10/27(金) 13:35
>824
だから、1000件を超えたら、もう一度「柳沢って誰だ? Part II」というスレッドを立てればいい。このスレッドは書き込めなくなるが、読む事はできる。
827佐藤純Ver.Dr.DRE:2000/10/27(金) 13:36
柳沢先生も包茎ですか?
828佐藤純Ver.Dr.DRE:2000/10/27(金) 13:37
キャップじゃなくて、皮を被せているんじゃ無いの?
829佐藤純:2000/10/27(金) 13:37
Al GoreとGeorge Bushはどうでしか?
選挙権もってますか?
830佐藤純:2000/10/27(金) 13:38
それとも専門馬鹿で医学胃害は興味なしですか?
831佐藤純:2000/10/27(金) 13:39
泌尿器科ってホモ医者多いですか?
832佐藤純:2000/10/27(金) 13:39
産婦人科でマンコが臭い場合はどうするんですか?
833名無しさん@お腹いっぱい。:2000/10/27(金) 13:40
↑ただ単に、1000番をゲットしたいだけのように思えるが・・・。
834佐藤純:2000/10/27(金) 13:40
こたえてkじゅれるまで寝ません
835佐藤純:2000/10/27(金) 13:41
馬鹿なNiggerの学生はいますか?
836佐藤純:2000/10/27(金) 13:48
先生のティムポは大きいと噂ですが、なんcm?勃起時
837佐藤純:2000/10/27(金) 13:49
馬鹿なChicanoはいますか?
838佐藤純:2000/10/27(金) 13:49
英語の下手な留学生にイライラすることはありますか?
839佐藤純:2000/10/27(金) 13:50
学生にされるとうれしいこと(Gradeをあげてやりたくなること)
はなにですか・
840佐藤純:2000/10/27(金) 13:50
Friendlyな学生とよそよそしい学生はどっちがいいですか?
841佐藤純:2000/10/27(金) 13:51
AttractiveなPresentationの仕方は?
842佐藤純:2000/10/27(金) 13:52
Thai FoodやChinesefoodを食べると口が臭くなるんですが、どうしたらいいですか
843佐藤純:2000/10/27(金) 13:52
だからThaiやChineyは臭いんですか?
844佐藤純:2000/10/27(金) 13:53
マンコの臭い女はどうしますか?
845佐藤純:2000/10/27(金) 13:54
846佐藤純:2000/10/27(金) 14:12
あげ
847佐藤純:2000/10/27(金) 14:27
あげ
848佐藤純:2000/10/27(金) 14:38
                    /⌒\
                    (    )
                    |   |
                    |   |
                    |   |
                    |   |    / ̄ ̄ ̄ ̄ ̄ ̄ ̄ ̄ ̄ ̄ ̄ ̄ ̄
                    |   | <    みなさん
                    (∀・  )   \_____________
                   _ノ_ノ´ ./
                   ( (´ ろ
                 ̄ ̄ ̄ ̄ ̄ ̄ ̄

          /⌒ ────── 、    / ̄ ̄ ̄ ̄ ̄ ̄ ̄ ̄ ̄ ̄ ̄ ̄ ̄
        /   .            ヽ  <   こんばんは!
        (   @`@`ノ_____/ノ  丿    \_____________
                       υυ(´ ろ
                 ̄ ̄ ̄ ̄ ̄ ̄ ̄ ̄ ̄

          /⌒ ────── 、
        /   .            ヽ
  〜′ ⌒ (∧ @`@`ノ_____/ノ  丿
   UU⌒⊂( ; ゚Д)つ      υυ(´ ろ
   ̄ ̄ ̄ ̄ ̄ ̄ ̄ ̄ ̄ ̄ ̄ ̄ ̄ ̄ ̄ ̄
        ∧
/ ̄ ̄ ̄ ̄   ̄ ̄
|   ・・・とりあえずちんちんどけろ
\___________


849佐藤純:2000/10/27(金) 14:46
                    /⌒\
                    (    )
                    |   |
                    |   |
                    |   |
                    |   |    / ̄ ̄ ̄ ̄ ̄ ̄ ̄ ̄ ̄ ̄ ̄ ̄ ̄
                    |   | <    みなさん
                    (∀・  )   \_____________
                   _ノ_ノ´ ./
                   ( (´ ろ
                 ̄ ̄ ̄ ̄ ̄ ̄ ̄

          /⌒ ────── 、    / ̄ ̄ ̄ ̄ ̄ ̄ ̄ ̄ ̄ ̄ ̄ ̄ ̄
        /   .            ヽ  <   こんばんは!
        (   @`@`ノ_____/ノ  丿    \_____________
                       υυ(´ ろ
                 ̄ ̄ ̄ ̄ ̄ ̄ ̄ ̄ ̄

          /⌒ ────── 、
        /   .            ヽ
  〜′ ⌒ (∧ @`@`ノ_____/ノ  丿
   UU⌒⊂( ; ゚Д)つ      υυ(´ ろ
   ̄ ̄ ̄ ̄ ̄ ̄ ̄ ̄ ̄ ̄ ̄ ̄ ̄ ̄ ̄ ̄
        ∧
/ ̄ ̄ ̄ ̄   ̄ ̄
|   ・・・とりあえずちんちんどけろ
\___________


850佐藤純:2000/10/27(金) 14:47
だれもいないの?
あそぼ
851佐藤純:2000/10/27(金) 14:47
こんばんわ
852佐藤純:2000/10/27(金) 14:53
こんばんわ
げんき
853佐藤純:2000/10/27(金) 14:53
わんわん
854佐藤純:2000/10/27(金) 14:56
柳沢の顔がみたい
柳沢のHPがみたい
855佐藤純:2000/10/27(金) 15:01
柳沢のチンポもみたいけど
856>柳沢の顔がみたい :2000/10/27(金) 15:07
Nature10月5日号のNewsに顔写真が出ているぞ。
まあ、偽医者の君には手に入らないと思うがな。
857佐藤純:2000/10/27(金) 15:13
ここにのせて?
>856
858佐藤純:2000/10/27(金) 15:14
Natureといえば、講談社だっけ?
安井雅人もでたよね?
感じあってる?
859佐藤純:2000/10/27(金) 15:15
JHU.EDUのマサト ヤスイって有名?
860佐藤純:2000/10/27(金) 15:15
Nature日本碁盤って公団者だよね
861佐藤純:2000/10/27(金) 15:16
柳沢暗殺計画があるらしい
Niggerの生徒を差別してねらわれているそうだ
862佐藤純:2000/10/27(金) 15:18
Natureって子供向けだからちょっと・・・。
863佐藤純:2000/10/27(金) 15:19
チンポでかいらしいぞ
20センチ
864佐藤純:2000/10/27(金) 15:19
柳沢のトウサツ写真がみたい
865佐藤純:2000/10/27(金) 15:20
柳はぶさいく?
866佐藤純:2000/10/27(金) 15:27
hげ
867佐藤純:2000/10/27(金) 15:29
精子
868>867:2000/10/27(金) 15:38
もう寝ろ!
869>ALL:2000/10/27(金) 15:48
田舎脳外科です。
嵐もきてるしスレも重くなったンで
パート2のほうへ移動しましょう。
(おれならタイトルは「YNに聞け」にするんだが)

http://cocoa.2ch.net/test/read.cgi?bbs=hosp&key=972622446
870>869:2000/10/27(金) 15:50
勝手に決めるなボケ!
871名無しさん@お腹いっぱい。:2000/10/27(金) 16:06
仕切りたがり。
872堕ちた研究者:2000/10/27(金) 16:36
bokki tyuu
873名無しさん@お腹いっぱい。:2000/10/27(金) 17:46
2ちゃんならではの雑音はしょうがないね。
アホがハッキリしているのでS/N比は高く、
読むのにたいした支障はない。
佐藤クン、さあもっと沢山書き込んだらどう??(冷笑
874堕ちた研究者:2000/10/27(金) 17:47
hage
875名無しさん@お腹いっぱい。:2000/10/27(金) 17:47
そろそろあげておこう
876堕ちた研究者:2000/10/27(金) 17:50
bokki age
877名無しさん@お腹いっぱい。:2000/10/27(金) 18:00
re bokki
878名無しさん@お腹いっぱい。:2000/10/27(金) 18:03
bokki hage
879名無しさん@お腹いっぱい。:2000/10/27(金) 18:04
bokki
880名無しさん@お腹いっぱい。:2000/10/27(金) 18:11
rebokki
881名無しさん@お腹いっぱい。:2000/10/27(金) 18:32
たしかにS/N比が高くて、くだらんレスを飛ばし読むのが楽だ。
コントラストがきついから、アホレスかいてもほとんど注目されんぞ。
882名無しさん@お腹いっぱい。:2000/10/27(金) 18:48
むう。荒らしにすらならないほどの階梯の差ってわけか。荒らそうと
試みて結局荒らすこともかなわぬというのは、一体どんな気持ちなの
かねえ。
883名無しさん@お腹いっぱい。:2000/10/27(金) 19:46
Yanagisawa's bokki raise
884名無しさん@お腹いっぱい。:2000/10/27(金) 19:48
210 名前:ポスドク@JHU投稿日:2000/10/23(月) 17:48
中学の時の柔道部の監督(柳沢正史)。いつもジャージ姿でギンギンに朝立ちしたまま
朝練に顔出してた。
目がさめてくると部員に稽古つけるんだけど、たとえば背負い投げを
かけようとすると腰のあたりに勃起したティムポが当たる。
寝技の稽古のときなんて・・  襲われそうになった女の気持ちがよくわかる・・

怖い先生だったので誰も文句言えずにいたがオレが三年の時女子部員が入ってきて
最初の朝練で号泣したのでそれ以降は練習を見てるだけになった。
(でも朝立ちはしている・・)211 名前:Old Friend投稿日:2000/10/23(月) 17:49
小学校の時、水泳のスクールで柳沢正史という少年がいつも、準備体操で勃起していた。。
インストラクターの向井に興奮していたのかなあ
でもいつもいつも準備体操で勃起して、ピチピチの競技用海パンにテントを貼らせていたのは不
気味だった。
ぜってー病気
212 名前:旧友投稿日:2000/10/23(月) 17:50
柳沢は中学の水泳の時間、デカイチンポを勃起させて、女子に悟られない様に
はずかしそうにしていたのがバレバレだった。
中2ともなると女子もいい体してるからな〜
厨房ですな。乳首とか透けていたもん
パッドをいれるのをしらなかったんだろね。
一部の女子
885Postdoc@JHU:2000/10/27(金) 20:12
bokki age
886名無しさん@お腹いっぱい。:2000/10/27(金) 20:58
sarashi age
887FORCEPS:2000/10/27(金) 21:14
>881@`882
>S/N比があまりにも高く、アラシがアラシになっていない。

2ちゃんねるでは、これまであまり見られなかった現象のようだ。
観察していると面白い。アラシのアイデアが他にあるかな?

でもそんなことより、Dr. M.Yの書き込みは(気晴らしなんだろうけど)
文章でIQを推定できるってのがよく分かるね。
888>887:2000/10/27(金) 21:23
天に向かって唾す、というのがどういうことかよく分かりますね、
これ見ていると。
889名無しさん@お腹いっぱい。:2000/10/27(金) 21:24
>>887
推定できるほどのIQは君にあるの?
彼の文章からIQを推定するには、君が彼以上のIQである必要があるのでは?
それともただ漠然とIQ高いなって思ってるだけ?(藁
890>887:2000/10/27(金) 21:37

矛先をほかのヤツに向けるという手口があるけど、もう底が見えてる
以上ダメだね、すでにアラシになり得ない。
891MY:2000/10/27(金) 21:40
さて、そろそろ日本に出発でございます。誰かが言ってたけど、ほんとに機内で眠くなるまでレス書き溜めようっと。(^^) 最近のアメリカンのフライトは、全機パワー・アウトレット付きなので二重丸です。

あ、ついでに、俺の時差ぼけ防止法でも..

とにかく出来るだけ機内で眠ること。時差と寝不足が重なると余計ひどくなります。一番いいのは、アルコールと抗ヒスタミン剤(古典的でBBBを透るやつ)を併用して、眠ってしまうことです。話題のメラトニンは、避けたほうが賢明です。理由は:

(1)飲んでも多少眠くなる程度で、効かない。宣伝されている体内時計のリセット効果は、せいぜい30〜60分のシフト程度で、意味なし。
(2)Depressionを誘発するという説あり。
(3)アメリカのスーパーなどで買えるメラトニンは、一部合成品ではなくウシ脳からの抽出物。怖いぞ〜。


それでは、行ってまいります。
892MY:2000/10/27(金) 21:50
MY先生
先生の語り野郎がいます。大変を手数うですが、ハンドルネームに以下の処理を
していただければ、まことに幸いです。

「キャップをかぶせる」って、どういうこと? top age sage ▲ ▼

固定ハンキャップ(なんちゃってニセモノキャップ)を設定することです。
匿名掲示板である為に、人気のあるハンドルネームを持つ人のニセモノが出た際の混乱を避けるためにある機能です。

○○○さんのニセモノは、"○○○"といった風に、名前が"括弧"でくくられます。
ニセモノ被害でお困りの方は、ひろゆきへのメールで申請してください。

その際には、利用プロバイダのメールアドレスと判るかたちお願いします。
リメーラ、転送メール、webメールなどでの申請はお断りする場合があります。
使いかたは、申し込んだ方本人にメールで説明します。

ひろゆきへのメール
mailto:[email protected]
http://www.2ch.net/faq.html
893892:2000/10/27(金) 21:54
HN
MY→MY先生へ
894名無しさん@お腹いっぱい。:2000/10/27(金) 23:38
キャップは今受けつけてたっけ?節穴さんのほうがよくない?
895894:2000/10/27(金) 23:43
げ、ここcocoaだからダメなのか>節穴さん 鬱だ…
896>895:2000/10/27(金) 23:54
cocoaだとだめなの?
897名無しさん@お腹いっぱい。:2000/10/28(土) 01:45
先生、いつ帰ってくるのかなあ。
898名無しさん@お腹いっぱい。:2000/10/28(土) 06:45
あの先生、どこへ行くのにもノート抱えて行くから、演題が退屈だと
学会場から2ちゃんにカキコするかもしれないよ。
899名無しさん@お腹いっぱい。:2000/10/28(土) 07:11
900番を
900名無しさん@お腹いっぱい。:2000/10/28(土) 07:11
もらいます。
901名無しさん@お腹いっぱい。:2000/10/28(土) 07:41
>キャップは今受けつけてたっけ?
NO
902名無しさん@お腹いっぱい。:2000/10/28(土) 08:12
thimpo
903一庶民:2000/10/28(土) 08:39
617の投稿をした者です。
実は書きこむ前に殆ど他の書きこみを読んでなかったもので、
「同じ日本人として誇らしく思う…」という表現には、何の含みもありません。
先生が、そうなのか否かは知りませんが、そういう可能性について思い至らず、
無神経な表現を使った自らの不見識を、恥じております。人の痛みに気付かなかったのは自分の方でした。
ところで、617の投稿の中で、違う分野の研究者…の下りは、嘘でした。
気に懸かっている人が、恐らく被差別側にあって、
自己肯定感を回復すべく今まさにもがいている最中にある、というのが本当のところです。
私には、あんな馬鹿なお願いをするしかなす術もありませんでした。
非礼を詫びたかっただけです。もう消えます、ドロン…
904名無しさん@お腹いっぱい。:2000/10/28(土) 08:52
905田舎脳外科:2000/10/28(土) 09:34
┌─────────────────────────┐
│     ( ̄ ̄)                                 │
│    / 2ch \        投 稿 診 断 結 果         │
│    | Λ_Λ |                            │
│    |( `∀´ )|                   (診断番号 002) │
│    \__/                                │
└─────────────────────────┘
  【投稿者】
    スレッド   : (病院、医者)  柳沢って誰だ?
    お名前   : MY
  【得点】 (100点満点)
    98点
  【チェック項目】
    (レ ) すばらしい。師匠と呼ばせてください
    (  ) 不覚にも笑ってしまいました
    (  ) 自作自演は止めてください
    (  ) ネタなのがばればれです
    ( ) 二度と投稿しないでください
    (  ) おまえもな、そして俺もな
    (  ) コピペ厨房うざい
    (  ) 氏ね、逝ってよし
    (  ) ごみスレ、オマエみたい
   【アドバイス】
    ねーみんな part2へいこうよー。
906元祖ポスドク:2000/10/28(土) 10:15
>>903
なにがいいたいんですか?
907元祖ポスドク:2000/10/28(土) 10:38
ちんちん元気ねー
908名無しさん@お腹いっぱい。:2000/10/28(土) 11:18
>田舎脳外科
おまえ、学会終わったら質問するって言ってただろうが。留学希望なんだろ、学術的な質問しろよ!
909田舎脳外科:2000/10/28(土) 11:35
>908
だって先生いないしパート2で
「Aアンタゴニストとノンスペと両方すぱに
効果あるって脳毛の雑誌にかいてあって
Bアンタゴニストもネズミに効いたってのもあったんですけど
どれがいちばんききそうですか?」
くらいきこうとおもってたの。
いま文献やっと4本目。
あと、おれは虚血屋じゃなくて同門のやつらのために聞いてたの。
過去ログみてね。なんかおこられそうだけど
いまのおれはてーまにも師匠にも金にも不自由してない。
不足してるのは実力とやるきと英語力。
よって当分留学しまへん。
質問期待してくれてありがと。
もちょっとべんきょうしますわ。
910元祖ポスドク:2000/10/28(土) 11:36
905はただの馬鹿
911>909:2000/10/28(土) 12:13
おまえなんて、逆立ちしても入れるラボじゃねーよ。木瓜!
912An Evo-Devo postdoc researcher:2000/10/28(土) 12:50
私はTransgenic mouseを用いて進化発生学を米国で研究しております。

HHMIホームページで先生が進化的なコメントをされているのを見つけ、
驚くとともにうれしく思いました。
>Yanagisawa said the biochemical link between orexin and
>narcolepsy is still a mystery. "But if you just think
>about it philosophically@` it makes sense@`" he added.
>"When an animal gets hungry@` it had better be alert.
>It would be bad from an evolutionary standpoint to be
>sleepy when it's time to hunt for food."

この二つの独立な機能が獲得されたのは進化上いつ頃、とお考えでしょうか?
また、どちらがオリジナルの機能と考えられますでしょうか?
どの生物までorexinの機能がさかのぼってたどれるか、生物としての根元的な
欲求を司るものだけに、非常に興味があります。

失礼で不躾な質問とは思いますが、このような機会は滅多にないもの、
と思わず手が動きました。お許し下さい。
913KIR:2000/10/28(土) 13:21
xenopusとサカナではオレキシンがあるそうですね。
914名無しさん@お腹いっぱい。:2000/10/28(土) 13:38
魚類ではナルコがありうるのでしょうか?
915名無しさん@お腹いっぱい。:2000/10/28(土) 13:40
そういえばサメでは、天地逆(仰向け)にすると一瞬気を失ったように
まったく動かなくなるものがありますが、ちょっとナルコっぽい・・・
916名無しさん@お腹いっぱい。:2000/10/28(土) 15:13
>+jdfajlkj
917名無しさん@お腹いっぱい。:2000/10/28(土) 15:51
>913

そもそも魚は眠るのですか?(レム睡眠ある?)
誰か魚をやってる人、教えて。
918KIR:2000/10/28(土) 16:07
>917
眠るんじゃないですか?
でも、REM睡眠とnonREM睡眠は大脳皮質にある程度厚さがないと区別できないと
おもいますね。

そもそも、オレキシンがあると言っても、魚でも睡眠・覚醒を制御しているとは限らないと思います。
alphaMSHや、MCHなんかは、下等動物では体色を制御するホルモンですが、
ほ乳類では食欲を制御する神経伝達物質になっています。

オレキシンも下等動物の脳では違う役割を持っていてもおかしくありません。
しかし、見事に視床下部に局在しているようですね。
919自作自演君へ:2000/10/28(土) 16:22
自作自演
自作自演
自作自演
自作自演
自作自演
自作自演
自作自演
自作自演
自作自演
自作自演
自作自演
自作自演
自作自演
自作自演
自作自演
自作自演
自作自演
自作自演
自作自演
自作自演
自作自演
自作自演
自作自演
自作自演
自作自演
自作自演
自作自演
自作自演
自作自演
自作自演
自作自演
自作自演
920名無しさん@お腹いっぱい。:2000/10/28(土) 16:34
zebra fishにオレキシンを投与する実験とか(脳内投与はたいへんそうすね・・・)、オレキシンのmutation とかは見つかっているんでしょうか?
921名無しさん@お腹いっぱい。:2000/10/28(土) 16:40
>919

スペースの無駄。
922KIR:2000/10/28(土) 16:51
golden fish(キンギョ)なら脳室内投与した実験がありますよ。
たしか、カナダのグループでしたね。摂食量があがるそうです。
923KIR:2000/10/28(土) 16:55
自作自演だとか書いているヒトがいるので、きっと、目障りなのでしょうから、もう書き込みはやめます。
ご質問などは直接メールでお答えしますので、アドレスを教えてください。では。
924堕ちた研究者@JHU:2000/10/28(土) 16:58
>923
にどとくんな
ハゲの柳沢
そうおめ〜は目障りじゃ
925名無しさん@お腹いっぱい。:2000/10/28(土) 17:13
>924
おめーこそ目障りだ、この低能!バカ
926名無しさん@お腹いっぱい。:2000/10/28(土) 17:17
徹底的に無視しよう。書き込み止める必要ないよ。
927名無しさん@お腹いっぱい。:2000/10/28(土) 17:39
やなちゃん今ごろ飛行機の中かなあ。
928名無しさん@お腹いっぱい。:2000/10/28(土) 17:41
柳沢先生は、11月3日の日本医事倶楽部のパーティーに出席するために帰ったんですか?
929堕ちた研究者@JHU:2000/10/28(土) 17:41
やなちゃんかいな
930MY:2000/10/28(土) 19:10
>>611 >>612 ゴア vs ブッシュ

あはは、出ましたね政治の質問。

アメリカは、日本に比べて(いや世界のあらゆる先進国とくらべて)非常にhyper-capitalisticな国です。アメリカに住んでいると、日本がいかにsocialisticな社会であるかが良く分かります。まず、貧富の差の大きさと、それに伴う社会的経済的階層のコントラスト。日本の大企業で、社長と卒後1年目の新米サラリーマンの給料を比べたら、その比はせいぜい数10倍どまりでしょう?これがアメリカの大企業では、数1000倍にもなり得るのです。昨年ダイムラー・ベンツとクライスラーが合併したときも、両社の社長の年収が10倍以上も違っていたことが話題になりました。逆にアメリカの貧困層は、本当に貧乏です。それに比べたら、日本は(良くも悪くも)本当の金持ちも本当の貧乏人もほとんどいない社会です。たとえばアメリカの犯罪率は、これがある限り、いつまでも下がらないでしょうね。俺の思うに、アメリカ人って、「それでも良いじゃないか」って思う人が多いようです。日本人だったら、例えば「街の安全」は、国民全てにタダで与えられた公共の財産ですよね(もちろん間接的には税金という形でその代金を支払っているが)。でも、アメリカでは、「安全は金を出して買うもの」なのです。

もう言いたいことは判ったと思いますが、(俺自身も含め)平均的な日本生まれの日本人の価値観は、アメリカの基準では圧倒的にDemocrat寄りなのです。俺もその一人です。

PS: 前出で頂いた、キャップを使えばというアドバイスですが、現在キャップ登録は停止中だそうです。
931MY:2000/10/28(土) 19:11
>>596 教官の逆評価について

そうですね、学生が教官を逆評価できるシステムは、個々の教官に、自分の教え方に関するフィードバックを提供するという点で、大事だとおもいます。ただ、それをあまりにも重視するあまり、いわゆる「学生に媚びを売る」教官が増えてしまったのでは、本末転倒ですよね。実際、日本と大学の、特に文系の学部では、そういう教官が少なからずいる、ということも耳にします。まず学生の側に、「真面目に」 逆評価するような態度が培われていないと、何も意味のないことになります。

学生による逆評価だけでなく、言い古されたことですが、現在の日本の大学(とくに国立大学)には、一度教官になってしまった者に対する、強制力のある再評価システムは、全く存在しないと言ってもいいのではないでしょうか。そりゃ、仕事をしなければグラントは来なくなるでしょうが(これもそうでもないのか?)、それでも教官としての身分までは危うくならないし、給料も一定の率で上がり続けるし、社会的地位も全く安泰ですよね。これ、どこかおかしいと思います。
932MY:2000/10/28(土) 19:13
>>520 文献をどこまで読むか

ほう、HHMI Investigatorでも、そういうタイプの方も居るのですねぇ。差し支えなければ、どなたか教えてください。俺の知っているHHMIの人々は、実に勉強家が多いようですので、ちょっと好奇心です。俺自身はというと、まぁこれに関しては中庸でしょうか。いつもいつも文献を読みまくるには自分は怠け者すぎますし(いかんなぁ)、かといって自分の仕事に関係の深い主要論文を無視できるほどの大人物でもありません。

日頃から文献を読むことは、もちろん研究者として極めて大事なことだと思いますよ。ただ、それによって、自分自身の研究がいわゆるfashionに走り、自分のnicheやユニークさを見失ってしまうのでは、本末転倒です。世の中で起こっている重要な事には(自分の分野とは少し距離があっても)精通し、かつ自分自身はデンと構えて我が道を行く、というのが俺の理想の研究者の姿ですね。
933MY:2000/10/28(土) 19:16
>>517 >>675 知的所有権(IP)・特許について

これについては、一家言ありますので長くなりますよ。 生命科学・医薬・バイオ分野(biomedical engineeringを除く)での近年のIPには、大きく分けて4つのタイプがあると思います:

(1)薬理活性のある人工化合物(薬物あるいはその前駆化合物)に対する特許。微生物などからの活性天然化合物(発酵産物など)もここに含めて良いでしょう。
(2)ヒトないしは高等動物由来の天然の生理活性物質およびその製法に対する特許(ほとんどが蛋白質)。例えばリコンビナント・インスリンやリコンビナント・エリスロポエチン、その変異誘導体などがここに入ってきます。
(3)いわゆるドラッグ・ターゲット(内因性の薬物標的分子)に関する特許。例えばエンドセリン受容体・オレキシン受容体とそのリガンド(この場合、内因性生理活性ペプチド)などがこれに当たります。
(4)そこまでの機能特定がなされていない内因性分子に関する特許。例えば、機能のわからない新しいGPCR遺伝子に対する特許。極端には、単なるEST(無名cDNA)に対する特許。

(1)〜(4)へ行くに従って、人工的「発明」の要素が減り、より「発見」の要素が増えて行っていることが分かると思います。問題は、これを何処までIPとして認知すべきか、という点です。

まず(1)と(2)に関しては、そのものが「薬」になりうる物質ということで、堂々とIPとして認められるべきであるということに異論を挟む余地はあまりないと思います。また逆に(4)に関しては、少なくとも国際特許の舞台においては有効な特許として認められない、というのが最近の動向です。一時期これを乱発した米国も、今はその指針に従いつつあるようです。

つづく...
934MY:2000/10/28(土) 19:17
そこで意見の分かれるのが、(3)の範疇です。つまり、機能がかなりハッキリ判っている内因性分子(遺伝子)で、薬物の作用点になりそうなもの、という事です。現在の風潮では、ほとんどの場合、このような薬物標的は正当なIPとして認められていると思います。例えば、エンドセリン系やオレキシン系(受容体およびリガンド)に関する特許が実際に発行されています。

しかし、これは特許法の本来の精神に照らして本当に正しい動向でしょうか?俺自身は、この点おおいに疑問に思っております。特許というのは、なにか世の中の役に立つ物を「作り」あるいは「発明」した場合に、その権利を保護し報酬が生ずるようにして、発明へのmotivationと健全な競争を促すことが、本来の目的だと思います。しかし生命科学分野での現状をみると、製薬業界・アカデミア(公的非営利研究機関)に限らず、この薬物標的としての内因性遺伝子に対する特許を取りまくり、将来発明されるであろう所の、それに対して働く薬物(化合物)までをも自分たちの所有と主張してロイヤリティーを要求する、ということが堂々と行われています。おっしゃるように、特にアメリカでこの動向がピークに達しています。それでいいのだろうか?

俺の考えでは、この動向は決して健全なものではありません。たとえばオレキシン受容体に関しても、TS君の最初のCell paperが、SmithKline Beechem社との共同研究になっているということで、他の製薬会社はオレキシン受容体アンタゴニスト・アゴニストの開発を始めるにあったって、必ず一度は躊躇するようです。製薬企業の研究者の方々と話していると、ふたことめに出てくるのは、「しかしその標的はどこどこの会社が特許をとっているから..」というようなコメントです。薬物標的の同定が、実際に臨床で使えるクスリを作るまでの長い長い道のりのほんの第一歩でしかないことを考えると、この状況はどうみても不健全です。クスリ作りへの健全な競争を阻害しています。ひいては、そのような特許を与えること自体が、社会に対するdisserviceだと思います。

自分がその手の特許を持っていて言うのもなんですが、この薬物標的IP崇拝主義は、近未来にきっと崩れてゆくと思います。必ず揺り戻しが来ますよ。ほんの10年・15年前には、我々の身体に最初から存在ところの薬物標的を「発見」しても、そんなものは特許の対象にならないという考えが主流だったわけで、そちらの方がよほど健全な考え方だと思います。

結論としては(特に >>675 のご質問に関して)、日本の業界は、現在のアメリカ主導のバイオ特許の現況に惑わされず、コツコツとやってゆくべし、という一言ではないでしょうか。 薬物標的を「発見」することと、薬物を実際に「発明」することとは、全く違うレベルの話なのです。まだ負けてもいない戦(いくさ)を、アメリカ側の「hype」に惑わされて、負けたものと勝手に決めつけてはいけませんぞ。
935MY:2000/10/28(土) 19:21
..ということで、残りはまた明日(日曜日)にでも。

う〜む、日本はやはり飯が旨いわい!
936名無しさん@お腹いっぱい。:2000/10/28(土) 19:25
>Dr.MY
心ゆくまで日本の飯を食ってください!!
937あるポスドク:2000/10/28(土) 19:36
本物の柳沢先生がカキコしてくださっているのでうれしいです。
私は先生のセミナーを何回か聞いたポスドクです。
先生が以前、東大医学部で、Endothelin KO mouseとHirschsprung氏病
のセミナーされた時に、お話をお聞きしたのが最初でありました。
当時院生だった私のくだらない質問に丁寧に答えてくださったのが印象的
でした。
また時々このスレをのぞいてみます。
938名無しさん@お腹いっぱい。:2000/10/28(土) 19:37
もう日本に着かれたのですか?
939堕ちた研究者@JHU:2000/10/28(土) 19:50
こら柳沢いまなんじだとおもてんだ。
土曜日の6時AMだよ?
940名無しさん@お腹いっぱい。:2000/10/28(土) 19:56
>939
ボケッ!! 死ね!
941みんなの酒豪ちゃん:2000/10/28(土) 20:12
>940
なんで?
942みんなの酒豪ちゃん:2000/10/28(土) 20:13
いいなあ
先生
和食!!!!!!!!!!!!!
おれはシアトルで残飯でもくってます
943みんなの酒豪ちゃん:2000/10/28(土) 20:14
機内食じゃない?
>938
944みんなの酒豪ちゃん:2000/10/28(土) 20:19
哀れな柳沢
機内食に感動
Dallasの和食はそんなにまずいのか・・・。
シアトルでよかった
945名無しさん@お腹いっぱい。:2000/10/28(土) 20:22
>943
飛行機からネットって出来るの?
946611@`612:2000/10/28(土) 20:23
MY先生、お返事ありがとうございました。先生のお話、全く私も同感です。もちろん私は、アメリカでは選挙権はありませんが、私の周りも(アメリカ人ですら)みんなDemocrat寄りです。でも、最終的にはブッシュが勝つ可能性も高いし、アメリカという国はよくわからないです。
947名無しさん@お腹いっぱい。:2000/10/28(土) 21:47
先生のお話、相変わらず面白いですね。本当にためになります。
948名無しさん@お腹いっぱい。:2000/10/28(土) 21:47
柳沢先生、師匠と呼ばせてください。
949名無しさん@お腹いっぱい。:2000/10/28(土) 22:06
ユダヤ人をどう思いますか?
950名無しさん@お腹いっぱい。:2000/10/28(土) 22:14
はじめまして、楽しく拝見させていただいております。これからの(21世紀の)医学・生物学研究は、どのように展開していくとお考えですか(身勝手な質問で申し訳有りません)?
951名無しさん@お腹いっぱい。:2000/10/28(土) 22:20
>>934
>自分がその手の特許を持っていて言うのもなんですが、

やはり先生御自身が特許を持たれているのですね。特許料とか凄そう・・・。
952名無しさん@お腹いっぱい。:2000/10/28(土) 22:25
記念カキコ
953名無しさん@お腹いっぱい。:2000/10/28(土) 23:33
まとめましょ
954名無しさん@お腹いっぱい。:2000/10/28(土) 23:33
日本へ帰ったら、コンタクトレンズのバイトしますか?
955名無しさん@お腹いっぱい。:2000/10/28(土) 23:34
1000を超えたら表示できなくなるって言うことは、読めなくなるってこと?
956名無しさん@お腹いっぱい。:2000/10/28(土) 23:41
1000越えますので、続きはこちらで。

http://cocoa.2ch.net/test/read.cgi?bbs=hosp&key=972743655

なお柳沢って誰だ?part2(これ↓(は荒らしの立てたスレッドですので無視してください。
http://cocoa.2ch.net/test/read.cgi?bbs=hosp&key=972622446&ls=50
957MY:2000/10/29(日) 03:51
>>956 の方には申し訳ありませんが、「柳沢って誰だ?Part 2」のスレッドのほうが内容が(いまのところ)より真面目なので、そちらに投稿させていただきました。以降、そちらをご覧下さい。尚、俺自身としては、「..誰だ?」のタイトルも気に入っておりますのでお気遣いなく。
958売名行為君へ:2000/10/29(日) 12:00
売名行為
売名行為
売名行為
売名行為
売名行為
売名行為
売名行為
売名行為
売名行為
売名行為
売名行為
売名行為
売名行為
売名行為
売名行為
売名行為
県知事でも狙ってるか?あん?
959名無しさん@お腹いっぱい。:2000/10/29(日) 12:33
>958
妬む人間は醜いですな。ナメクジ以下。
960FORCEPS:2000/10/29(日) 14:32
>959
いいじゃないですか。958は2ちゃんねるならではのカキコですよ。
学会でお偉いさん達に囲まれてしまったDr M.Y.には、下々は話もできない。
ここでは気楽に何でも言えちゃいますからね。そのメリットに伴う副作用でしょう(藁

御本尊がPart IIの方へ移るらしいですね。
このPart Iは保存しとこ。
961○ヌ:2000/10/29(日) 22:01
記念。
962MY:2000/10/31(火) 04:27
963名無しさん@お腹いっぱい。:2000/11/01(水) 04:00
age
964&#xFFFF;&#xFFC1;:2000/11/02(木) 15:16
age
965IKUIKUALDN:2000/11/04(土) 14:40
掲示板見せていただいています。
論文のトピックが出ていたので、もしよろしければ下記のHPをご覧下さい。
11/2日に開設したばかりです。いろいろなご意見、お聞かせ下さい。

http://members.aol.com/ikuikuland/index.htm

966Cr:2000/11/05(日) 11:53
宣伝ヤメロ>965
967名無しさん@お腹いっぱい。:2000/11/05(日) 16:03
age
968名無しさん@お腹いっぱい。:2000/11/06(月) 11:48
age
969へなちょこコロ助子Ver.通院:2000/11/06(月) 11:50
1000もちかいべ
970名無しさん@お腹いっぱい。:2000/11/10(金) 19:28
age
971Part3の解答は:2000/11/11(土) 07:06
>>680の質問です。
972真・スレッドストッパー
書けませんよ( ̄ー ̄)ニヤリッ